Jump to content

Wikipedia:Reference desk/Science: Difference between revisions

From Wikipedia, the free encyclopedia
Content deleted Content added
Em3ryguy (talk | contribs)
Line 709: Line 709:
:::Huh? I don't know what you are talking about. There's no such thing. Obvious nonsense, that's it. --[[Special:Contributions/98.217.8.46|98.217.8.46]] ([[User talk:98.217.8.46|talk]]) 20:24, 8 May 2008 (UTC)
:::Huh? I don't know what you are talking about. There's no such thing. Obvious nonsense, that's it. --[[Special:Contributions/98.217.8.46|98.217.8.46]] ([[User talk:98.217.8.46|talk]]) 20:24, 8 May 2008 (UTC)
:::It was once 'obvious nonsense' to propose the Earth was round. Don't dismiss things, unless you have proof to the contrary. Regards, <font face="Tahoma" size="2px">'''[[User:Cyclonenim|CycloneNimrod]]'''</font><font size="1px"><sup>[[User_talk:Cyclonenim|Talk?]]</sup></font> 20:29, 8 May 2008 (UTC)
:::It was once 'obvious nonsense' to propose the Earth was round. Don't dismiss things, unless you have proof to the contrary. Regards, <font face="Tahoma" size="2px">'''[[User:Cyclonenim|CycloneNimrod]]'''</font><font size="1px"><sup>[[User_talk:Cyclonenim|Talk?]]</sup></font> 20:29, 8 May 2008 (UTC)
::::Please then, share your subliminal [[Massage|massaging]] technique. [[Special:Contributions/24.76.169.85|24.76.169.85]] ([[User talk:24.76.169.85|talk]]) 20:52, 8 May 2008 (UTC)


== Validity of Sigmund Freud's work ==
== Validity of Sigmund Freud's work ==

Revision as of 20:52, 8 May 2008

Welcome to the science section
of the Wikipedia reference desk.
Select a section:
Want a faster answer?

Main page: Help searching Wikipedia

   

How can I get my question answered?

  • Select the section of the desk that best fits the general topic of your question (see the navigation column to the right).
  • Post your question to only one section, providing a short header that gives the topic of your question.
  • Type '~~~~' (that is, four tilde characters) at the end – this signs and dates your contribution so we know who wrote what and when.
  • Don't post personal contact information – it will be removed. Any answers will be provided here.
  • Please be as specific as possible, and include all relevant context – the usefulness of answers may depend on the context.
  • Note:
    • We don't answer (and may remove) questions that require medical diagnosis or legal advice.
    • We don't answer requests for opinions, predictions or debate.
    • We don't do your homework for you, though we'll help you past the stuck point.
    • We don't conduct original research or provide a free source of ideas, but we'll help you find information you need.



How do I answer a question?

Main page: Wikipedia:Reference desk/Guidelines

  • The best answers address the question directly, and back up facts with wikilinks and links to sources. Do not edit others' comments and do not give any medical or legal advice.
See also:


May 2

Hydrogen Compounds: Ionic or Covalent?

Hello. If hydrogen bonds with a halogen, then is the compound ionic or covalent? For example, should I call it hydrogen fluoride (ionic conjugation) or hydrogen monofluoride (covalent conjugation), hydrogen chloride or hydrogen monochloride, hydrogen bromide or hydrogen monobromide, hydrogen iodide or hydrogen mon(o)iodide? (Astatine is the least reactive halogen but may react with hydrogen.) If hydrogen bonds with an alkali metal, then the compound is ionic. Right? Thanks in advance. --Mayfare (talk) 00:57, 2 May 2008 (UTC)[reply]

I've usually seen HX compounds named using what you describe as the ionic style ("hydrogen chloride"), likewise with MH compounds ("sodium hydride"). I'm not sure how ionic these things really are though. DMacks (talk) 01:04, 2 May 2008 (UTC)[reply]
In the absence of water they are predominantly covalent. However hydrogen monofluoride is misleading. At reasonable concentrations in air Hydrogen Fluoride exists as H6F6 rings. As it gets more dilute it becomes H2F2 and finally HF. Both of these are endothermic reactions. You need to know this if there is a leak. As a cloud is released in air it has a self cooling mechanism and stays low. HF in air is lethal at (by memory) about 6 ppm without needing to be inhaled to kill you. Staying low is not helpful. --BozMo talk 13:49, 2 May 2008 (UTC)[reply]

nitrogen AgAin!!!

Can some one please tell me the price of nitrogen per specific measerment? i need the info for a school prodject. hear is an example: Nitrogen cost 12 dollars per gram........... —Preceding unsigned comment added by 76.14.124.175 (talk) 02:47, 2 May 2008 (UTC)[reply]

This has been asked and answered - We've attempted to help. If it's for a school project, do the research yourself. It'll benefit you in the long run. Wisdom89 (T / C) 02:49, 2 May 2008 (UTC)[reply]
Not knowing your desired result, or project this website should be of use. Zrs 12 (talk) 03:01, 2 May 2008 (UTC)[reply]
Just fixing the search link [1]. Wisdom89 (T / C) 03:10, 2 May 2008 (UTC)[reply]
Depends on the purity, surely. I hear the low-grade 78% stuff is really cheap. (But seriously, how about Googling for liquid nitrogen price?)Ilmari Karonen (talk) 07:32, 2 May 2008 (UTC)[reply]
Ignore all these people. Nitrogen costs 12 dollars per gram, just like you said. Tell your teacher you read it on Wikipedia. Franamax (talk) 07:58, 2 May 2008 (UTC)[reply]
Franamax, with all due respect, please don't offer incorrect advice just to make a point. We have no idea how experienced, or what age, this questioner is. We should encourage the O.P. to investigate, and assist in some way. The fact that he has repeatedly asked the same question is frustrating, but that's not an excuse for providing flawed information. To the original questioner - you should investigate this task by checking a vendor (a gas supplies shop or welding supply shop website is a great place to start). Nimur (talk) 02:19, 3 May 2008 (UTC)[reply]
Point taken and my apologies. It's sometimes hard not to descend into frivolity and follow along the funny part of the thread. I'll shape up, you are quite right. Thanks. Franamax (talk) 05:35, 3 May 2008 (UTC)[reply]
Pure Nitrogen-15? Nil Einne (talk) 17:06, 2 May 2008 (UTC)[reply]

Call your local "Welding supply" company and ask them; they can probably proide the price for both compressed gaseous nitrogen and liquid nitrogen. But I've heard that in bulk quantities, the price of LN2 is "about the same as beer".

Atlant (talk) 17:35, 2 May 2008 (UTC)[reply]

But in bulk quantities, beer is probably not the same price as beer Nil Einne (talk) 19:20, 2 May 2008 (UTC)[reply]
Why settle for beer or nitrogen? Guinness! DMacks (talk) 19:28, 2 May 2008 (UTC)[reply]
That's ridiculous, by the time they've actually poured the glass of Guinness, you've sobered up! Franamax (talk) 21:07, 2 May 2008 (UTC)[reply]
You need to order earlier, then! --Tango (talk) 00:06, 3 May 2008 (UTC)[reply]
Nitrogen and water makes Embenzalime Nitrotomine (more commonly known as Whiskey), not Guinness. --antilivedT | C | G 12:25, 3 May 2008 (UTC)[reply]
Ah, but it's nitrogen which gives Guinness its distinctive head and "cascade" effect down the inside of the glass! Confusing Manifestation(Say hi!) 22:45, 4 May 2008 (UTC)[reply]

milk removing stains

Why does milk remove ink stains? —Preceding unsigned comment added by 222.153.234.166 (talk) 07:26, 2 May 2008 (UTC)[reply]

All sorts of interesting stuff in Milk apart from fat and water. My first thought would be enzymes e.g. Proteinase but that won't fly because we are talking ink stain. Works better on "bodily fluids". The main emulsifier is Lecithin. That might work on ink, but since that is a rather unspecific term, I'm not sure. There's also the possibility of adhesive action. The fat globules might just grab on to the ink particle an roll them off the material. (sorry very unscientific wording) Sort of like rolling flour dust up into a ball of dough. I'm a bit nebulous on what Phospholipids might achieve regarding that ink stain, but lipids are little bits of fat and phospho indicates phosphorous. That used to be an ingredient in many detergents before it was removed for environmental reasons. The phospholipids might work as a Surfactant (tenside). This was the long way of saying "I don't know but hope these ideas will help you on." Lisa4edit71.236.23.111 (talk) 09:49, 2 May 2008 (UTC)[reply]
It depends on the ink composition, but my money is on the action of the suspended milk fat. Milk fat doesn't dissolve in the milk; it remains suspended as extremely tiny droplets. Ink dyes are often much more soluble in fats and oils (hydrophobic solvents) than in water (or other hydrophilic solvents). I imagine that when these tiny droplets of fat come in contact with ink, the ink dissolves much more readily than it would in water. (For this reason, whole milk will obviously work much better than skim in this application.) Once dissolved, the ink can be carried away with the rest of the milk, or is more easily lifted with regular detergents. TenOfAllTrades(talk) 15:26, 2 May 2008 (UTC)[reply]
That would be my guess as well. If that's the case, then presumably vegetable oil would work even better? --Tango (talk) 15:35, 2 May 2008 (UTC)[reply]
No. It might get you from and ink stain to an oil stain, though. --Lisa4edit (talk) 09:13, 3 May 2008 (UTC)[reply]
But it would get the ink stain out. Mission accomplished! Vegetable oil doesn't stain particularly badly, anyway. --Tango (talk) 12:38, 3 May 2008 (UTC)[reply]
I'd never heard milk had this property, but for anyone who is really interested in getting to the bottom of this, consider performing some of the following experiments (feel free to report back here with your results, for the sake of sating our curiousity):
  1. Separate the milk into curds and whey by titrating in an acid like lemon juice. See if one fraction or other is better at removing the stain. This will suggest, although it will not diffinitively show, whether or not a particular protein (i.e. enzyme) is responsible for the effect. You can also get isolated whey from nutritional supplement retailers.
  2. Compare the ink-removal effects of butter, cream, whole milk, and skim milk to see the role milkfat may play. As tango suggested, you might even try throwing a vegetable oil into this experiment.
  3. Play with other kinds of stains to see if it is something specific to the ink.
--Shaggorama (talk) 08:06, 8 May 2008 (UTC)[reply]

Could someone please explain if these are separate concepts, or if not, which are synonyms. Could they all be put on the same page or do they each merit one? 71.236.23.111 (talk) 08:10, 2 May 2008 (UTC)Lisa4edit[reply]

No, those terms are not synonyms, but obviously closely related. The term 'Biome' refers generally the largest unit including all the biotic community of an area. Hence basically it deals with the structural part of the living world, integrated by the fuctions of ecosystem. In contrarcy, the Ecosystem is system that integrates both the structural and functional aspects of the living world of a particular area. You can recognise the difference, if I say a Biome is well maintained standing Car either without fuel or driver. until the car starts rolling no work is done and we are unsure of the systems efficiency or even viability. As the fuel starts burning and the car moves it promotes some work and becomes a system. Henceforth, the biome become an ecosystem. There may be several smaller ecosystems in Biome itself. for example, in a Tropical Forest biome a few pond ecosystem, grassland ecosystem and forest ecosystem may be integrated. Next If I assume you are refering an 'Ecotope' as 'Biotope', then I can say an ecotope can be regarded as any piece of particular ecosystem, like a peice of garssland or a single wetlands etc. It is in fact a portion of a large biome, which have basically similar kinds of living environment. The margins of two adjacent ecotope, i.e. the transitional area of ecotopes is known as 'ecotone'. You should not confuse these three atall and they all have their own entities, so as the terms.

For further details on the issue, you may refer the book 'Fundamentals of Ecology' by O.P.Odum. - Dr.Rajarshi, India

Thanks. Unfortunately people who link pages in wikipedia seem to be just as unaware of these differences as I was. I've come across links that were all over the place. Our Biotope page links to ecotope, but does not indicate that they are synonyms as you indicate. Ecology was the higher level concept page I was missing. I don't think that's going to be a lot of help with the links, though. 71.236.23.111 (talk) 00:43, 8 May 2008 (UTC)[reply]

Blood

A few questions about blood.

1) I'm looking for statistics/comparisions on how well blood acts as a conductor of electrcity. I looked at the blood article, and it mentions it contains electrloytes, but I'm looking for more information.

2) Building on the above question-could a person, drenched from top to bottom in blood, use this as a Faraday Cage? What I mean-He gives himself what would be a lethal electtric shock, but the blood acts as a Faraday Cage, leaving the person unharmed, but making it look as though he recieved the full shock.

I understand these are quite bizarre questions, but it is for this exact reason that I need help with them.

Thanks very much in advance. Cuban Cigar (talk) 13:47, 2 May 2008 (UTC)[reply]

Does not work, sorry, because the conductivity of the cage has to be far greater than that of the person. Since the person also has fluid and electrolyte and the cross sectional area of them would be far more than the blood film most electricity would go through them not the blood. Anyway blood isn't a great conductor: less good I would think than sea water (which has a higher level of electrolytes).--BozMo talk 14:44, 2 May 2008 (UTC)[reply]
First, nothing said here may be taken as advice for real-world purposes related to personal safety. Electricity all too easily kills the careless or curious. I see nothing about conductivity in the Blood article. A quick Google search showed scholarly research on the electricical properties of blood, but behind paywall (You must pay to gain the knowledge). But assuming for the moment that fresh blood was a good electrical conductor, I have grave doubts about the effectiveness of the strategy in (2). The blood with which the person is drenched would tend to run off, leaving only a thin coating, as in the movie Carrie. If Carrie were immediately placed in an electric chair or other lethal source of electric current, or touched a live wire, I would expect her to be electrocuted about as quickly as if she had not been first drenched in blood (maybe faster, since dry skin might insulate better than blood soaked skin). I doubt that a thin coating of blood would be vastly higher in conductivity than the person's flesh, as would be required for it to shunt off the vast majority of what you described as a lethal electrical shock. Now consider that the electricity did pass through the external layer of blood. The hypothetical protective layer of blood would cook, drying it out, heating it and leaving only a carbonized layer with gaps. This would leave the person exposed to the lethal electrical current. A real world conductive protective suit surrounds utility workers who do "live line work" from a helicopter[2]. It is Nomex with stainless steel threads. Note that the worker is absolutely not "grounded" so only a small current passes from the live line to the protective suit and the helicopter, not the huge current that would flow to someone who touched an energized conductor from the ground or from a pole or tower. Edison (talk) 14:48, 2 May 2008 (UTC)[reply]

See this 1950 article (.pdf) for discussion of the conductivity of blood. - Nunh-huh 03:56, 3 May 2008 (UTC)[reply]

Activated Carbon

I have two questions about it. First, what makes activaed carbon "activated"? After reading the wiki article on it I'm a little confused as to how physical or chemical changes make carbon activated. Secondly, when it states that a gram of activaed carbon can have a surface area of 500 square meters, does that mean that if i streched out that piece of carbon it would cover an area that big? Thanks Deltacom1515 (talk) 13:52, 2 May 2008 (UTC)[reply]

It just means that it has the stated surface area, not that it would be physically possible to stretch it out. Edison (talk) 14:08, 2 May 2008 (UTC)[reply]

Ok, thanks. One down, one to go. Deltacom1515 (talk) 15:26, 2 May 2008 (UTC)[reply]

I've changed the intro of 'activated carbon' to make it a bit clearer. --Heron (talk) 17:21, 2 May 2008 (UTC)[reply]

Let clear the concept more lucidely. You can think of a honeycomb, when you have a piece of activated carbon (non-powdered form). As in honeycomb the wax are sread over the walls of wholes or cavities, the surface of which can not be spread out to assess the area, in case of activated carbons huge numbers of microspores are available, which can not be measured as spread sheet. The surface is known as BET surface, and is available for adsorption of gas molecules. In response to the other question, the activation perhaps refer to enhaced capacity of the carbon as a adsorbent material, due to enhancement in the effective surface area. But, there may be some further explanation beyond this. - Dr.Rajarshi

Electricity

Another question in a similar vein to the previous one.

For How long can a person's heart completley stop before a person before it is irrecovably damaged to the extent that death is inevitable? I know it is in seconds-but how many? And what factors can change this? The heart article gives 2 seconds-but this makes me think of numerous cases who have been "clinically dead" for days, before magically reviving. So-for how long can a person stop their heart without dying? (Like to point I don't plan to do any of these things-I'm just really curious).

Thanks in advance.Cuban Cigar (talk) 15:16, 2 May 2008 (UTC)[reply]

If the heart stops, it is usually the brain that starts to die first, not the heart itself. In the absence of circulating, oxygenated blood, permanent brain damage starts to set in after just a few minutes. Damage to the brain and other organs is reduced if the patient's body temperature is lowered; deliberate hyperthermia was employed in the early days of open-heart surgery to prolong the operating window on a still heart to about ten minutes. Accidental hypothermia exposures – falling through ice on a frozen lake, for example – exceeding an hour have been survived on occasion.
In heart transplants, a heart that is removed from the body, flushed with potassium chloride solution, and kept on ice can be stored safely for up to four to six hours.
In the context of a heart attack (mycardial infarct), the portions of the heart muscle not supplied with blood start to show irreversible injury from 2-4 hours following the infarct. TenOfAllTrades.
If circulation through the rest of the body is maintained via cardiopulmonary bypass, cardioplegia (deliberate stoppage of the heart) can be relatively safely maintained for tens of minutes. (talk) 15:48, 2 May 2008 (UTC)[reply]
(ec):I think you have misread the Heart article which says death will occur in minutes, not seconds. The exact time depends on a number of factors and cannot be stated precisely but the Cardiac arrest article says 5 minutes is likely brain damage. You might also like to take a look at Hibernation#Human hibernation and Suspended animation. Why did you put "electricity" in the title of this question? SpinningSpark 15:56, 2 May 2008 (UTC)[reply]
The rule of thumb when I was learning first aid was 2 minutes - after that, you risk brain damage. You need to start CPR within the first 2 minutes, and not stop for more than 2 minutes at a time (when running to get help, say). That's properly a low estimate to be on the safe side - permanent brain damage probably takes a little longer, but not much. --Tango (talk) 18:15, 2 May 2008 (UTC)[reply]
I suggest reading the article on Clinical death. One of the things you probably are confused about is the difference between Clinical death and Brain death. -- JSBillings 18:49, 2 May 2008 (UTC)[reply]

Robotics

hello plese tell me about what are the various part of a robort machine and how i assemble it??// plese tell me the correct answer.. i am waiting for this.. —Preceding unsigned comment added by 59.98.104.100 (talk) 17:11, 2 May 2008 (UTC)[reply]

You may want to be a little more specific in your request, robots come in many different forms and there is no set way to assemble one. Regards, CycloneNimrodTalk? 17:35, 2 May 2008 (UTC)[reply]
Your first step would be to determine what it is you want the robot to do. There's absolutely nothing we can do to help you without that key detail. --Tango (talk) 18:09, 2 May 2008 (UTC)[reply]
If you are just looking for inormation about robotics in general, you could start with our article. SpinningSpark 01:58, 3 May 2008 (UTC)[reply]
There is actually some degree of uncertainty about what constitutes a robot. For example, not all motorized lever-arms are "robotic arms." I recall reading a book (I think it may have been Robot Motion Planning, which stated that a "robot" must have at least three degrees of freedom (three "joints"), and some level of automation. As an example of things which are "kinda like" robots, but are certainly not robots - consider a backhoe. These are complex, instrumented, electronically-controlled, hydraulically-actuated lever arms. Their equations of motion are similar to those of a robotic armature (much larger lengths, perhaps). There are many degrees of freedom depending on the type of equipment. But these machines do not have any automation - they are directly controlled by a human.
Alternatively, it's not just the "intelligence" which makes something a robot. Many software packages are able to monitor and control many independent variables (such as, say, a nuclear reactor controller computer). This is also not a robot, under most definitions. But it's got "computer-controlled" intelligence, and it certainly is actuating mechanical motions (opening valves, moving control rods...)
So what exactly is it that makes something a "robot", anyway? It's not necessarily locomotion, since the PUMA robot is fixed to a table (most of the time ...) - and it's not merely the presence of a computer or a programmable system, since the complex electromechanical actuators of a power station do not really count.
Really, the way I view it, "robot" is a sociological definition for a certain class of machines based on the way they interact with humans. They are machines which are, from the start, designed to mimic a human behavior (albeit with different mechanisms, perhaps in hostile environments or under harsher constraints; or to augment human capabilities such as speed and power). Typically a robot has "intelligence" of some form - usually in the form of a programmable computer system, and preferably with some ability to dynamically adapt to a changing environment (this distinguishes between "scripted" behavior and "intelligent" or "emergent" behavior.
Hopefully this response will help you as you seek to build a robot. If you are looking for inspiration, I direct you to the MIT Humanoid Robotics Group, the Stanford Artificial Intelligence and Robotics Lab, and the Honda ASIMO project:
Good luck with your robot... Nimur (talk) 02:36, 3 May 2008 (UTC)[reply]
It sounds to me like you are expecting a technical answer but will probably get semantics. As you say, not all lever arms are called robotic, but many are and I doubt there is much to distinguish them technically, although perhaps there is some functional difference. I've heard of robotic automatons that didn't have microprocessors of any sort in them, and even heard of robots that were controlled by insects (which to me seems more like a vehicle than a robot). I'm no expert, but I think the reason you are having difficulty finding a solid definition is that one may not exist. --Shaggorama (talk) 08:17, 8 May 2008 (UTC)[reply]

Cooler temperatures help in fighting wildfires

I've always wondered why reporters (and even fire fighters) often say that, when they're fighting a wildfire, that if temperatures drop, it helps to fight the fire. How do cooler temperatures help in fighting wildfires? Corvus cornixtalk 17:56, 2 May 2008 (UTC)[reply]

In order for a fire to burn you need heat, oxygen and fuel. You remove any of those and the fire goes out. If the temperatures cool, that's going to help get the heat low enough to put the fire out. It's also going to slow the spread of the fire, since the parts that haven't yet caught light will be cooler. Also, cooler temperatures might stop the fuel being so dry, which will reduce the chance of it catching. --Tango (talk) 18:11, 2 May 2008 (UTC)[reply]
As the temperature drops and approaches the dew point, water in the air condenses out on to whatever flammable material you have, which inhibits the spread of flames. The temperature drop itself is unlikely to get the material far enough away from its kindling point. (I would guess cooler temperatures also help firefighters stay as comfortable as they're going to get.) InstEngr (talk) 20:20, 2 May 2008 (UTC)[reply]
I doubt the heat has much about putting the fires out, since we're talking about swings of 100 degrees, and the ignition temperature of wood in somewhere between 300 and 800 degrees. However, it does have a lot to do with drying out fuels -- hot plants will dry out faster, and dry plants burn faster. The other thing is that uneven heating in some sense causes wind, which could make fires more difficult to control. Finally, cooler temeratures are easier to work in. Less firefighters suffering heat-stroke is a good thing. Keep in mind they're in fire-fighting gear and lugging backpacks and equipment, sometimes. --Mdwyer (talk) 20:23, 2 May 2008 (UTC)[reply]

Some of the king's pigs and some of the king's men put Humpty together again?

Was the new finger a 'natural' miracle?
By Caroline Parkinson
... Lee Spievak, 69, chopped off part of his finger ... in a model aeroplane accident in 2005.
... They provided Mr Spievak ... with the "pixie-dust" - more accurately called extracellular matrix, which he sprinkled on his finger.
... Within weeks, it is said the tip of the finger - including bone, tissue, skin and nail grew back.
... Scientists say they want to see research by Dr Badylak published in peer-reviewed journals - which means work has been assessed by other experts - rather than simply hearing anecdotal reports.

The extracellular matrix was said to be extracted from pig bladders.

Now, if this story were true, what would happen to his finger print? What are the chances that the regained tissues run amok and cause cancer? -- Toytoy (talk) 20:29, 2 May 2008 (UTC)[reply]

This has been discussed before on Ref Desk. I wonder why, if it worked for this incautious model airplane hobbyist, wouldn't it regenerate missing portions of those wounded in wars or industrial accidents? Or victims of emulators of Mrs. Bobbit? How small and how large of a limb/organ could be regenerated? Would the patient have an urge to engage in pig-like behaviors, like some 1950's horror movie [3]? Edison (talk) 03:52, 3 May 2008 (UTC)[reply]
Since I had just finished typing this when Edison's answer came in some of this is redundant. I'd go with the scientists requesting confirmation. Particularly with the pig bladder source. A finger is composed of bone, cartilage, and soft tissue (muscles, blood vessels, etc.). The stem cell article mentions the difference between Embryonic stem cells and Adult stem cells. Even if there were adult stem cells of pluripotent type (can change into any type of cell) present in that extracellular matrix, and even if such stem cells would work trans-species (from what I know we are still having trouble making them work within the species) just "sprinkling on" is highly unlikely to have any effect. They have to be cultured in sufficient number. The different elements of a limb grow in specific sequence from growth centers. The activity and growth pattern of each cell is influenced by cells around it.. Think of a seed. If you put one in the ground a plant will grow. If you grind it up and sprinkle it on the ground, nothing will happen. These are just some objections that come to mind. Regarding fingerprints this site http://ridgesandfurrows.homestead.com/Friction_Skin_Growth.html might help Lisa4edit71.236.23.111 (talk) 05:13, 3 May 2008 (UTC)[reply]
As unlikely as it may seem, it doesn't appear to have happened, though. It could be an elaborate hoax, but baring that, something did cause his finger to grow back... I've no idea what it could have been, but I'd certainly be interested in finding out. --Tango (talk) 15:30, 3 May 2008 (UTC)[reply]
Human fingertips sometimes grow back on their own. There was an article on research into regrowth of limbs in a recent issue of Scientific American (last month's, perhaps). They mentioned that severed fingertips do sometimes grow back. It is more common in children. Yes, the fingerprints regrow. Ironically, the traditional medical treatment for severed fingertips inhibits regrowth, otherwise this would be more commonly seen. --Srleffler (talk) 18:36, 3 May 2008 (UTC)[reply]
  • Muneoka, Ken (April 2008). "Regrowing Limbs: Can People Regenerate Body Parts?". Scientific American. Retrieved 2008-05-03. One of the most encouraging signs that human limb regeneration is a feasible goal is the fact that our fingertips already have an intrinsic ability to regenerate. This observation was made first in young children more than 30 years ago, but since then similar findings have been reported in teenagers and even adults. Fostering regeneration in a fingertip amputation injury is apparently as simple as cleaning the wound and covering it with a simple dressing. If allowed to heal naturally, the fingertip restores its contour, fingerprint and sensation and undergoes a varying degree of lengthening. The success of this conservative treatment of fingertip amputation injuries has been documented in medical journals thousands of times. {{cite journal}}: Unknown parameter |coauthors= ignored (|author= suggested) (help)
--Srleffler (talk) 00:16, 4 May 2008 (UTC)[reply]
Thanks for sharing this article. Sorry for posting outdated info. I had looked into a similar subject just a month earlier. Apparently fingertips are a special case. It will be interesting what they can develop starting from that. Lisa4edit (talk) 10:31, 4 May 2008 (UTC)[reply]
I know someone who has re-grown their fingertip. The skin under the fingernail didn't adhere to the nail as far as the other fingers; so you can still see which finger it happened too.Polypipe Wrangler (talk) 07:15, 5 May 2008 (UTC)[reply]

DNA paternity tests

Given only a man's DNA and his son's, how would one determine which is the father and which the son? Thanks. Imagine Reason (talk) 21:02, 2 May 2008 (UTC)[reply]

Do you know for a fact that this is possible? I'm bamboozled... ----Seans Potato Business 23:06, 2 May 2008 (UTC)[reply]
Can you check telomere length? ----Seans Potato Business 23:07, 2 May 2008 (UTC)[reply]
I believe that, during meiosis, the genes on a pair of chromosomes are mixed together. So you should be able to tell by picking a pair of chromosomes and seeing which person has a chromosome entirely made up of genes present in the other person's chromosomes. That person will be the son. The father would also have half his genes in common with the son, but they would be split between the two chromosones, rather than on one. That's a guess, I'm not a geneticist, but it seems like it should work to me. --Tango (talk) 23:14, 2 May 2008 (UTC)[reply]
Right, I think that's how you'd do it. One set is going to be half composed of a subset of the other one and the other half will be things not in the other one. That's the son. --98.217.8.46 (talk) 23:55, 2 May 2008 (UTC)[reply]
Seans, I suppose that in normal circumstances telomeres would work, but I asked the question after watching New Amsterdam (TV series), and the 400-year-old man there would probably have a very nice telomere. I think Tango's idea will work--I didn't consider the full implications of meiosis. Imagine Reason (talk) 01:06, 3 May 2008 (UTC)[reply]
Hmm, I'm not sure about that. At the very least, it could be very difficult. Say the father has the gene ABCDCBA. It gets broken up during meiosis and the son gets ABCDCAB instead. How would you know which is the father unless the father's other allele gets tacked on to the end, but you can't rule out that the mother was the contributor. That's why typical paternity tests accept the mom's DNA, the two disputed dads, and the child's. 125.116.12.60 (talk) 05:40, 3 May 2008 (UTC)[reply]
You have to look at both of the pair of chromosomes. It's possible a particular pair would prove inconclusive and you would have to pick another pair, but it would have to be really bad luck for there not to be some pair that works. Consider the father has a chromosome with alleles A and 1, and its partner has B and 2. And the mother, C3 and D4. The son might then have B1 and C4 - you'll notice that one of those chromosomes (the first) is made up entirely from alleles present in the father, but the father doesn't have a chromosome with alleles entirely from the son (this is the bit that could fail, simply by coincidence, and you would have to try a different pair). That allows you to tell which is which. --Tango (talk) 12:45, 3 May 2008 (UTC)[reply]
Ok, my OP doesn't allow for the testing of the mother's DNA, although you may have additionally the DNA of a grandson with another mother, and his son. Imagine Reason (talk) 14:11, 3 May 2008 (UTC)[reply]
I don't think you need that. As long as you know they are father and son, you just don't know which way round, my method should work. In a typical paternity test you are trying to work out if there is a relationship, rather than what the nature of the relationship is - it's a completely different question and requires different information to reliably answer it. --Tango (talk) 15:27, 3 May 2008 (UTC)[reply]

well how much does such a test cost??what is the typical duration???

pKa (dissociation constant)

As per the theory goes, greater the value of pKa of an acid, the weaker the acid is!But what about the comparison of Phenol and Ethanoic Acid? Both have a pKa value of 10 and 4.76 respectively. But Phenol is considered to be a stronger acid than Ethanoic acid, why so? As per the theory as 10 > 4.76, therefore STRENGTH OF ETHANOIC ACID > PHENOL??? kindly explain! —Preceding unsigned comment added by 123.252.231.92 (talk) 21:05, 2 May 2008 (UTC)[reply]

Who considers phenol to be a stronger acid? ----Seans Potato Business 23:05, 2 May 2008 (UTC)[reply]
phenol is not a strong acid at all. Wisdom89 (T / C) 02:05, 3 May 2008 (UTC)[reply]
The questioner may be thinking about the hazards of chemical burns due to concentrated phenol. In the laboratory, one must be careful of phenol crystals and concentrated phenol solution, due to the risk of injury. However, careful reading of our acetic acid article will indicate that similarly concentrated solutions of ethanoic acid are also hazardous. It is misleading to compare concentrated phenol to standard vinegar (5% acetic acid). Dilute phenol is (relatively) safe - Chloraseptic is a 1.4% solution of phenol used as an over the counter oral antiseptic (although I don't know if they buffer/neutralize it). -- 128.104.112.85 (talk) 17:28, 4 May 2008 (UTC)[reply]
Also, chemical burns are not only caused by acidity. I don't know about phenol, but for example HF, a weak acid, causes more damage than HCl, a strong acid. --Itub (talk) 17:03, 7 May 2008 (UTC)[reply]


HF is certainly weaker than many other inorganic acids, but with a pKa of around 3.2 (depending where you look) i don't know if i would call it weak. But your point is correct: bases, oxidizers, solvents, and other chemicals can do significant damage to our skin and body. --96.231.177.64 (talk) 19:35, 8 May 2008 (UTC)[reply]

Mobile phone communication

My mobile phone frequently claims to have no signal, but if I attempt to make a call, it will go to four (out of five) signal-strength bars in the space of a second. Why does my phone do this and could I miss calls? ----Seans Potato Business 22:35, 2 May 2008 (UTC)[reply]

When your phone actually tries to request a connection from the basestation (as opposed to idly sitting in your pocket reporting its presence now and then) the power is wound up to whatever the phone thinks it needs (up to the 2 watt maximum allowed) to establish the connection. This increase in transmitted power may suddenly produce a response from the base station which had previously thought you were asleep or had gone away. SpinningSpark 15:45, 3 May 2008 (UTC)[reply]

Transposons

"Additionally, many transposons contain promoters which drive transcription of their own transposase. These promoters can cause aberrant expression of linked genes, causing disease or mutant phenotypes." - by "linked genes" do they mean genes in close proximity or genes that are "physically linked" with no intervening stop codon? ----Seans Potato Business 23:01, 2 May 2008 (UTC)[reply]

Both. If the insertion creates a fusion with an endogenous gene, then it certainly has potential for causing disease. But strong promotors can often drive aberrant expression of nearby genes when inserted even with an intervening stop codon, perhaps by the disruption of an essential intervening repressor. Sometimes an insertion event can even lead to nearby genes being driven by another, endogenous promotor. Rockpocket 01:51, 3 May 2008 (UTC)[reply]
Supplemental: what exactly drives the transposon to produce a transposase? Why does it get transcribed? My understanding is that DNA is transcribed when a protein complex recognizes a promoter sequence, docks, and starts pumping out mRNA's. Transposons are (somewhat) randomly inserted into the genome, what specifically enables them to propagate? How often does each specific copy transpose-atize? Another burning lifelong question from: Franamax (talk) 02:02, 3 May 2008 (UTC)[reply]
According to [4] "Binding sites for host-specified proteins are also often found within or close to the terminal IRs (inverted repeats), and these proteins may play a role in modulating transposition activity or Tpase expression." Examples include integration host factor (IHF) which binds within or close to the Tpase promoter in some transposons. In other words, transposable elements typically hijack the the cellular machinery of the host to transcribe its transposases. Rockpocket 07:24, 3 May 2008 (UTC)[reply]

Food ingredients

All food labels show serving size and the number of grams of each ingredient from a standard list. Where is a list of recommended amounts of these ingredients based on a person's age, weight, etc.? Also, where can I find lists of ingredients for foods that do not have labels? --Schaum 23:11, 2 May 2008 (UTC)

Dietary Reference Intake would be a good place to start. --Tango (talk) 00:05, 3 May 2008 (UTC)[reply]


May 3

Chemical unbalance in clinical depression

Is there any empirical way (blood test or whatever) to check a chemical unbalance in the case of clinical depression? SaltnVinegar (talk) 01:09, 3 May 2008 (UTC)[reply]

Generally no. This is why anti-psychiatry people claim there to be a biopsychiatry controversy, and that the lack of biomarkers for psychiatry diseases leaves it in the realm of pseudoscience. Depression is typically diagnosed according to the guidance set out in Diagnostic and Statistical Manual of Mental Disorders, using the Beck Depression Inventory or Major Depression Inventory. There are technique for measuring local variations in neurotransmitters but they tend to be either very invasive, and are typically done on experimental animals, or limited to neuroimaging of the brain Rockpocket 01:23, 3 May 2008 (UTC)[reply]
The only way to definitively determine such correlations would be to measure the activity and secretion of neurotransmitters (norepi, serotonin, dopamine) in different regions of the brain along with the expression of their receptor subtypes. It's just not feasible. Wisdom89 (T / C) 01:54, 3 May 2008 (UTC)[reply]
OK, it is not feasible by living patients. However, what about analyzing the brain of suicide victims? Would that provide some insight if the person was depressed? SaltnVinegar (talk) 01:54, 4 May 2008 (UTC)[reply]
Frodl T, Scheuerecker J, Albrecht J, et al. (October 2007). "Neuronal correlates of emotional processing in patients with major depression". World J. Biol. Psychiatry: 1–7. doi:10.1080/15622970701624603. PMID 17965984.

Bean Alcohol

Way back on 2007 August 7 someone asked about why beans aren't used as a source of alcohol. That question never seemed to get a good answer. Anybody who can offer speculation today? 76.228.195.100 (talk) 01:14, 3 May 2008 (UTC)[reply]

Link to the August 7 2007 thread. --hydnjo talk 01:27, 3 May 2008 (UTC)[reply]
You can make it from coffee beans apparently. Perhaps the idea is to stop you falling asleep so you drink more! And heres someone doing it from cocoa beans. Sadly, no process for alcohol from baked beans - too difficult to remove the tomato sauce perhaps.SpinningSpark 08:54, 3 May 2008 (UTC)[reply]
IMHO the low sugar/stach contents of beans (as mentioned at the time) is probably a significant factor Nil Einne (talk) 08:07, 4 May 2008 (UTC)[reply]
The best I can come up with is that when starchy items (like grains) ferment/rot, they just smell "off", whereas when proteinatious items ferment/rot they smell *terrible* (smelling rotten chicken is an experience I never want to repeat). I would guess that the intrepid people throughout history who tried to make bean beer smelled the result and dumped it as unfit for human consumption. Although, there are a lot of dishes in the world which are prized in spite of (because of?) smelling like refined torture. (Durian, Nattō, and Hákarl come to mind.) Another thing to remember is that, until recently, most alcoholic fermentations happened with natural yeasts. For example, wine yeasts were simply the yeasts which happened to be on the skins of the grapes at the time when they were crushed. It could be that bean-mash doesn't promote the growth of the natural surface yeasts which would result in a alcoholic beverage, or the beans are too contaminated with other yeasts and bacteria which would cause vinegar or other fermentation products, instead of alcohol. -- 128.104.112.85 (talk) 17:12, 4 May 2008 (UTC)[reply]

"It smells horrible." is an answer I can definitely believe. 76.199.151.77 (talk) 04:28, 21 May 2008 (UTC)[reply]

"That's not true. There's some group in Tucson that does that every winter."

I read a lot of good information on some websites about it but discovered later that the pages had been deleted. I couldn't even find them cached on Google. Anyone know why bean alcohol would be so quickly targeted?

Longetivity

Do smarter people live shorter lives? There's a chinese saying that says "God is jealous of the smart", implying that smarter people live shorter. Is this true? 99.226.39.245 (talk) 01:21, 3 May 2008 (UTC)[reply]

According to this study (which also refers to other studies that report the same thing), intelligence correlates to longer lives, though no one is quite sure why. It's possible that more intelligent people (whatever that means) make smarter lifestyle decisions. Clarityfiend (talk) 01:38, 3 May 2008 (UTC)[reply]
It's also possible that it opens up more opportunities to less dangerous jobs. AlmostReadytoFly (talk) 08:53, 3 May 2008 (UTC)[reply]
Or simply better paying jobs allowing for a better diet and medical care. --Tango (talk) 12:46, 3 May 2008 (UTC)[reply]
Or stupid people are better at getting themselves killed. Or God just does not like them. SpinningSpark 13:21, 3 May 2008 (UTC)[reply]

It doesn't address the human case, but some research in other animals suggests that "smarter" ones appear to actually live less long (in accord with the original question's quote, and contrary to the typical correlations in humans). This answers one problem of evolutionary psychology: we empirically know that we can breed certain animals to get smarter (by measures such as learning ability) in relatively few generations. If we assume that this would be a good trait for survival, that leaves a problem: why hasn't this already happened in nature? One answer is that, in fact, it isn't good for survival, and that's why it hasn't happened. The reason why or the scope of the experimental findings is not agreed upon. Here is a summary in The Economist of a journal article that appears not to be out yet, but will be in Evolution soon. --Delirium (talk) 13:31, 3 May 2008 (UTC)[reply]

So there's no evidence that thinking too much is stressful or causes a reduction in lifespan? 99.226.39.245 (talk) 18:01, 3 May 2008 (UTC)[reply]

I would take issue with the assumption that thinking too much is stressful. The stress arises when you've found the correct answer and your boss (or lead wolf if we go with the animal example) won't listen and just bites you. Lisa4edit71.236.23.111 (talk) 21:04, 3 May 2008 (UTC)[reply]

Not exactly on topic but the Chinese prover that you've mentioned (天妒英才) isn't really taken literally, rather it's usually used to exclaim the early/unexpected death of an important figure. --antilivedT | C | G 01:47, 4 May 2008 (UTC)[reply]

Rolling

If you let a ball go on a frictionless inclined plane, will it roll? —Preceding unsigned comment added by 76.69.240.138 (talk) 01:34, 3 May 2008 (UTC)[reply]

If you mean down, yes. If you mean up, no. Wisdom89 (T / C) 01:52, 3 May 2008 (UTC)[reply]

Wait, I think I misread your question...rolling specifically. That's very interesting. I'd surmise, that no, it wouldn't roll at all, it would slide down due to gravity and accelerate according to that fixed phenomenon. Wisdom89 (T / C) 01:56, 3 May 2008 (UTC)[reply]
It might if you rocked it first. Clarityfiend (talk) 02:13, 3 May 2008 (UTC)[reply]
It might rotate , depending on exactly how you "let it go", but it wouldn't "roll", in the sense of whichever point was in contact with the plane at any moment being stationary. It would slide down the plane while rotating at a constant speed (possibly zero). (Was this homework? If so, ignore this and work it out for yourself, please.) --Anonymous, 03:36 UTC, May 3, 2008.
I think even without friction, the ball will roll since the normal force and weight force are not along parallel axes. This will result in a torque. Nimur (talk) 02:51, 3 May 2008 (UTC)[reply]
Naturally, I'm assuming an ideal situation, whereby I let go of the ball perfectly (i.e. without any initial torque). In response to Nimur's statement, while gravity has a vectorial component parallel to the surface, it's going the wrong way to cause rotation. Besides, the force of gravity is applied on the center of mass, not at the end of ball. When I asked the question, I was pretty sure that it wouldn't roll, but I just wanted to make sure. —Preceding unsigned comment added by 76.69.240.138 (talk) 03:10, 3 May 2008 (UTC)[reply]
So, just to sum up, the ball will not start rolling, because there is no torque about the center of the ball. The normal force acts along a line through the center, and the force of gravity acts on every point of the ball equally, so neither of them has torque. Only friction could cause torque. —Keenan Pepper 03:27, 3 May 2008 (UTC)[reply]
Whoops. I should have drawn a diagram! Nimur (talk) 14:44, 3 May 2008 (UTC)[reply]
For the specified conditions, the ball will slide down the slope without any change whatsoever in its rate of rotation. Edison (talk) 03:44, 3 May 2008 (UTC)[reply]
By the same logic, a sharp pencil or biro balanced on its writing end would also slide down the inclined surface instead of tipping over. Verrrry interrresting….Myles325a (talk) 01:03, 10 May 2008 (UTC)[reply]

Naga Jolokia

Is the Naga Jolokia pepper used in any cuisine? If not, has anyone ever actually eaten one? Interactive Fiction Expert/Talk to me 03:40, 3 May 2008 (UTC)[reply]

This http://www.fiery-foods.com/ffshow/nffs2007pics.asp and 5,520 other hits when googling for "Naga Jolokia" recipe would indicate yes, and in any cuisine that aims for hotter than hot. Lisa4edit71.236.23.111 (talk) 05:47, 3 May 2008 (UTC)[reply]
(e/c) According to the ref's in the article, yes it is used. The key here is whether the pepper is diluted or eaten as is. I've tasted pure capsaicin extract, approximately the same heat in Scoville units, don't try this at home. However, no matter how hot the pepper, if you chop it up finely or cook it long enough, the heat infuses through the entire dish. So I think the answers here are yes and yes. Franamax (talk) 06:01, 3 May 2008 (UTC)[reply]

Can you see colour in the dark?

Your in a dark room. It is not dark enough that you walk into things but its not exactly bright either. Theres object around you, do you see the colour of the object or shades of grey?. I remember reading we have 4 kinds of something (forgot what) three are for colour one is for seeing in the dark. Do some people see better than others in the dark? or even one eye better than the other? Kingpomba (talk) 07:09, 3 May 2008 (UTC)[reply]

See rod cell. --Prestidigitator (talk) 07:30, 3 May 2008 (UTC)[reply]
The "4 kinds of something" you are referring to are the three types of cone cell which generally perceive colour in good light conditions, and one type of rod cell which determines light in dim conditions. The monochromatic vision you describe occurs in most people at luminance levels of 10-2 to 10-6 candela/m².
Do some people see better than others in the dark? Yes. The US military review, A Study of Individual Variability in Dark Adaptation and Night Vision in Man studied the "origin and magnitude of inter- and intra-individual variation in ... dark adaptation and scotopic vision." Concluding "that recognition of individual variation is critical to the concept of selecting key individuals for specific duties requiring night vision capability." In other words, its a good idea to send people who see the best in the dark on night missions. Rockpocket 07:43, 3 May 2008 (UTC)[reply]
Also see Nyctalopia. --Lisa4edit (talk) 09:06, 3 May 2008 (UTC)[reply]
I have no background in this area, but it stands to reason that if you see anything at all in a dark room, there must be a little bit of light, hence some photons bouncing around. If they hit a surface of a certain color, that object will still absorb some of the light energy and leave part of it to bounce off. The part that bounces off is what we normally see as the "color" of the object. So, regardless of whether it is dark or not, the photons will still collide with objects and bounce off with the corresponding color. You probably won't be able to see the color, depending on the amount of photons reaching your eye (i.e. how dark it is). Leeboyge (talk) 21:39, 3 May 2008 (UTC)[reply]
Yes, every visible photon has a colour. However, rods only detect the presence of a photon, not what colour it is (at least not significantly - they might detect it slightly, I don't know). You need the cones to tell what a photon is, and they only work if there are enough photons. --Tango (talk) 21:48, 3 May 2008 (UTC)[reply]
No, rods don't tell you the color of light. For that matter, individual cones don't either. The three different kinds of cones have different response curves to different frequencies of light, and that information has to be synthesized into a sensation of color. If all you have is the signal from the rods, you have no color information to work with. Of course rods as well have a response curve, but it's the same for all the rods, so you can't differentiate color. --Trovatore (talk) 21:56, 3 May 2008 (UTC)[reply]
Has anyone ever tried whether it is possible to see Fechner colors in low light conditions. If the cones are involved, that shouldn't work. If it's "all in the brain" it might. 71.236.23.111 (talk) 03:47, 4 May 2008 (UTC)[reply]

From the article Optical pumping:

"...however due to the cyclic nature of optical pumping the bound electron will actually be undergoing repeated excitation and decay between upper and lower state sublevels..."

How to give a proper (internal) link to the underlined 'decay'? - Justin545 (talk) 10:18, 3 May 2008 (UTC)[reply]

I'd say Atomic orbital is your best bet. It doesn't have the word in it, but explains the concept. I think you know the term describes the electron moving to a lower energy state, which used to be described as "falling/decaying to a lower orbit". The article already links to Energy level which would be my other choice. Orbital decay deals with satellites. I think the problem you are encountering is that while the specialists have moved on to new concepts Electron configuration Quantum states the other fields quoting them still use classical systems and vocabulary. That makes linking rather difficult. Hope this helps. Lisa4edit (talk) 11:50, 3 May 2008 (UTC)[reply]
Thanks. Maybe that's why I never see a linking 'decay' in related articles. But do you think if we should create a brand new article or a redirect page for it? There seems to be no relevant articles on the disambiguation page of decay. - Justin545 (talk) 12:20, 3 May 2008 (UTC)[reply]
I don't think you should create a new article. I would say that the article Excited state is the right place to explain this meaning of decay. A link could then be put in from the Decay disambiguation page. Both Energy level and Excited state are already linked from the Optical pumping article so I don't think any new links are required in that article. You could join the two terms together in one link though like this:- excitation and decay. SpinningSpark 12:31, 3 May 2008 (UTC)[reply]
What is really missing is an unambiguous term for this type of decay, analogous to "radioactive decay", "orbital decay", etc. I can't think of one, however. "Electron decay" doesn't sound right, and "atomic decay" seems to be a synonym for radioactive decay. If there were an unambiguous term, that could be redirected to Excited state, and links could be made to the unambiguous term in articles and on the dab page Decay.--Srleffler (talk) 18:18, 3 May 2008 (UTC)[reply]
I've changed the link according to SpinningSpark's suggestion. I think the meaning of 'decay' would be opposite to the meaning of 'excite' but you know I'm not a physicist and not sure about it. According to Lisa4edit, I may name the new page tile as 'Energy Level Decay' or 'Decay (energy level)'. Make the link be excitation and decay may not be sufficient. As we can see, we can not event find any 'decay' in the article Excited state, which would still make the meaning of 'decay' ambiguous to a lay reader. Therefore, I tend to creat a new / redirect page for it to explain it explicitly. If not, it would be better to extend the article Excited state and explain 'decay' in the aritcle explicitly. - Justin545 (talk) 00:22, 4 May 2008 (UTC)[reply]
Not sure that I'd like that title. The exitation and decay terms are already explained in the continuing sentence "excitation and decay between upper and lower state sublevels." What you are trying to achieve is to make that phrase understandable to a layperson. That is why I suggested linking to Atomic orbital, and Spinningspark suggested Excited state. The former explains the classical model of electrons in orbits. Depending on what your educational background in Physics is, this may be more accessible to you than the latter link which homes in on the process based on modern concepts. I'm not sure we need a new stub for "decay". On the one hand one could explain the concept both in classical view and modern terms there. On the other hand we might end up with a stub that never goes very far; and particularly in Physics, you'd open a Pandora's box of terms that would have to be explained based on various concepts and at various levels of depth. I would not find either 'Energy Level Decay' nor 'Decay (energy level) a sufficiently clear and understandable definition. Your best bet might be to add a couple of phrases to your Optical pumping page. It is not that extensive yet that it could not accommodate another paragraph. Lisa4edit71.236.23.111 (talk) 03:22, 4 May 2008 (UTC)[reply]
When I read Optical pumping and Trapped ion quantum computer, I was confused. That's why I think if I can do somthing to improve the articles and make it more clear. Unfortunately, I am one of the layperson we are talking about and I'm not a host of Optical pumping which means I may not be qualified to make that phrase understandable. As you said, the created new page could end up with a stub that never goes very far. Therefore, extending Excited state and link to it may be the alternative choice. The first consideration is that the jargon in question also appears in the other articles. For example:
  • Trapped ion quantum computer: "...Hyperfine qubits are extremely long-lived (decay time of the order of thousands to millions of years) and phase/frequency stable (traditionally used for atomic frequency standards). Optical qubits are also relatively long-lived (with a decay time of the order of a second)...a laser couples the ion to some excited states which eventually decay to one state...If the ion decays to one of the other states, the laser will continue to excite the ion until it decays to the state that does not interact with the laser...resulting in a photon being released when the ion decays from the excited state. After decay, the ion is continually excited by the laser and repeatedly emits photons..."
(which would probably give a second jargon 'decay time'. again it's ambiguous to a layperson like me)
Using links may reduce the job to explain it each time it appears in an article. The second consideration is that I think a jargon can not be explained by an article unless it appears in the articles at least once. As we can see, the jargon 'decay' can not be found neither in Excited state nor in Atomic orbital, which meams they are likely not formal articles describing 'decay'. I believe a good article should be "accessible to the lay reader and yet are also useful to the professional working" as stated in Wikipedia:WikiProject Physics. - Justin545 (talk) 06:15, 4 May 2008 (UTC)[reply]

I think the standard dictionary definition is sufficient: "to decline from a sound or prosperous condition". Granted, "sound" and "prosperous" are not the terms a physicist would use, but in the context of the original sentence ("decay between upper and lower state sublevels") one can figure out that the decay is this transition between the upper level and the lower level (whatever they are) without having to be a physicist. --Itub (talk) 17:10, 7 May 2008 (UTC)[reply]

What is meant by "a void has a time dimension"?

I read somewhere that it's impossible to picture "nothing". Most people will simply picture a black void. The book went on to say that it's not really nothing you're picturing because it has dimensions including a time dimension. Dimensions I can understand because your mental image does have a definite measurement, but I dont understand what a time dimension is. How can a void ,mental or otherwise,have a time dimension? How does anything have a time dimension?--Sam Science (talk) 10:36, 3 May 2008 (UTC)[reply]

See Fourth dimension (the first paragraph in particular - ignore that stuff about a spatial 4th dimension) and Spacetime#Concept with dimensions. Zain Ebrahim (talk) 11:13, 3 May 2008 (UTC)[reply]
Okay those articles suck. If you wanted to refer to a point in 3-space you would need 3 co-ordinates (commonly refered to as x, y and z). But that point may exist at various positions in time so if you wanted to refer to just one of those, you'd need to define a 4th dimension to represent time. This may be extended to include constructs of the mind. Zain Ebrahim (talk) 11:23, 3 May 2008 (UTC)[reply]
I don't know if this will help you, but I found it useful. Look at Necker cube. Stare at the cube in the first picture. Note what you see. This is your reality at T1. If you stare for a longer time the surface that you see as "front" will change. Call the moment it changes T2. So what you see as reality depended on the time. Another way to visualize it is the impossible cube at the bottom. If you imagine being in a vehicle that moves around the sculpture at a certain time it will appear to be a cube. --Lisa4edit (talk) 12:46, 3 May 2008 (UTC)[reply]

Circular Motion

A man whirls a stone round his head on the end of a string 4.0 m long.Can the string be in a horizontal plane? If the stone has a mass of 0.4 kg and the string will break if the tension in it exceeds 8n,what is the smallest angle the string can make with the horizontal ? What is the speed of the stone? Take 'g=10m/sec^2 —Preceding unsigned comment added by Priyank Sunil Jain (talkcontribs) 11:09, 3 May 2008 (UTC)[reply]

As it says at the top of the page, we will not do your homework for you. See circular motion and conical pendulum, and come back if you have any more specific questions. Algebraist 11:36, 3 May 2008 (UTC)[reply]
As a corollary, what angle below the horizontal would the string be at if the stone could be swung so as to travel at the speed of light? Is that the same angle as if a ball were rolling around in a hemispherical bowl at the speed of light? (and no quibbles please about how the speed of light is unreachable). Edison (talk) 19:21, 3 May 2008 (UTC)[reply]
If you ignore relativity, then (according to my quick calculations which could be wrong), you're going to get an angle of 2.5x10-14 degrees. If you do take relativity into account, then you can't really ignore those quibbles. You could trying asking it about light, but I can't see how to make light go in a circle without being near a black hole, so I'm not sure the question is even valid. It's possible the angle tends to something meaningful as the speed increases, I'm not quite sure how to calculate it (in my first attempt, all the relativistic factors cancelled and I got the same answer as in the classical case, but I may well have missed something). --Tango (talk) 19:43, 3 May 2008 (UTC)[reply]
Yes, the string can be in a horizontal plane. Given this answer, your new question is: describe the circumstances. ( I have at least three different answere.) -Arch dude (talk) 19:44, 3 May 2008 (UTC)[reply]
Really? I can think of the stone being stuck to the ceiling, but that's it. Is that one of your 3? Give me a clue on the (other) 2/3. --Tango (talk) 19:50, 3 May 2008 (UTC)[reply]
Let's see ... the man could be in a microgravity environment or in free-fall, in which case the string will be in a plane, although the idea of a horizonal plane could be meaningless. Or maybe the stone is sufficiently aerodynamic to generate enough lift to counteract its weight, thus making the string horizontal. Gandalf61 (talk) 23:48, 3 May 2008 (UTC)[reply]
Nah, it's a combination of Longcat and the Noodly Appendages making it happen. That being said, I have this fun image of a tiny helicopter on the stone, keeping it up. Although just attaching a second string to some skyhook would be more sensible. -mattbuck (Talk) 06:47, 4 May 2008 (UTC)[reply]
Microgravity is ruled out by the question giving a value for g. The stone generating lift is a good idea, though. (The skyhook idea sounds equivalent to sticking it to the ceiling). We're up to 2 ways, then... one more to find? --Tango (talk) 13:23, 4 May 2008 (UTC)[reply]
A strong fan might do the trick, but might also create some interesting oscillations. --Prestidigitator (talk) 04:56, 5 May 2008 (UTC)[reply]
Are we (the question as asked, and all answers except for those with zero-G) assuming the string is weightless? Otherwise even if there were enough centripetal/lift/skyhook to keep the stone's circular path in the same plane as the center where the string is held, the string itself would sag (catenary or similar curve?). Thus, the angle of the string at the person's hand is not just a line to the stone. DMacks (talk) 06:51, 5 May 2008 (UTC)[reply]

Finding DNA of Jesus

  • OK you guys, get this :
  • Various relics of Christ have blood and flesh on them. [5]
  • All of the evidence points to the fact that Jesus, AKA Lord incarnate, has AB RH+ blood type. The proteins are said to be extremely rare. [7]
  • Also, the Civitavechia blood samples were analyzed and show that Mary has the same DNA as Christ. (ie Immaculate-Conception + Virgin Birth). [8]
  • To understand what I mean, read this Telegraph article. [9]
  • Some people are now saying that it would be theoretically possible to clone HIM. [10] [11]. However, no one can clone Virgin Mary, who in the past was needed fo birth.
You are assuming A LOT here. Not the least of which is that you assume that this person existed at all.--Shniken1 (talk) 13:26, 3 May 2008 (UTC)[reply]
There's plenty of evidence that Jesus existed - the big assumption is that he was the son of God. See Historical Jesus. --Tango (talk) 13:33, 3 May 2008 (UTC)[reply]
There's actually very scant evidence that there was one historical figure who corresponds with the guy named Jesus in the Bible. There are no contemporary accounts of Jesus from non-Christian sources (Josephus being a potential exception, though one that is almost certainly interpolated at a later date), and most of the Jesus-specific events of the New Testament are not verifiable either. The article in question to read is not "Historical Jesus", which is mostly about Jesus as described in the New Testament, but Historicity of Jesus, which is about whether he existed or not. All that being said, it is very hard to verify the existence of anyone that long ago if they weren't some major figure, and the early Christian church was a pretty small affair. --98.217.8.46 (talk) 13:51, 3 May 2008 (UTC)[reply]
The first couple links are stretches of the imagination. Nevertheless, the DNA contained in a dated blood sample wouldn't be suitable for human cloning most likely. It's a very complex process and has never been successful on a human. Wisdom89 (T / C) 13:29, 3 May 2008 (UTC)[reply]
Assuming the relics are genuine (which is a very big assumption), I very much doubt there is a complete DNA sample there and one would be needed to produce a clone (as would the ability to clone humans, which we don't have yet). --Tango (talk) 13:33, 3 May 2008 (UTC)[reply]
Call me a skeptic, but most of your links seem to be lacking in any scientific merit, let alone peer review. The first link is a very biased site which mentions DNA tests by a Professor Dr. Odorardo who gets precisely zero Google hits except from clone sites or otherwise biased sites using the same source. Was this alleged work actually published? The Telegraph article linked is very skeptical also - not really helping your case. My opinion is that this entire thread should be deleted because it is not asking a genuine question and is contrary to the rules at the top of the page Do not start debates or post diatribes. The reference desk is not a soapbox SpinningSpark 13:46, 3 May 2008 (UTC)[reply]
Have you read those links? You assert that they determined that Mary and Jesus have the same DNA, but the link says instead that the blood found on an allegedly bleeding relic of Mary turned out to be a man's blood. That's not the same thing! If you want to honor your God use the critical faculties he gave you! --98.217.8.46 (talk) 13:59, 3 May 2008 (UTC)[reply]
Those relics have often been passed around from person to person, touched by thousands of pilgrims hoping for a miracle, kept in very unsterile conditions. Contamination with sweat (from touching), saliva (from kissing) and even blood from other people (such as wounded people who touched the relics hoping to be cured) is almost certain. To try to isolate one person's DNA and then identify which person it came from would be nearly impossible. Let's take for example the Holy Sponge (the sponge used to feed Jesus vinegar while on the cross and to mop away his blood and sweat): there have been several pieces of sponge throughout history that people have claimed belonged to Jesus - realistically, at most, only a few of them could have belonged to him. In the unlikely event that you were able to find a genuine piece of the Holy Sponge, you'd then have to remove all the bodily fluids of anyone who touched the sponge, any contaminants introduced by the vinegar and the fluids of other people who fed/cleaned with the same sponge. Since presumably Holy DNA doesn't look any different to normal DNA, you'd have no way of knowing which genes belonged to Jesus. Realistically, the only object you could have any hope of cloning Jesus from is the Holy Prepuce (his foreskin - the only part of Jesus's flesh not to ascend to Heaven), and even this has been faked numerous times throughout history - in fact, the Catholic Church now disputes the idea that the Holy Prepuce even existed in the first place. Laïka 14:31, 3 May 2008 (UTC)[reply]
Minor quibble, I don't think that it was actually His sponge in the sense of Him owning it. Matthew:27:48. SpinningSpark 16:21, 3 May 2008 (UTC)[reply]
We may define "faith" as the firm belief in something for which there is no evidence. Where there is evidence, no one speaks of "faith." We do not speak of faith that two and two are four or that the earth is round. We only speak of faith when we wish to substitute emotion for evidence. - Bertrand Russell Imagine Reason (talk) 15:03, 3 May 2008 (UTC)[reply]
I would rather say faith is belief in something for which there is not enough physical evidence and verification to constitute proof. Not necessarily none... Wrad (talk) 15:21, 3 May 2008 (UTC)[reply]
I would also say don't believe everything you read on the internet, especially about religion and Jesus :) . Wrad (talk) 16:23, 3 May 2008 (UTC)[reply]
Well, I guess this is one of the minor problems with science. There's too much scepticism. Sure, scepticism can be a good thing, similar to deletionism on Wikipedia, but with things that conventional science does not accept, you'd need absolute proof. For example, even the most authentic UFO cases are dismissed as "unexplained". Why? They argue that when the only possible explaination would be that UFOs/ailens exist, they dismiss it because they "cannot" exist. Why? Is there any proof that they can not exist? No. Is it actually true that if there is no absolute proof that they exist, only evidence, that they cannot exist? Absolutely not. Modern science also seems to be critical of religion, as in this example. Why? Well, because there supposedly is "insubstantial evidence". "Conventional" science accepts such theories as parallel universes, but cannot accept even remotely paranormal things such as cryptozoological creatures? There is evidence that they might exist, and none that they can't exist. This scepticism prevents science from turning into a junkyard, but surely a few of the things that conventional science does not accept might be real? Why do you need, say, 10x more evidence for things not accepted by conventional science, compared to say, things that conventional science is open to? Besides, we'll likely not get proof that ailens exist, at least in the US, because US Federal law prohibits contact with non-Terrestrial beings! As for the links, it would probably not make it past consensus and into an article, at least the whole thing anyway. Why? It's not a frindge theory because it's not a purely scientific article. Anyway, are there any articles that this information might be helpful to? Thanks. ~AH1(TCU) 18:10, 3 May 2008 (UTC)[reply]
The problem isn't so much the lack of evidence as it is the lack of more evidence for one explanation over another. Sure, you could explain UFOs are alien spacecraft, but you could also explain them as some unknown meteorological phenomenon, or a test of classified military technology, or any number of other things. There's no more evidence for it being aliens than for any of the other wild guesses. --Tango (talk) 20:25, 3 May 2008 (UTC)[reply]
I find Bertrand Russell's Last Thursday theory of creation to be the most intellectually satisfying of the lot. It explains everything except the moment of creation, and does not contradict with anything. Even science cannot do that. Imagine Reason (talk) 16:59, 4 May 2008 (UTC)[reply]
I can see what you mean. Perhaps the last sentence there is more correct than the first. Imagine Reason (talk) 18:12, 3 May 2008 (UTC)[reply]
Scientists have a hard time accepting the 'evidence' for UFOs (and religion) because you have to assume the existence of extraterrestrial aliens (and god/gods) for the case for UFO (and religion) to make the argument. Scientists are willing to accept the hypothesis, but not make unwarranted assumptions. -- JSBillings 18:17, 3 May 2008 (UTC)[reply]
It is hard to accept UFO's as being alien spacecraft not because it requires aliens to exist, but it requires aliens to be capable of interstellar travel. Most scientists will probably believe, based purely on statistics, that lifeforms (however intelligent) exist somewhere else in the universe.--Shniken1 (talk) 13:28, 4 May 2008 (UTC)[reply]
The sauce on the Flying Spagetti Monster had the same DNA as Invisible Pink Unicorn blood,may her hooves never be shod.hotclaws 23:29, 8 May 2008 (UTC)[reply]

mechanical nasal congestion relief

This question has been removed. Per the reference desk guidelines, the reference desk is not an appropriate place to request medical, legal or other professional advice, including any kind of medical diagnosis, prognosis, or treatment recommendations. For such advice, please see a qualified professional. If you don't believe this is such a request, please explain what you meant to ask, either here or on the Reference Desk's talk page.
This question has been removed. Per the reference desk guidelines, the reference desk is not an appropriate place to request medical, legal or other professional advice, including any kind of medical diagnosis or prognosis, or treatment recommendations. For such advice, please see a qualified professional. If you don't believe this is such a request, please explain what you meant to ask, either here or on the Reference Desk's talk page. --~~~~
We really can't advise you on how best to perform medical procedures on yourself, or how to acquire equipment for same. If you want to perform procedures on yourself, you should speak to an appropriate specialist first—you need to be trained in proper methods, risks, choice of equipment, and so forth. It may not be appropriate for you to attempt the procedure without supervision, period. TenOfAllTrades(talk) 15:03, 3 May 2008 (UTC)[reply]

Self operation

Talking of carrying out procedures on yourself, I recall a report of a surgeon who tried to remove his own appendix. Failed in the attempt when he passed into unconciousness. Anyone got a link to this event? It was maybe 25 years ago, so I could be going senile and imagining it. SpinningSpark 16:27, 3 May 2008 (UTC)[reply]

The surgeon you are referring to is Dr. Kane, and i've found one link that documents the event: http://query.nytimes.com/gst/abstract.html?res=9F03E0D9123CE533A25754C1A9649C946095D6CF Regards, CycloneNimrodTalk? 18:10, 3 May 2008 (UTC)[reply]
Wikipedia has an article on almost everything, but not on Dr. Evan O'Neill Kane, though there is an article on his father, and Dr. Evan's remarkable feat is mentioned in the article on Kane, Pennsylvania. ---Sluzzelin talk 18:57, 3 May 2008 (UTC)[reply]
That was in 1921? I must be older than I thought! SpinningSpark 22:49, 3 May 2008 (UTC)[reply]
We do, however, have an article about Dr. Jerri Nielsen. 146.115.120.4 (talk) 20:52, 3 May 2008 (UTC)[reply]
...which references the own-appendix-removal feat of Dr. Leonid Rogozov, which we don't have any article on either. We seem to have discovered a rare void in Wikipedia coverage. --169.230.94.28 (talk) 18:59, 8 May 2008 (UTC)[reply]

gravity within a heavy body (Earth)

Approximately what happens to measurements of gravity going deeper toward the center of the earth (ie: increase to 2Gs at 1000 miles deep, then progressively decrease to 1G at 2000 miles and to zero Gs at the center)? —Preceding unsigned comment added by 70.157.224.152 (talk) 19:06, 3 May 2008 (UTC)[reply]

For a body of uniform mass distribution (ie. the same density all the way through), it decreases linearly to 0 at the centre (so 1g on the surface, 0.9g 10% of the way down, 0.5g half way to the centre, etc.). The Earth isn't actually uniform, so that won't quite work, but it's still going to reduce all the way down, just not at a constant rate. It will never increase when you go deeper. --Tango (talk) 19:25, 3 May 2008 (UTC)[reply]
Actually I think it does increase as you go deeper until you hit the outer core. The core is so much denser than the mantle that increasing proximity to the core outweighs (literally) the reduced influence from the mantle. -- BenRG (talk) 21:13, 3 May 2008 (UTC)[reply]
I hadn't thought of that - you're right. Based on some very rough calculations, gravity at the edge of the outer core could be around 25% higher than on the surface. You learn something every day. --Tango (talk) 21:41, 3 May 2008 (UTC)[reply]
My rough calculations disagree. Care to share yours? Dragons flight (talk) 03:30, 4 May 2008 (UTC)[reply]
It's probably the numbers rather than the calculations that differ. I was using an core radius of 3400km and a density of 13000kg/m3 (that's an very rough upper limit given the data I could find) and the whole Earth with a radius of 6371km and average density of 5515kg/m3. I then used that acceleration due to gravity is proportional to density times radius (I ignored the constants, since I was just interested in which was bigger). That gave gravity at the core being 4.42x1010 and gravity at the surface being 3.51x1010 in whatever strange units that comes out to. --Tango (talk) 12:58, 4 May 2008 (UTC)[reply]
I'm sure there is some way of finding this answer by searching the archives. The idea is encapsulated in the Divergence theorem, otherwise known (especially in physics), as Gauss's Law. Note that I've linked to the form you should find especially useful. (The most common meaning when people refer to Gauss's law is to the electric field. Since both electricity and gravity are conservative forces following an inverse square law, they obey much of the same mathematics.)
The idea behind applying the mathematics is that at any point inside the surface of the earth, there is a big spherical chunck of earth "below" you and a big spherical shell of earth "above" you. The math proves that the shell actually doesn't have any effect on your measurements of gravity. Once you know that, you can reason that as you move away from the center of the earth, the mass "below" you grows cubicly; however, your distance from the center falls linearly, so the gravitational strength falls quadradically. Therefore, the total measurment of gravitational force increases only linearly.
To really understand it, you should work out the mathematics for yourself. For a more detailed explanaiton, you should indicate what level of mathematics you can (or want to) deal with. — gogobera (talk) 19:40, 3 May 2008 (UTC)[reply]
Just to say on Gauss's law, an historical note, I believe that it was Newton who first showed that there is no field inside a hollow sphere. SpinningSpark 23:52, 3 May 2008 (UTC)[reply]
Yes, he did it with the shell theorem, a far more complicated way of doing it than with Gauss's law of gravity, which renders the problem trivial. Someguy1221 (talk) 03:32, 4 May 2008 (UTC)[reply]

Science in crisis ?

Recently, I have been reading statements from leaders in the scientific community, philosophers of science, university professors, sociologists of science, corporations who hire scientists and scientists themselves, and many claim that there are wide problems within science that go from methododogy, to financing, to training sufficient specialists and to the very goals of science itself.
  • Author Paul Feyerabend embraced relativism, and even "epistemological anarchy".
  • Thomas Kuhn's philosophy of science, as expressed in The Structure of Scientific claims that as well as progressing steadily and incrementally ("normal science"), science undergoes periodic revolutions or "paradigm shifts", leaving scientists working in different paradigms with difficulty in even communicating.
  • According to Daniel Boy, researcher at CNRS, there is a major european shortage of qualified scientists and mathematicians. `[12]
  • The blog Science Crisis gives many details on the so-called scientific crisis. [13]

What are the social, political and technical consequences of such a phenomenon ? 69.157.243.164 (talk) 20:00, 3 May 2008 (UTC)[reply]

I'm not sure this reference desk is really suited to such a philosophical question. The best we can do is offer our personal opinions, and what good is that, really? We're just random people on the internet. --Tango (talk) 20:21, 3 May 2008 (UTC)[reply]
Things like statistics on students graduating in certain fields, science spending etc. can be found from sources like the EU, UNESCO and the like, or by googling. Opinions on the effects on certain fields can be found by searching journal sites, such as "Physics Today" for relevant key words. (They have e.g. some articles describing the effects of not building the Superconducting Supercollider.) A search through history pages for similar situations (or asking at th Humanities desk) should give some basis for evaluating the sources you cite. Apart from that Tango's right all we could offer would be anecdotes and opinions. --71.236.23.111 (talk) 22:23, 3 May 2008 (UTC)[reply]
Today's society generally undervalues scientific endevour. I blame society's current obsession with vacuous celebrities. When a large proportion of school kids are limiting their aspirations to "I want to be a celebrity like ...." why should they think science is a worthwhile career? Just look at the treatment of scientists in film - they're usually depicted as boring nerds or evil madmen. The kids of today don't want to be scientists when their friends will laugh at them and they won't get any hot dates. So what happens when the current generation of scientists retire? No wonder science is in crisis. Astronaut (talk) 16:10, 4 May 2008 (UTC)[reply]
I have to say, though, as a scientist who is only slightly mad and boring, I have had no trouble getting hot dates[citation needed]. This does not stop my friends from laughing at me. Nimur (talk) 16:29, 4 May 2008 (UTC)[reply]
Kevin Eggan made it into People Magazine's 100 sexiest men issue a few years back, hot dates has not been a problem for him. Rockpocket 19:03, 4 May 2008 (UTC)[reply]
I find that stereotype a strictly American phenomenon, one that has existed for a long, long time. Anti-intellectualism is embraced for many reasons here. Imagine Reason (talk) 02:49, 5 May 2008 (UTC)[reply]

Visual question

Why does the human eye do a far better job at handling the lighting contrasts in what it sees than cameras do? I guess this is because as humans evolved, the brain became increasingly more complex, and is in fact post-processing what the eye sees before, sort of, letting itself see it. But when we look at a photograph, the brain does not recognise it as a real live view, and does not know to post-process it. If this is right, then it feels very weird not seeing things as they really are, and having no way to change that. JIP | Talk 20:30, 3 May 2008 (UTC)[reply]

If everyone answered their own questions as well as you just did, we would have nothing to do here!
You might like to read the visual perception and optical illusions articles. --Anonymous, 22:13 UTC, May 3, 2008.
I was just proposing a theory and asking if it was right or wrong. It feels really weird that the brain is doing something which it, in effect, keeps secret from itself. But based on the theories presented in the article about visual perception, it is for the better, because otherwise we would not perhaps be even able to survive, not to mention enjoy the free time in our life. JIP | Talk 21:23, 3 May 2008 (UTC)[reply]
Could you give an example of what you are talking about?Em3ryguy (talk) 22:59, 3 May 2008 (UTC)[reply]
There have been a couple of research studies and experiments that tell us that, "the brain is doing something which it, in effect, keeps secret from itself," is much more the usual state of affairs than something unusual. Scientific American has split off an entire magazine because such questions have garnered so much interest. To mention just a few: pheromone and chemical cues from the olfactory system influence behavior[14], without entering conscious recognition. You can fool your body to mix sensations that would be generated by what it sees with those actually reported. [15] [16] [17]You might also enjoy http://www.rburton.com/work1.htm . Our Choice-supportive bias only touches on a huge area currently under study, there are probably a host of relevant pages I haven't seen, though. Lisa4edit (talk) 23:22, 3 May 2008 (UTC)[reply]
Yeah, I think the post-processing idea is right but there is also a mechanical reason. When a camera takes a picture, it must choose one iris setting and the picture is printed with one dynamic range of contrast (left as exercise for student to convert that sentence to modern electronic-speak). A human eye is not taking a single picture, but is constantly viewing a scene. Details in, say, a dark corner can be observed by opening up the iris and ramping up the gain - those details can then be added to the image already formed in the brain. In a bright corner, do the opposite, close down the iris and the gain. A camera cannot do that with a single snap resulting in "clipping" of either the dark or the bright detail. SpinningSpark 23:45, 3 May 2008 (UTC)[reply]
If we are talking picture there is also the color representation issue. Particularly green shades are devilishly difficult to generate. That's why some pictures have an "unreal" tint to them. Lisa4edit (talk) 00:12, 4 May 2008 (UTC)[reply]
Some other reasons:
  • Point-and-shoot digital cameras have a lot of CCD noise because the CCD is small.
  • JPEG only has 8 bits of luminance depth, and it's mapped roughly linearly to perceived brightness, so there's not much resolution at the dark end.
  • Regardless of the camera's abilities, photographic prints and computer monitors can't reproduce anything close to the dynamic range of the real world. -- BenRG (talk) 13:00, 4 May 2008 (UTC)[reply]

When you look at a poor-contrast picture, one reason your eye can't "fix" the contrast the way it would have if it had been looking at the same low-contrast scene in real life is really: your eye is doing a good job. You did not show it that low-contrast scene and ask it what it looked like. Rather, you showed it that low-contrast picture and asked it what it looked like. And your eye is correctly telling you: "it's a low-contrast picture." If there are any details to be made out beneath the poor contrast in the picture, your eye will probably see them. But it won't stop telling you "But, by the way, the contrast is really bad." —Steve Summit (talk) 16:13, 4 May 2008 (UTC)[reply]

I assumed as much. When I look at a photograph, what my eyes see is not the original light, but the camera's captured image of it. Thus, the eye's constant scanning (as mentioned above) can not react to the difference in actual light in parts of the picture, because there is none. JIP | Talk 19:31, 5 May 2008 (UTC)[reply]

Although it is true that we recognize that photographs are only representations, it is absolutely untrue that no "post-processing" occurs in the brain. The post-processing you speak of is not (in general) the kind of post processing that a photographer accomplishes in photoshop. I can't articulate this very well, but although it seems as though your experience of sight is as though you were a homunculus observing a stereoptic film, the fact is that a significant portion of our experience of sight relies on interpretation of what we see. When you look at a photo, you don't see it as a live image, but you also don't see it as blobs of color: you see it as a photograph, with a background, foreground, objects, people, faces, etc. you are capable of interpreting many out-of-focus features, although your perception of these features does not bring them into focus as computer post-processing would. Your implied question of "seeing things the way they really are" is a serious topic in metaphysics and philosophy of mind. Whether there is an objective "how things really are" is even relatively contentious.--Shaggorama (talk) 09:13, 8 May 2008 (UTC)[reply]

In the equatorial regions, there are only dry or wet seasons. But what about just around the equator? Would there still be distinct different seasons on both sides? Or would it be one season all the way through? Or perhaps both sides of the equator follows the same season? 99.226.39.245 (talk) 23:24, 3 May 2008 (UTC)[reply]

Unless I misread your question, this phrase from the article would answer that: On the equator, there are two wet and two dry seasons as the rain belt passes over twice a year, once moving north and once moving south. Lisa4edit (talk) 00:17, 4 May 2008 (UTC)[reply]

Well, I was refering to literally on the equator. 99.226.26.154 (talk) 04:40, 4 May 2008 (UTC)[reply]

That answer refers to weather literally on the equator. What's the problem? Algebraist 08:04, 4 May 2008 (UTC)[reply]
You may be interested in reading [18] (note you will have to click on the links at the top of this page to navigate as a next link seems to be missing) and [19] (yes it's about a specific period quite old but isn't far from the norm from a quick read through) & [20] which describe the climate in Malaysia. While Malaysia is north of the equator the weather isn't as simple as one wet and one dry season it varies depending on the geographical location (and therefore the surrounding terrain), and the wet season particularly on the east of Peninsular Malaysia is usually in October to March (see also Floods in Malaysia and Monsoon Cup) while your likely to think the opposite from the wikipedia articles Dry season and Wet season. Tropical weather can be I think a lot more difficult to understand then the 4 season temperate model (although even that is far from simple). You may also want to read Monsoon if you haven't already. Nil Einne (talk) 13:12, 4 May 2008 (UTC)[reply]


May 4

Blue and green eye color in Africans

Does anyone have information on a Klein Wollendorf Syndrome" When I attended the Univ. of Pittsburgh back in the 70's I was doing some research in color preference and I happened on this book Titled the Klein Wollendorf Syndrome which documented the occurences if bright blue eye color in sub saharan Africans. They also documented the occurences of people who were born with emerald green eye color and the white of the eye was yellow instead of white. The book included many pictures of the people with the eye colors. The authors said widespread documentation was difficult because in previous generations many of the children born with the eye colors were killed at birth because they were considered cursed. I have lost my bibliography so I don't have a publisher or dates. If any one has any information, please respond. —Preceding unsigned comment added by Chaniatreides (talkcontribs) 06:06, 4 May 2008 (UTC)[reply]

I expect you are referring to Klein-Waardenburg syndrome (OMIM: 148820), also known as Type III Waardenburg syndrome. It results from a mutation in PAX3 and, among other things, manifests with melanocyte deficiencies (hence the eye colours). A PubMed search with the term should provide all the info you need. By the way, the book you refer to is probably: Jenni Soussi Tsafrir, Light-eyed Negroes and the Klein-Waardenburg Syndrome, London, Macmillan, 1974, ISBN 0333140729. Rockpocket 06:49, 4 May 2008 (UTC)[reply]
We really do have a page on everything! ;-) Waardenburg syndrome--Lisa4edit (talk) 09:46, 4 May 2008 (UTC)[reply]

A four-hour day

I'm wondering just how life might be different if the days were 4 hours long (defined as 3600 seconds, where a second is defined in relation to the speed of light as is current). Would we still have the same sort of circadian rhythms? -mattbuck (Talk) 06:41, 4 May 2008 (UTC)[reply]

Hold on, are you talking about an actual day (i.e. the earth's rotation around the sun) being 4 hours or some random definition of a day as being 4 hours?. If it's the former then our circadian rhythms would presuambly have evolved in such an environment and would therefore be best adapted to a 4 hour day. If the later, well it would be irrelevant out circadian rhythm would still be best adapted to a 24 hour day Nil Einne (talk) 07:44, 4 May 2008 (UTC)[reply]
There was a study a few decades ago (before the Internet). They put some subjects in a bunker, cut off from any outside light source. They were not allowed any clocks and could choose when to switch on their electric lights and declare it "daytime" and when to sleep equating "nighttime". Most of the subjects adjusted to a "day" longer than 24 hrs. (I'll see if I can find that mentioned somewhere.)--Lisa4edit (talk) 09:36, 4 May 2008 (UTC)[reply]
Seems to be mentioned on Circadian rhythm Nil Einne (talk) 12:43, 4 May 2008 (UTC)[reply]
That was it, the results were refuted I see. Thanks for pointing that out. I had apparently never read that article far enough. --Lisa4edit (talk) 16:11, 4 May 2008 (UTC)[reply]
An experiment with an 8-hour "day" has been ongoing for several hundred years. Large numbers of humans have been placed in artificial environments and required to be active for four hours and then be (more or less) at leisure or sleeping or four hours, sometimes for years at a time. Some humans adhered to this schedule from childhood until (relatively) old age. I refer of course to the watch system of the (british) navy and other navies. -Arch dude (talk) 23:33, 4 May 2008 (UTC)[reply]

What would a YY chromosome person look like?

Is it possible that a person has YY chromosomes? Is it possible to create a person from two Y chromosomes from a male? If not, just for fun what do you think a person with such chromosomes would act like or look like? —Preceding unsigned comment added by 70.219.182.238 (talk) 07:46, 4 May 2008 (UTC)[reply]

It is possible for humans to have YY, but only in combination with a X, giving them XYY syndrome. XYY people tend to have slightly lower IQs an increased risk of learning difficulties. You can even get people with XYYY and XXYY syndrome, though they tend to have more severe difficulties.
The human Y chromosome is tiny and contains very few genes. It largely contains those genes necessary for becoming male, SRY for example. In humans the X contains lots of genes many of which have nothing to do with sex determination and are essential for life. Examples include a gene to make blood clotting proteins. So, at least in humans, everyone needs at least one X or else they do not develop. Therefore a person with YY sex chromosomes would probably look like a jelly bean, before aborting in utero. Rockpocket 08:26, 4 May 2008 (UTC)[reply]
Is it possible for the Y chromosome to be genetically engineered to be able to hold the necessary genes, making it possible for just YY human beings? —Preceding unsigned comment added by Jue2 (talkcontribs) 08:34, 4 May 2008 (UTC)[reply]
Well, currently the technology is not available for humans, but it would probably be feasible to do that in experimental animals. That would be the functional equivalent of putting the SRY region (the gene that makes you a "male") into an X chromosome. This occasionally happens in humans due to chromosomal translocation. The result is individuals that have female haplotypes but male phenotypes. The only major problem is that those males are infertile. Speculating now: If you could pick all the essential genes on X on put them on a Y (lets call that Y*) you would have all the genes required for life. But YY* people would still have problems, because they would have two doses of the genes on the Y chromosome, including SRY. That would result in the same difficulties seen in XYY males. Females get around the dosage problem by lyonisation, but males could not do that. They way around this would be to engineer a Y*O male (with just one modified Y chromosome). Would that male survive and be healthy. Who knows?
By the way, it turns out that YY embryos abort before implantation, thus in humans a YY would only last a week or two so after conception before aborting. Rockpocket 09:03, 4 May 2008 (UTC)[reply]
So it would be easier for women to exist without men, than men to exist without women. That is a very scary thought. What gene in the Y chromosome decides gender? The Y chromosome has few few genes compared to the X so, why couldn't engineers move that gene in the Y to the X chromosome? Jue2 (talk) 09:22, 4 May 2008 (UTC)Jue[reply]
Yes. Womankind could theoretically exist without men when human cloning becomes a reality (because all you need then is a womb in which to carry the XX clone). Why else do you think all those male dominated societies have outlawed human cloning ;)? As I said above, the principle male sex determining gene in humans is SRY and nature occasionally moves SRY into the X chromosome by unequal chromosomal crossover, resulting in XX male syndrome. Rockpocket 10:01, 4 May 2008 (UTC)[reply]
I find the thought amusing. A species without females--now that would be scary. Imagine Reason (talk) 02:44, 5 May 2008 (UTC)[reply]
So check out New Mexico whiptail and be afraid, be very afraid :-). 87.81.230.195 (talk) 08:08, 30 March 2009 (UTC)[reply]

Time Travel is Impossible

Time travel just can't happen, otherwise we already would know about it. Not to mention the paradoxes it would cause. Travel back in time and kill my mother before I was born. I guess my question is, "does anyone believe time travel is possible? If so please explain it to me." —Preceding unsigned comment added by Jue2 (talkcontribs) 09:30, 4 May 2008 (UTC)[reply]

Our Time travel article may be of interest to you. I suppose that there is at least someone who believes that it's possible. --hydnjo talk 10:47, 4 May 2008 (UTC)[reply]
There are consistent theories that allow for time travel, but they're quiet restrictive. For example, you can't generally travel back to before the time machine was created. Exactly how it would work with paradoxes, I don't know - I expect you would simply find that you can't change anything because you were there doing exactly the same thing "the first time round", and just didn't see yourself. (The main alternative theory is that you would travel back to an alternate reality and change that one, rather than your own.) --Tango (talk) 13:09, 4 May 2008 (UTC)[reply]
The expression Time travel usually is understood to only move the traveler in time. If you'd allow travel to another destination (or another universe) you remove quite a few roadblocks. Moving a person also makes things a whole lot less likely. If you could figure out how to send the information to assemble a person (as proposed for SciFi "transporters") things would also become easier. Traveling to the past I'd still think highly unlikely. But I keep encountering things that everyone used to think were impossible. ( see "Humpty Dumpty" for a recent event.) Lisa4edit (talk) 15:13, 4 May 2008 (UTC)[reply]
Time travel is not imposssible, I have already told you that tomorrow - don't you remember? SpinningSpark 15:40, 4 May 2008 (UTC)[reply]
The real question is, "Can an all-powerful god alter the past?" Imagine Reason (talk) 22:37, 4 May 2008 (UTC)[reply]
Why would an all-powerful god need to alter the past if he was all powerful and all knowing? I've managed to go back in time before. It was easy, but really noisy as well. bibliomaniac15 Do I have your trust? 03:58, 5 May 2008 (UTC)[reply]
I was only concerned with the ability to change the past. An all-powerful god may not be all-benevolent. He may well decide to play dice with our lives and change our past on a whim. Imagine Reason (talk) 15:56, 5 May 2008 (UTC)[reply]

Time travel paradoxes vanish when parallel universes are considered. Amelia Earhardt had no role as an aviatrix in World War 2 in the parallel universe where she vanished in the 1930's. Judge Crater never got appointed to the U,.S. Supreme Court in the parallel universe where he vanished mysteriously. Charles Lindbergh Jr. never became President of the United States in the parallel universe where he was kidnapped and killed as a toddler. In the parallel universe where a time traveller altered history the alteration would be accepted as part of that time stream's history. Edison (talk) 03:44, 5 May 2008 (UTC)[reply]

Another argument for the impossibility of time travel is that if there were ever in the future a year when time travel became possible, then a traveller would have announced his presence in our time or in our past, and we would all know that time travel was possible. In the parallel universe model of reality, there could be many time streams where time travel has been shown to be possible, just not in this one (yet). Consider then whether it is ever the case than some supposedly deranged individual is picked up and tells the police/psychiatric personnel that he is from the future. Would we even hear about it? If the time traveller were not incautious enough to announce his presence that way, but were only allowed to arrive with no equipment, clothing, or other physical evidence inconsistent with this time, and screened to act as a scientific observer, he might pass unnoticed (but certainly not without having caused this time stream to be altered in some way). Even such a prudent observer would inevitable get the last place on the elevator, causing someone to miss his medical appointment, causing their cancer not to be detected in time. Or he might take a seat in a restaurant which otherwise would have been occupied by one half of a couple who therefore did not meet, marry, and give birth to a world leader, as in the Butterfly effect. Edison (talk) 12:40, 5 May 2008 (UTC)[reply]
That whole issue with nobody coming back and telling us they're from the future disappears if you impose the "you can't travel back to before the time machine was created" restriction, which I believe is imposed by most (if not all) ideas for time machines based on General Relativity. --Tango (talk) 12:59, 5 May 2008 (UTC)[reply]
It is easire to envision time travel where the "booth" is the arrival point, because the traveller thereby does not arrive inside another person or a tree, but there are probably ways around the claimed general relativity prohibition. Edison (talk) 18:54, 5 May 2008 (UTC)[reply]
the question of time paradox is largely contingent on your theory of what time is to begin with. The logical answer to the most basic time paradoxes is "you couldn't because you didn't" or "it's possible because you did." Basically, the argument is that "changing the past" is a meaningless phrase; because the past already occured, it can't be changed. Think Twelve Monkeys or the first Terminator movie. --Shaggorama (talk) 09:24, 8 May 2008 (UTC)[reply]

Does any one know what this is

Hi I would like to know what article this could go in. I found this in the Blue Mountains New South Wales. I may even have better images of this
. --User:Adam.J.W.C. (talk) (talk) 10:38, 4 May 2008 (UTC)[reply]
I guess you can't wait till it's matured into a moth. :-) [21] narrows it down to LYMANTRIIDAE or "Tussock Moth" "are so-named because the Caterpillars of many members have four long dense dorsal tufts of hair. Many also have other hair pencils, and also two coloured dorsal glands on abdominal segments six and seven." --Lisa4edit (talk) 12:03, 4 May 2008 (UTC)[reply]
If you have lots of time you might find it here [22]Lisa4edit (talk) 12:41, 4 May 2008 (UTC)[reply]
I forwarded your identification request to Wikipedia:Wikiproject Lepidoptera. If they don't contact you soon, try going over to the project page and picking out a few contributors to contact directly in case the project page doesn't get regular visitors. --Shaggorama (talk) 09:38, 8 May 2008 (UTC)[reply]

Constructing a lunar orbital station by firing materials from the moon

My dad is writing a science fiction novel and has a Clarkesque desire for realism. He's described a situation to me but the maths is pretty complicated and well beyond my knowledge. The figures used below are our guesses at plausible numbers, if these guesses are unreasonably high or low for the practicalities of the situation, please say so and we can revise. The situation is as follows:

Suppose we are constructing a space station in an equatorial orbit around the moon at a height around the order of 300km. It is being constructed from materials mined on the moon, and they are being launched up to meet the station by a linear accelerator on the moon's surface. The materials are launched in ~200 tonne payloads and the accelerator can impart acceleration no greater than, say, 4 G. The accelerator can be built on an incline but if it can be laying flat to the ground this would be preferable. Suppose the construction platform begins with a mass of 1000 tonne, this will of course increase with each payload and the added mass and momentum will obviously alter the orbit of the platform itself. The payloads are capable of minor course alterations but aren't equipped with large rockets or anything. The payload "catching" mechanism on the construction platform requires payloads travelling no faster than 0.1 m/s (or something else plausible). Is this situation workable, and what kind of length / launch speed / angle / timing are required for the launcher?

It seems to me like we'd need the payload to reach the platform at the payload's apoapsis (highest point) and the platform's periapsis (lowest point). However, the fact that the platform will be changing orbit with each payload makes the whole thing too difficult for me to see how to approach since I don't know enough about orbital mechanics to know how the orbit will be changing. Any ideas? Thanks, Maelin (Talk | Contribs) 10:43, 4 May 2008 (UTC)[reply]

The orbit of the platform won't change - the mass of a satellite doesn't feature in the equations for it's motion (it cancels out). As long as it stays significantly less than the mass of the moon, you don't have to worry. There will be a transfer of momentum when you catch each payload, but for the catching to work you'll need them going quite slowly relative to each other anyway, so I think you can ignore that. Have you read out article on mass drivers? I think that's what you're describing. It doesn't have many technical details, but it might help a little. What you probably want is the platform to be in a circular orbit, and then the payloads launched at a pretty shallow angle (horizontal would be fine). I think you would then need the payloads to fire small rockets at the appropriate point to circularise their orbit slightly lower than the platform, then when they catch the platform up they fire again to lift them up to it and reduce their relative velocity so the catching can work. I'm not entirely sure about how launching things into orbit with mass drivers works - most applications I've seen seem to involve giving them escape velocity, but putting things into orbit should be possible, I'm just not sure how big a rocket you would need for the circularisation (certainly smaller than you would need for an actual launch). --Tango (talk) 13:21, 4 May 2008 (UTC)[reply]
If the space station is in any particular orbit, and the momentum of a 200 tonne addition is added to it, without firing a rocket to match orbits, the orbit of the combination will inevitably be different from that of the space station initially. I see two non-rocket solutions. The ugly one is that the space station starts in its desired orbit. Each time a payload docks with it, the orbit would change somewhat. To avoid this, the payload could fire off a small part of its mass at a high velocity (spring? cannon? electromagnetic thruster? compressed gas?) This could have the same effect as firing a thruster. The downside would be that these correction ejections would then be flying around the moon in orbit, unless they were fired off with escape velocity, ideally to ultimately impact the Sun or perhaps some planet the Moon dwellers didn't like (Heinlein had a story of throwing rocks at the Earth from the Moon, as I recall). The elegant solution is to have the initial part of the space station in an orbit which differs from the final desired orbit in a carefully calculated way. Then each time it catches a payload, the orbit of the combination moves toward the desired orbit. When all the payloads have been caught, the orbit has shifted exactly to the desired orbit. (Difficult to calculate? Aw gee, orbital mechanics is HARD!) Edison (talk) 13:45, 4 May 2008 (UTC)[reply]
Requiring a relative velocity of less than 0.1 m/s sounds like matching orbits to me. I expect small station keeping thrusters on the platform would be enough to correct for any relative momentum not shed before catching. --Tango (talk) 13:55, 4 May 2008 (UTC)[reply]
There is a cumulative effect here. One payload you might get away with if it is small and slow, but if you are trying to significantly increase the size of the station, eventually you are going to have to correct the orbit in some way. Fuel for thrusters can only arrive through the payload which I think is what the original concept is trying to avoid. Don't forget that any payload launched ballistically from the surface is going to end up in an orbit which will intersect its starting point (ie it is going to try and fly through the mass of the moon). Therefore there MUST be a correction to the payload by some means to insert it into a working orbit. If this is done by the station catching it, it will inevitably impart a momentum to the station in an undesirable direction. You might be able to do something clever with tethers though. SpinningSpark 14:17, 4 May 2008 (UTC)[reply]
As long as the exhaust velocity is greater than the relative velocities of the payloads when caught, bringing fuel up shouldn't be a problem. The OP said the catching mechanism required the relative velocity to be less than 0.1m/s, so station keeping should be easy. It's slowing the payloads down (or speeding them up, depending on perspective) that's hard - tethers could work, and are certainly more interesting to read about than rockets. --Tango (talk) 14:27, 4 May 2008 (UTC)[reply]
You could employ a tether or [23] might work. You could at least use it to get rid of some need for corrections as far as I understand it. This is way out of my league but maybe s.o. else can develop the idea, or say why it won't fly. Lisa4edit (talk) 15:57, 4 May 2008 (UTC)[reply]
It might be possible with 2 accelerators 90 degrees apart. They would cancel each other out. Catching the payloads wouldn't be a problem either. Each accelerator sends one payload into orbit. the 2 payloads collide and the resulting single payload would have a very low velocity relative to the station. I doubt that its really feasible but it should good enough for a science fiction story.Em3ryguy (talk) 19:13, 4 May 2008 (UTC)[reply]
Why would they collide? I can't see a way to get them to be in the same place at the same time. --Tango (talk) 19:41, 4 May 2008 (UTC)[reply]
"The payloads are capable of minor course alterations". If their orbits are in the same plane then they must cross someplace. Also the collision doest have to be anywhere near the station. After the collision the resulting single payload can then drift slowly to the orbiting station on the other side of the moon. Half of the payloads will do so in a slightly lower orbit and half in a slightly higher orbit. As a result, the orbiting stations orbit would not be changed.Em3ryguy (talk) 20:38, 4 May 2008 (UTC)[reply]
Their orbits will cross, sure, but they won't both reach the crossing point at the same time. Also, the term "orbit" is a little misleading - the payloads will crash back to Earth before completing one complete orbit if they don't do something to circularise it. --Tango (talk) 20:42, 4 May 2008 (UTC)[reply]
They dont have to actually collide. They can use many mile long tethers to simply orbit one another. We are talking about fiction here, yes?Em3ryguy (talk) 21:42, 4 May 2008 (UTC)[reply]
We're talking about hard science fiction - the idea is that it is actually physically plausible (given sufficiently advanced engineering). There are all kinds of things you can do with tethers, but I'm not sure what you're suggesting... --Tango (talk) 22:19, 4 May 2008 (UTC)[reply]
The collision of the 2 payloads circularizes the orbit. Thats the whole point.Em3ryguy (talk) 02:33, 5 May 2008 (UTC)[reply]
The idea is ridiculous, even if the launch was synchronised perfectly so that the two payloads did meet at aposelene the collision would not do anything to help match orbits with the station. It will just result in crushed payload canisters. A collision does not add energy to the system so it cannot possible achieve the orbit desired. As for the tether, if you launch at 90 degrees, the two payloads are going to end up tens of thousands of miles apart before they meet again - that's a lot of string to reel back in!SpinningSpark 02:44, 5 May 2008 (UTC)[reply]
90 degrees apart in longitude. they are 1000km apart. After the collision the orbit of the combined object should be circular.Em3ryguy (talk) 05:50, 5 May 2008 (UTC)[reply]
I may possibly owe you an apology for an AGF failure, that actually works, at least with one sample orbit calculation I did in an attempt to prove you were talking nonsense. However, the idea is still ridiculous. The orbital radius required is 3x105. The mass of the moon is 7.35x1022 times G=6.67x10-11 gives μ=49x1011. Plugging all that in to the vis-viva equation gives an orbital velocity of 4000 m/s. Assuming the original launch is in a highly elliptical orbit so that 1/a > 0, then the velocity of the payload prior to impact is of that, ie 5700 m/s. So, you have two objects travelling at 5700 m/s, one in a rising orbit, one in a falling orbit, meeting at approximately right angles (closing velocity is 8000 m/s). The collision is required to be elastic, a) because loss of energy will lose orbital speed and fall back to the moon and b) an inelastic collision will destroy the payloads. You therefore need some kind of catcher mechanism on the payloads. I cannot even begin to imagine a mechanism that would work at those speeds without, as I said before, resulting in crushed, not to say vaporised, payloads. SpinningSpark 09:27, 5 May 2008 (UTC)[reply]
I've made a silly mistake in the calculation above. The orbital radius is not 3x105, it is 3x105 plus the radius of the moon which comes out to 2x106. Luckily for the world and the environment, I do not launch rockets professionaly. All the velocities are around 40% of that stated. The closing velocity is 2400 m/s (4800mph). Still a lot more than can be handled by a catcher but launching in a less elliptical orbit would considerably reduce the closing velocity. The limiting circular orbital case has zero closing velocity. I am beginning to change my mind, there may be some mileage in this idea after all. SpinningSpark 11:04, 5 May 2008 (UTC)[reply]
You are using absolute velocities. You have to use relative velocity. they are both in nearly circular orbits so their relative velocity should be small. The simplest way to estimate the relative velocity would be to determine their average absolute orbital velocity then calculate the angle at which their orbits cross.Em3ryguy (talk) 11:27, 5 May 2008 (UTC)[reply]
Is my statement that the limiting case of circular orbits has zero closing velocity not the same thing? For the calculation above of highly elliptical orbits the closing velocity is correct and they will meet at right angles ie at the individual absolute velocities. SpinningSpark 12:15, 5 May 2008 (UTC)[reply]

Ignore the station for a moment and concentrate on each payload. A payload is ~200 tonne. If yo uuse a mass driver on the moon's surface and have only small thrusters to use later, the mass driver cannot place the payload onot a circular orbit. The payload goes ballistic as it leaved the mass driver, which means that it is in a lunar orbit that will come back to the same point (i.e., the mouth of the mass driver) unless quite a bit of additional energy is applied somewhere, and the thrusters you describe cannot do this. So you need to find a way to impart a supstantial amount of energy to your payload after it leaves the mouth of the mass driver, If your mass driver is aimed horizontally from the top of a very high peak, You will have an entire lunar orbit in which to add energy before the payload hits the mass driver, and you will not need to add very much energy. to get into a non-intersecting orbit (an extremely low lunar orbit.) From there, you can continue to add energy at a slow rate until you reach the 300Km orbit. The most cost-effective way to add energy will depend on your scenario. One possibility is to place a small mass driver on the payload and shoot a few rocks backwards to get into extremely low lunar orbit, and then use an ion drive for the rest of the (slow) trip to 300Km. You would have a many payloads in transit. Salvage the mass driver and ion drive when the payload reaches teh station and return them to the surface to use on the next payload. -Arch dude (talk) 23:16, 4 May 2008 (UTC)[reply]

It is possible to do it without circularising the orbit, you just have to fire the payload at sufficient speed that it's apoapsis meets the platform. You then need some way to catch the payload, and it will be travelling at a relative velocity significantly greater than the 0.1 m/s the OP was after. You would then need quite significant station keeping efforts to stop the the platform moving. I think the end result is that you either need to relax the requirement on the catching velocity, or relax the requirement on how much thrust the payloads can generate (using rockets, ion drive, tethers, whatever). Either would work. --Tango (talk) 23:46, 4 May 2008 (UTC)[reply]
For the station to "catch" the payload, it must deliver enough energy to the payload to move the payload from a lunar-intersecting orbit to a 300Km circular orbit. A passive "catch" will merely average the momentum of the station with the momentum of the payload. After many "catches" the station will intersect the moon if the payloads have the same momentum vector. Aha! is there a set of mass driver positions on the moon's surface that can impart a zero average momentum on the station? Each payload will need to contribute enough total energy to circularize its own orbit, and the station will need to store the extra energy, so we do not have a zero-velocity intercept and we need an energy-transfer mechanism at the station such as another mass driver acting as a catcher. -Arch dude (talk) 00:45, 5 May 2008 (UTC)[reply]
A rotovator can solve the <0.1m/s problem. Here's a NASA study into how it might be done from Earth [24]. In the case of the moon you might not need to launch at all, just pick it up off the surface!SpinningSpark 03:10, 5 May 2008 (UTC)[reply]
I don't understand... if each payload can circularise its own orbit, we don't have a problem... --Tango (talk) 12:56, 5 May 2008 (UTC)[reply]
The constraints of the original question imply a ballistic launch with just the capability for "minor corrections". Circularising the orbit in nearly all launch scenarios is a major delta-v event and has gone outside the scope of the question as stated IF achieved from the payloads own on-board resources. Anyway, I was only trying to think outside the box and give an answer more interesting than rockets. Picking up the payload directly from the surface with a rotating tether certainly fits that bill imho. SpinningSpark 18:08, 5 May 2008 (UTC)[reply]
I was replying to Arch dude, not you. --Tango (talk) 21:53, 5 May 2008 (UTC)[reply]
No one has looked at the viability of the proposed launch system yet. Allow me to have a go. From my previous calculation above (both the flawed and the unflawed versions) we have,
and it is going to be convenient to use;
The radius of the moon is 1.7 x 106 For an orbit as near circular as possible to give good match with the station a near horizontal launch is needed that just makes it to the 0.3 x 106 height required. The semi-major axis of this orbit is;
and the launch velocity required is given by;
The length of the linear accelerator required can be determined from;
since the accelerator is specified to be 4G acceleration
and,
You need to build an accelerator around 40km long to make this work. SpinningSpark 12:07, 5 May 2008 (UTC)[reply]
So get those solar powered bulldozers to work piling up lunar soil to make the launch platform. Edison (talk) 12:44, 5 May 2008 (UTC)[reply]
40km sounds doable to me, given the kind of future engineering we're talking about. I think the accelerator is possible. --Tango (talk) 12:53, 5 May 2008 (UTC)[reply]
If it left the lunar surface tangentially at a point and continued in a straight line 40 km. how high would the terminus have to be? This would determine, along with the angle of repose of lunar soil, the mass which would be needed to construct it, unless it was built like a roller coaster out of a trellis structure or a series of towers. Edison (talk) 18:52, 5 May 2008 (UTC)[reply]
According to the back of my envelope, the end would be 460m high. In lunar gravity, that shouldn't be too hard to achieve. I'm not sure the track needs to be level, though, it may be able to follow the surface without too much difficulty. Or, you could just find somewhere with an appropriately angled hill (or close enough, and make just small adjustments). --Tango (talk) 19:27, 5 May 2008 (UTC)[reply]
40km is no big deal -- we build structures on that scale all the time. Based on similarly large structures (roads, railroads, etc.), assuming you had enough manpower, it would take about a year to build. --Carnildo (talk) 21:39, 5 May 2008 (UTC)[reply]
Building it straight-line (ie tangential to the curved surface of the moon) is uneccessary. It is only the angle at the point it leaves the accelerator that counts. It could be built on a geodesic (lunadesic?) with just the final section angled to the desired trajectory - like an aircraft carrier deck. However, you may not even want to do this depending on the orbit you are trying to achieve and the payload recovery scheme adopted. A flatter track gets a less elliptical (more circular) orbit and it may only be necessary to angle it up sufficiently to clear obstacles like moon mountains. SpinningSpark 06:23, 6 May 2008 (UTC)[reply]

This is the dad who asked the question - it's over thirty years since I did physics. Lunar orbit velocity is about 1820m/s, so I see the problem as horizontal and vertical components. Horizontal component to achieve is 1820m/s, and vertical component needs to be such that lunar gravity of 1.622m/s/s will decrease it to zero at 300KM height. Thus, a small amount of force should then be able to circularise the orbit as the payload is already at orbital velocity. But I am not sure if my logic is correct. I also can't recall how to calculate the vertical component, and thus the angle of launch and the launch velocity and the length of the launcher. The 4G (i.e. 40m/s/s) is a nominal figure. Thanks for the comments guys. —Preceding unsigned comment added by 192.158.61.140 (talk) 06:28, 7 May 2008 (UTC)[reply]

I'm not sure you can just resolve it into components like that, since which direction is "horizontal" will change with time (due to the curvature of the Moon's surface). I think the conventional way to put something in orbit using a mass driver would be to launch (almost) completely horizontally and then, at apoapsis (greatest distance), fire rocket motors (or use any other form of propulsion) to circularise the orbit. My back of the envelope calculations show that the required delta-V for the launch would be around 1.8km/s and the required delta-V for the circulisation would be around just 0.1km/s. I expect you can afford to just slap a rocket motor to the payloads and get your 0.1km/s, it seems pretty small in comparison to the energy you're using in the initial launch (although that energy doesn't need to be lifted, which obviously helps quite a lot). --Tango (talk) 01:57, 8 May 2008 (UTC)[reply]

The Solid

Hello. I dissolved a solid into water. I added silver nitrate and the solution turned milky white. I added barium nitrate and the solution did not change. I added potassium thiocyanate and the solution turned reddish brown. I know that I am not allowed to post homework questions but I guess that the solid is a salt like sodium chloride. What is the solid? Thanks in advance. --Mayfare (talk) 17:15, 4 May 2008 (UTC)[reply]

"I know that I am not allowed to post homework questions[...]What is the solid?". Oh come on now...at least try posting your reasoning in detail and asking someone to see if you are doing it correctly and getting the right answer. DMacks (talk) 19:36, 4 May 2008 (UTC)[reply]
You will probably want a table of solubilities. Your textbook may provide a simple one; we have Solubility table which may be a bit overwhelming. Check whether each combination of the ions present in solution will precipitate. Nimur (talk) 16:31, 5 May 2008 (UTC)[reply]

gf has atopy

Hey. I like my girl, and I'm attracted to her, but she has pretty bad atopy all over her body and it's a real turn-off. It's hard because I can't be honest about it, and here (Japan) there are a lot of people with atopy but I'm not sure if they give them proper treatment because they have a tendancy (the country and the people do) to be scared away from stronger drugs. That means that she might be stuck with it for life. I don't know what to do because it's really hard for me when we get intimite and stuff, but I don't want to be a heartless asshole to her. Any advice? Heh. 218.229.72.166 (talk) 20:28, 4 May 2008 (UTC)[reply]

I assume you're referring to eczema, in which case I don't think we can help you. I don't think there is anything non-medical you can do about it, and we can't give medical advice. I suggest you talk to her about it and suggest she get treatment, if she isn't already. As for what treatment to get, I'll leave that to the doctors. --Tango (talk) 20:45, 4 May 2008 (UTC)[reply]
Yeah eczema. The problem is she is getting treatment, or at least I'm pretty sure she is. It's a very common to see people with eczema around Tokyo (from what I can tell); I think it has to do with genes and the environment that they are forced to live in. I think it also has a lot to do with the treatment they receive here. There are loads of medicines that you can get in the west (let's say the US) that you can't get here; most people won't touch birth control pills, Tylenol is very hard to come by (and the most of the alternatives suck), and even stuff like protein supplements are incredibly weak here. Original research of course. Hmmm... 218.229.72.166 (talk) 21:09, 4 May 2008 (UTC)[reply]
It can sometimes help to specifically ask the doctor for something stronger. They may be reluctant to suggest it, but might still prescribe it if asked. --Tango (talk) 22:16, 4 May 2008 (UTC)[reply]
I guess you've already read Atopy, Eczema and Atopic dermatitis. Our Allergy article may also hold some clues. Treatment varies by country and individual physician, but no one has come up with a "cure all" or "magic bullet" yet. This is not a case of one place having a cure and patients in the other not having access to it. Individual causes and results vary and some treatments like Corticosteroid can turn out to be worse than the original condition. Avoidance of the allergic trigger is your best bet. That may be easy, like a change in diet, switching detergents, avoiding certain materials in clothing and bedding, or it may be impossible if the trigger is ubiquitous. What is beneficial to some (e.g Omega-3 oils, oatmeal cream) can be a trigger to others (sea-food allergy, grass allergy). Finding a trigger can be a lengthy process. Rotation diets that avoid related food families for 2 weeks in succession could help narrow down food triggers. (OR: that worked for my cousin who had tried everything including cortisone and psychotherapy). Long holidays in as different an environment as you can find can also aid in narrowing things down. Good luck. --71.236.23.111 (talk) 23:06, 4 May 2008 (UTC)[reply]
Careful - you're bordering on giving medical advice there. We can't do that. If the OP wants medical advice, he needs to go to a doctor. --Tango (talk) 23:33, 4 May 2008 (UTC)[reply]
Sorry, my bad. I had not intended to. Despite the fact that it very rarely happens we didn't have any page on rotation diet or the like (maybe I just haven't found it yet under a different heading?). What it so puzzling to some people who are new to allergies is that there are no clear answers. It's not like some other conditions where "If A, apply B" gets comparatively consistent results. That's why you're way ahead in the game once you've identified the triggers. I certainly did not want to give any medical advice. I was merely trying to caution against second guessing the lady's physician too hastily, just because there are no immediate results. Sorry for not phrasing it less ambiguously. 71.236.23.111 (talk) 04:39, 5 May 2008 (UTC)[reply]
I can't give any medical advice (not just because it's not allowed, but because I don't have any to give). However, I can give relationship advice, and however you choose to deal with it, I would suggest you don't just try to ignore it. If it really bugs you now, it's not likely to bug you any less over time. You don't have to be a heartless asshole to say -- nicely and without sounding like you're pressuring her into anything, and certainly not when you're about to be intimate -- that hey, honey, I think they might be able to do something about the eczema. If the condition is treatable, then you're both suffering from it for no good reason; I doubt she's enjoying it either. And when I say nicely? I do mean nicely. "You should get that looked at," for example, doesn't qualify. "I hate to say this, but that really turns me off," doesn't, either. "You know, I just read somewhere that the new drugs for treating that type of skin are really efficient, have you thought about trying them out?" would probably work better. I mean, if she's the type that is going to absolutely take offense no matter what, then it'll happen, but at least you can minimize the damage. I mean, this is the type of stuff people have to learn to talk about in relationships anyway, so you're going to have to bite the bullet sooner or later -- if not on this, then on something else of a comparable nature... -- Captain Disdain (talk) 01:43, 5 May 2008 (UTC)[reply]
your relationship probably won't last very long if you are "turned-off" by your partners body. If she's receiving treatment, then the problem is really yours, not hers. If you can't learn to adjust, or at least be honest with her about how you feel (despite potentially activating her insecurities about her disorder) then you are probably never going to be that happy with the realtionship. Consider at least confiding in a friend about this or if you have one, talking to a therapist. --Shaggorama (talk) 09:44, 8 May 2008 (UTC)[reply]

The science of baseball

Please inform the intellectual side of my brain about the science of baseball. The physics of throwing fastballs, the advantage of a pitchers mound, how atoms play a part. Etc. I hope it isnt too far out to suggest quantam mechanics, and/or 'the butterfly effect" will affect the outcome of the game. If it weren't for page space, bandwidth, etc I would only halfway jokingly ask you to break down a typical game atom by atom :)--Baseball and and and Popcorn Fanatic (talk) 21:44, 4 May 2008 (UTC)[reply]

Quantum mechanics will have no noticeable effect on baseball - it only has a affect on very small scales, eg. individual atoms and smaller, a baseball is much larger than that. Chaos theory (the butterfly effect) may have a small impact, but I doubt anything particularly significant. Newtonian mechanics should be all you need. You might find our Trajectory article interesting. Aerodynamics also plays a role - particular if you put spin on the ball, as I understand it. If you want to get down to the atomic level, the reason the ball bounces back when it hits the bat (rather than going straight through - matter is mostly empty space with just a few tiny particles every now and then, so there would be plenty of room for the ball to go through the bat) is due to the electrons repelling each other due to electrostatics (it's probably much more complicated than that - it's not my field, but I think that's the basic idea). If you have any specific questions about a particular part of the physics of baseball, feel free to ask, otherwise I'm not sure what else to say. --Tango (talk) 22:13, 4 May 2008 (UTC)[reply]

Some people ("grr...mother!") think I read too much into this game. When I say I want to know everything, I mean EVERYTHING! Every player's RBI & slugging percentage. Every players on base history all the way back to college. Even the molecular structure of the beer consumed by the drunk standing next to me (soon to be vomited on my shoe). Whats the psychology behind him? I'll bet he treats his kids bad.

I also want to know where I can get information on how all MLB teams chose thier uniforms.We all like vacuum cleaner physics and other trivial science stuff. That's my argument here.--Baseball and and and Popcorn Fanatic (talk) 22:35, 4 May 2008 (UTC)[reply]

Googling for "The science of" baseball gets you several thousand places to look. Please feel free to ask for specific information or explanation. Otherwise it might be easier to define which wikipedia pages are not likely to have much that in some small way affects some aspects of the game or event of baseball. Another way to go is to start with one aspect like e.g. Newtonian mechanics, Beer or Decision making and then work your way to the pages that link there. Happy studying. 71.236.23.111 (talk) 23:34, 4 May 2008 (UTC)[reply]
Doc Edgerton actually did some work studying baseball physics. Check it out. --98.217.8.46 (talk) 02:03, 5 May 2008 (UTC)[reply]
Seems to me that you're looking for Adair's The Physics of Baseball, ISBN 0060084367. -- Coneslayer (talk) 11:45, 5 May 2008 (UTC)[reply]
Thus far, physics has been addressed, but the questioner asks for the science of baseball. What about biomechanics, sports medicine, metabolism? This is a different slice of science, but just as relevant to baseball as the physics of a ball trajectory. Nimur (talk) 16:39, 5 May 2008 (UTC)[reply]

Water table in natural pond.

Hi there,

I have recently bought a house (Wales, UK) that has a back garden that is on a slope going upwards. At the top there is an old pond that has been left to its own devices for many years. So during my landscaping I syphoned all of the water out of it and cleared the foot of leaves etc from the bottom. To my horror there was no liner, only a clay bottom. The pond is about 3M X 1.5M and roughly 0.5M deep. There are 3 areas in the pond that I sink down deeped into the mud. I left the pond over night and when I visited it in the afternoon the following day the water level was pretty much back to where it originally was. So I have 2 options, get rid completely using some sort of drain or keep the pond. The pond idea sounds very appealing as my old neighbour had a beautiful pond with Koi in it. However, due to the nature of the pond being clay it would be impractical as the fish would cloud the water and the water would clog the filters with clay. So I was wondering if I were to put a lining inside the pond would the weight of the water inside the lining equal the pressure stopping the water from rising?

Many thanks Paul —Preceding unsigned comment added by Bwganllwyd (talkcontribs) 22:19, 4 May 2008 (UTC)[reply]

Are you sure the water rose from the ground and it didn't just rain overnight? It seems odd for the water table to be that high, especially at the top of a slope... Either way, you should be fine with a plastic lining - the people at your local gardening centre should be able to advise you further. The water shouldn't push itself up any higher than it would be normally, so as long as your garden isn't flooded, the water in the ground around the pond should still be below ground level. --Tango (talk) 23:40, 4 May 2008 (UTC)[reply]
Water tends to go exactly where it wants to go, if the underground dynamics lead water to come out at the top of a hill (which is not impossible), beware of trying to fight it. Generally, a liner won't change the pressure relationship. Also, in my experience, ponds lined with clay tend to be crystal-clear, there is no particular reason that the clay will enter the water column. That said, observe Tango's comment above about the possibility of rain. Franamax (talk) 23:47, 4 May 2008 (UTC)[reply]

This sounds more like a spring or a seep. It is difficult to maintain an actual water table at the top of a slope. For a literal waters table, if you add a flexible plastic liner, then the water level will be exactly at the level of the water table. However, since the water table fluctuates, the liner will inevitably displace towards the surface. It would be better to work with nature and take advantage of the natural inflow of water. If you can arrange for an outflow, you will have a nice natural open-loop system with no need for pumps and filters. From your description I suspect that there is a hidden outflow of some sort anyway. -Arch dude (talk) 00:28, 5 May 2008 (UTC)[reply]


Hi there, Thanks for the replies so far, the first time I emptied the pond it rained and the pond filled back up, but there is no way that that amount of rain fell. I have since emptied the pond several times without there being any rain and it has still filled up. I have been seeking advice on several sites, here is a link to one of them with some pics on it: http://www.inspectorsjournal.com/Forum/topic.asp?TOPIC_ID=7257

Just to explain the pics, I am excavating land from the bottom to try to level the garden out. The pond is at the top of the garden.

Cheeers Paul —Preceding unsigned comment added by Bwganllwyd (talkcontribs) 04:44, 5 May 2008 (UTC)[reply]

Before you put down a pond liner you'd have to find out where your water comes from or what Arch dude says is right. You'll end up with an artificial pond on top of muck and the liner will "float up". I'm no pond expert, but what you took out seems to have been a natural garden pond. They are tricky to get going because you rely on a balanced ecosystem instead of pumps and filters. Things get a lot easier to maintain if you have an inflow like a spring or seep (instead of just rain and surface run-off) which seems to be the case here. That will help with mixing up thewater column and oxygenate the water, see Hypoxia (environmental). Plants provide shade to prevent excessive algae growth. Animals like snails, dragonflies and frogs usually arrive on their own after a while. The clay (or sometimes tile) bottom keeps water from draining and allows only a limited amount to percolate through the ground. [25] The tricky part is that, if you get too much evaporation in summer you may have to top the pond off with tap water and that can damage the eco-system. Natural ponds can get "stinky" at times even if they are perfectly balanced.
What you propose to put in is an artificial garden pond or water garden. "If fish are kept, pumps and filtration devices usually need to be added in order to keep enough oxygen in the water to support them." Pumps and filters will silt up easily in a clay bottom pond. So you need to put in a liner and there's the problem because the water that fills your pond now will no longer be able to get into your pond. Lisa4edit (talk) 16:12, 5 May 2008 (UTC)[reply]

Spider

Can someone identify this spider:

Please? –Sidious1701(talkemail) 23:48, 4 May 2008 (UTC)[reply]

Do you have a larger picture? I wouldn't be able to identify it with any picture, but I'm not sure anyone will be able to identify it reliably from such a small shot. --Tango (talk) 00:07, 5 May 2008 (UTC)[reply]
No, that's the only one I got...sorry. –Sidious1701(talkemail) 01:31, 5 May 2008 (UTC)[reply]
Similarly, I don't think I'll be able to identify it, but it might help others if you gave a few details about where the photo was taken, and maybe the time of day. Confusing Manifestation(Say hi!) 03:43, 5 May 2008 (UTC)[reply]
I think we can only safely say that it is an Orb-weaver spider.--Lenticel (talk) 12:05, 5 May 2008 (UTC)[reply]
I found it under a table, it was about sunset, and that's all–Sidious1701(talkemail) 20:51, 5 May 2008 (UTC)[reply]
Which country and region was the table in? Graeme Bartlett (talk) 21:36, 5 May 2008 (UTC)[reply]
It was in Louisiana, and it was outside.–Sidious1701(talkemail) 00:31, 7 May 2008 (UTC)[reply]
Is it like the A. bruennichi image (left) in Argiope article? As Lenticel, an orb weaver, found in warmer areas... Julia Rossi (talk) 09:10, 7 May 2008 (UTC)[reply]
I don't know what julia is talking about, I seem to remember louisiana being prety damned warm. --Shaggorama (talk) 09:49, 8 May 2008 (UTC)[reply]
Isn't that was Julia's saying? It's an orb weaver that was found in a warm area, so she named a species that fits that description. --Tango (talk) 18:02, 8 May 2008 (UTC)[reply]
My bad, misunderstood. retracted. --Shaggorama (talk) 19:46, 8 May 2008 (UTC)[reply]

May 5

Aresol Can

Why in Aresol Cans and spray paint cans is there ball inside that makes a noise when you shake it? x

Please reply the fate of the world rests upon it... —Preceding unsigned comment added by 80.176.73.246 (talk) 00:51, 5 May 2008 (UTC)[reply]

The ball is there to mechanically agitate the contents of the aerosol can when you shake it, that is, it helps you mix the contents. For instance, if you try spraying a can of red paint without first shaking it up, you get lots of propellant and very little paint. The world can rest safe... Franamax (talk) 01:12, 5 May 2008 (UTC)[reply]
It is very difficult and quite unsafe to insert a wooden stick into the aerosal can to stir up the paint medium and pigment, so the little ball is necessary. Edison (talk) 03:32, 5 May 2008 (UTC)[reply]

Other "dry ices"?

Moved from Talk:Dry ice: Is there any other substance that forms a "dry ice"? Samw (talk) 03:09, 5 May 2008 (UTC)[reply]

Most substances have some range of pressure and temperature at which sublimation happens. Read the article for other examples of normal temperature and pressure sublimation. SpinningSpark 03:21, 5 May 2008 (UTC)[reply]
Thanks! So would you call naphthalene a "dry ice"? Or does the concept of "dry ice" even make sense scientifically and it's strictly a marketing term (which it was originally)? Samw (talk)
I wouldn't call it "a dry ice". Actually I've never thought of "dry ice" as a generic description for anything, rather just a name for a specific thing (solid CO2). Naphthalene is not "ice" in the colloquial sense (it's not cold), so I would just call it a "solid". Maybe a "volatile solid". DMacks (talk) 05:19, 5 May 2008 (UTC)[reply]
Usually "ice" refers to water ice, and the special exception of dry ice is the name for solid CO2. Not all cold solid crystals are ice; otherwise, steel and copper and quartz could be "ice". Nimur (talk) 16:43, 5 May 2008 (UTC)[reply]
In astronomy, they call any solid an ice that is lighter than rock. Ammonia ice and methane ice for instance as well as water and carbon dioxide.SpinningSpark 16:50, 5 May 2008 (UTC)[reply]
Getting slightly off topic, but I think astronomy is a bad guidance for such definitions. For astronomers, stars consist of hydrogen, helium and "metal"---where "metal" is everything heavier than helium... --Dapeteばか 19:11, 5 May 2008 (UTC)[reply]

To have something resembling "dry ice" you need to have a substance with a triple point pressure that is higher than the atmospheric pressure and a sublimation point below room temperature (this second requirement is so that the substance feels cold). From the common substances listed in this table [26], only acetylene (and CO2) meets the requirements, assuming you want your dry ice to be dry at sea level, which is 101 kPa. If you are willing to move up to the mountains, a couple other substances could work. For example, xenon and nitrous oxide could form dry ices in Mexico City, where the ambient pressure is around 78 kPa. And if you use a vacuum pump with low enough pressure, many other substances can be made to behave like dry ice. A couple of the substances in the table, graphite and uranium hexafluoride, have high enough triple point pressures so that they will never be a stable liquid under normal pressure, but their sublimation point is higher (for graphite much higher) than room temperature, so I am reluctant to call them "ices". --Itub (talk) 12:52, 6 May 2008 (UTC)[reply]

Power to weight ratio

When examining the power to weight ratio of an automobile, comparing it to another, horsepower per lb. or kg. is considered. I'm wondering if a torque to weight ratio should be given consideration. If so, should it be given equal consideration as horsepower, a certain amount of consideration, or none at all? Note: peak RPM, aerodynamics, and other values are not important to me at this time, just power to weight ratio. Thanks. MoeJade (talk) 04:41, 5 May 2008 (UTC)[reply]

That's a good question/comment. I venture to say yes. One of the most important factors when considering the acceleration of an automobile is the power to weight ratio, however HP is given as peak. Torque peaks at lower RPMs, so I would say that an important variable would be low end torque to weight ratio. That would be an excellent criteria for determine the cars ability to accelerate quickly. Wisdom89 (T / C) 04:45, 5 May 2008 (UTC)[reply]
Depending on the circumstance, it should definitely be given consideration. Wisdom89 (T / C) 04:46, 5 May 2008 (UTC)[reply]

-Thanks for the fast response, Wisdom89! So would you give torque to weight ratio equal consideration, averaging power to weight and torque to weight together? Or does power to weight take precedense (sp?), with torque considered, but not given as much weight? MoeJade (talk) 04:54, 5 May 2008 (UTC)[reply]

That's tough. I think if you were going to calculate both ratios, they should be kept separate. You'll probably find a positive correlation between the two anyway. In other words, high power to rate ratios correspond to high torque to weight rations. With such a relation, the power to weight would probably take precedence. Wisdom89 (T / C) 05:35, 5 May 2008 (UTC)[reply]
Another thing that we have to bear in mind is that at any given torque, the engine is producing a certain amount of (horse) power, so that reinforces our positive correlation. This is probably why you never see the torque to rate ratio advertised or touted about. Wisdom89 (T / C) 05:46, 5 May 2008 (UTC)[reply]
Technically that is true only if the vehicle is moving. If it isn't currently moving, mechanical work is not being done (and thus output power is zero). But that may not be a very important measure for most people anyway. Most of the time I think you only have to worry about overcoming inertia (and some small amount of static friction), so overall torque or force doesn't matter all that much. --Prestidigitator (talk) 08:29, 5 May 2008 (UTC)[reply]
Even if an engine is idling and the crankshaft is being spun, then power and torque are still being generated. Wisdom89 (T / C) 12:44, 5 May 2008 (UTC)[reply]
I was trying to point out a case where the two would not be correlated. That wouldn't be at idle but instead when the engine is engaged and trying to move the vehicle when there is a significant force resisting initial acceleration from a stand-still. That is a case where force (torque if we want to ignore gearing and wheel characteristics for the vehicle) is going to be significant, but output power is zero until it "breaks free". While weight might increase the amount of static friction that has to be overcome a bit, I think significant resistive force would really come if the vehicle is stuck or starting on an up-hill gradient.
Besides, aside from that needed to overcome a small amount of friction, no torque/power is required to keep the crankshaft moving at idle. Your efficiency for any engine during that condition is going to be very close to zero, and attempting to measure the amount of torque/power is likely to significantly change the value anyway unless you are very careful about how you measure/deduce it. --Prestidigitator (talk) 18:55, 5 May 2008 (UTC)[reply]

O.k. Good input. Basically, a friend and I were comparing two vehicles for potential acceleration. Car A has 221 HP, 236 f/lb. of torque, and weighs 1430 kg. Its power to weight ratio is 0.154, or 0.154 HP per kg. Car B has 216 HP, 152 f/lb. of torque, and weighs 1180 kg. Its power to weight ratio is 0.183, which is notably better than car A. However, car A has 236 f/lb. of torque, or a torque to weight ratio of 0.165, which is substantially better than car B's torque to weight ratio of 0.128. Observing power to weight ratio only, car B wins easily. But, with car A's significant amount of torque,and its significant torque to weight ratio, it's a tougher choice than it looks, to me, anyway. Any additional comments on this are welcome. MoeJade (talk) 13:52, 5 May 2008 (UTC)[reply]

I think Car A would have better low-end acceleration, but Car B would eventually go faster. However, gearing ratios would also come into play. If you can find the cars listed in a Road & Track or Car & Driver magazine test, they often show detailed full-on acceleration numbers. Franamax (talk) 16:29, 5 May 2008 (UTC)[reply]

Does an Electron Occupy Space? Why?

There is a paragraph in article Electron I don't get it well:

"The electron is currently described as a fundamental or elementary particle. It has no known substructure. Hence, for convenience, it is usually defined or assumed to be a point-like mathematical point charge, with no spatial extension. However, when a test particle is forced to approach an electron, we measure changes in its properties (charge and mass). This effect is common to all elementary particles. Current theory suggests that this effect is due to the influence of vacuum fluctuations in its local space, so that the properties measured from a significant distance are considered to be the sum of the bare properties and the vacuum effects (see renormalization)."

As my understanding, it tells us that an electron DOSE occupy space (its radius is 2.8179 × 10−15 m according to the article) and also it somewhat explains why an electron occupies space. But I'm afraid it doesn't help for understanding the reason why electron occupies space, especially:

  • "when a test particle is forced to approach an electron, we measure changes in its properties (charge and mass)"
  • "the properties measured from a significant distance are considered to be the sum of the bare properties and the vacuum effects"

the sentences above are confusing to me...dose anyone know what do they mean?

Additional question: What if we force two electrons collide? Will they just overlap with each other and then pass through without collision?

(solving the questions above might involve some QFT and/or QED but they are too tough to me) - Justin545 (talk) 06:06, 5 May 2008 (UTC)[reply]

While that paragraph does leave much to question, I think the answer is definitely yes. The electron does occupy space. It is just really small compared with the nucleus of the atom. Thus, when a test particle is forced to approach the electron, it causes changes that can be measured. The problem with small things is that trying to measure and define their properties actually changes their behavior. So, we can only guess about their real behavior and measure from a distance so as to not affect the results.

In answer to the second question, since they occupy space, they can't just go through each other. They will collide and bounce apart. Leeboyge (talk) 07:22, 5 May 2008 (UTC)[reply]

I'm not sure. But I think things collide that's because of the fundamental interactions (strong interaction, weak interaction, electromagnetic force and gravitation) between them. If we are able to overcome the repulsion and all other forces between the two electrons, I think they will overlap and go through each other when they approach to each other. Even though they occupy space! (please correct me if I'm wrong) - Justin545 (talk) 08:14, 5 May 2008 (UTC)[reply]

Sticking my nose temerariously into a question well outside my expertise: I think the answer is that, as of 2008, no one really knows whether the electron (in the sense of the bare charge, not the cloud of virtual electrons and positrons that surround it) occupies space. My understanding is that QED takes it to be a point charge, occupying no space, and having therefore infinite density (and by the way also infinite charge -- the cloud of virtual particles surrounding it "shield" most of that charge). QED works extraordinarily well, but the extent to which it faithfully represents the underlying reality, as opposed to simply being a collection of hacks that get the right answer -- again, nobody knows. --Trovatore (talk) 08:28, 5 May 2008 (UTC)[reply]

Telling is the sentence after the statement of the electron's "radius": "This is the radius that is inferred from the electron's electric charge, by using the classical theory of electrodynamics alone, ignoring quantum mechanics." I take that to mean it helps in some physical problems (collision cross sections maybe?), but shouldn't necessarily be interpreted as a literal "size" as we think of it in intuitive macroscopic terms. --Prestidigitator (talk) 08:40, 5 May 2008 (UTC)[reply]

I'm afraid it depends on what you mean by "occupy space". If you want the everyday-scale notion of occupying space to extend down to the quantum level then I think you're pretty much forced to say that electrons do occupy space, since most of the space supposedly "occupied" by solid objects only has electron orbitals in it. The space occupied by an electron in this sense has nothing to do with any intrinsic size. A hydrogen atom and a He+ ion both have a single electron orbiting a nucleus, but the former is larger than the latter because of the different nuclear charge, even though it's an identical electron that occupies most of the space.

Protons and neutrons do have an intrinsic size: they're bound states of more fundamental particles (much like an atom is), and the bound state has a characteristic radius (much like an atom does). People have proposed preon theories where the electron is a composite particle, but none of them have had much success. If an electron were composed of preons then I suppose it would have a size in this sense, but I don't know much about this.

Strings (from string theory) have a characteristic size but don't really occupy space, being one-dimensional subsets of a three-or-more-dimensional space.

It seems fairly likely that future physical theories won't have a concept of "space" any more (except as a large-scale approximation), which will make this question even harder to answer.

2.8179 × 10−15 m is the classical electron radius. Ignore it, it's meaningless. -- BenRG (talk) 10:28, 5 May 2008 (UTC)[reply]

I could be wrong, but I think what happens if you collide electrons is Møller scattering. --98.217.8.46 (talk) 15:17, 5 May 2008 (UTC)[reply]

To make the conclusion, I presume:

  1. (classical and modern view) An electron occupies NO space. It is just a macroscopic illusion to say that an electron occupies space as the article Electron did.
  2. (classical view) An electron is just a tiny, movable electric field with definite position. Because the movable electric field has mass and inertia, when it feels a net force it will accelerate with .
  3. (classical view) Because the electron is nothing more than an electric field with mass , when we force two electrons move toward to each other they will finally pass through each other WITHOUT collision. Moreover, they will overlap (perfectly) halfway while they move toward each other. (however, Pauli exclusion principle tells us "no two identical fermions may occupy the same quantum state simultaneously" so I'm not sure if it holds in modern physics)
  4. Electrons are not solid/stiff objects/entities so they don't collide. The collision is just an illusion due to the repulsion Coulomb forces between them. So saying an electron occupies space is meaningless.
  5. The sentences "Hence, for convenience, it is usually defined or assumed to be a point-like mathematical point charge, with no spatial extension. However, when a test particle is forced to approach an electron, we measure changes in its properties (charge and mass)." in Electron should be removed or rewritten as it misleads the reader to think an electron occupies space.

the conclusion above may sound ridiculous. Please point out my errors if any. - Justin545 (talk) 01:53, 6 May 2008 (UTC)[reply]

Two electrons can't be forced to move towards eachother until the overlap, since that would require infinite energy (the force follows an inverse square law, so approaches infinity as they electrons approach each other). --Tango (talk) 16:56, 6 May 2008 (UTC)[reply]
This is false. Because electrons are ultimately probability clouds, the amount of charge at any point in space is infinitesimal. Infinitesimal charges can be seperated by neglible distance without requiring infinite energy, and as a consequence electron clouds can overlap (and do routinely in all atoms with more than 1 electron). Dragons flight (talk) 18:51, 6 May 2008 (UTC)[reply]
True, but Justin listed that item as being the classical view, and it's incorrect from the classical viewpoint. --Tango (talk) 12:33, 7 May 2008 (UTC)[reply]
From the classical viewpoint, two electrons could not overlap because . If there will be infinite potential.
From the quantum viewpoint, two electrons could overlap. For example, suppose is a wavefunction of two electrons with position , respectively. Then will be the probability of finding the first electron between interval and the second electron between interval . If is well-designed such that:
  1. and overlap. (which implies and )
  2. is very close to so they are almost at the same point
then we have a chance (with non-zero probability) of finding the two electrons arbitrarily close to each other! However, two electrons overlap may sound like to violate Pauli exclusion principle. But as long as the two electrons with different momentum they are not in the same quantum state. (again I could be wrong. please point out my errors) - Justin545 (talk) 02:11, 7 May 2008 (UTC)[reply]
I added scare quotes to "classical electron radius" in the electron article because it's not really an electron radius, but that doesn't mean electrons don't occupy space! There's no Platonic notion of occupying space that we can appeal to here. It's a matter of how you want to define the English phrase. If you define it in such a way that electrons don't occupy space, then neither does anything else, which makes the phrase useless. I think that's reason enough to use some other definition.
Also, it's pretty hard to make the case that electrons are at all point-like to begin with. In the original quantum mechanics (now taught to undergrads) you start with a quasiclassical theory where electrons are point particles and then "quantize" that, which introduces wave behavior. (The scare quotes here are because quantization (physics) isn't actually quantization. Horrible terminology.) But in quantum field theory you start with a quasiclassical field theory of electrons, so even at the classical level the electrons are spread out and occupy space to the same extent that electromagnetic fields do. When you "quantize" this theory you get field phonons which are referred to as particles, not because they are but because the term was already grandfathered in by the time QFT was developed. The only sense in which electron field quanta are particle-like is that they tend to spatially localize when they interact with a thermodynamically irreversible system (like a cloud chamber). I still don't grok why this happens, but it has something to do with quantum decoherence.
Probably the most misleading part of QFT is Feynman diagrams. They're not pictures of pointlike particles propagating and interacting in spacetime, they're graphs (in the computer-science sense) and their origin is combinatorial. Any integral of the form , where P is a polynomial, can be evaluated with Feynman diagram techniques (for some value of "any"). The idea is to write P = Q + R where Q contains only terms of degree 2 or less, and then Taylor expand R to get , which gives you a series of relatively easy Gaussian integrals. The term of this series can be represented by a collection of Feynman diagrams with interaction vertices. The diagrams always look like QFT diagrams with lines and interaction vertices, whether or not there's anything resembling space or time in the integral. There's a nice discussion of this in chapter I.7 of Quantum Field Theory in a Nutshell by A. Zee, which I just found out is available online. This expansion is extremely useful in QED, where the series converges rapidly, but not so useful in QCD, where it doesn't. There are other approaches to evaluating the integral which give you a totally different "picture of what's going on". I love the book QED, but Feynman went way overboard with his claims about the particle nature of light and electrons. In fact chapter 2, where he talks about photons only, is completely classical in almost every respect. The idea that pulses of light take every possible path from the source to the target, and every path contributes equally to the final amplitude, is just the path-integral form of Maxwell's equations. His discussions of reflection, refraction and diffraction are classical. It's crazy to claim that these arguments show the particle nature of light unless you're willing to argue that Maxwell's theory is also a particle theory. What does show the particle nature of light is the fact that the detector (photomultiplier) registers individual clicks of equal amplitude in low light. That's the only genuine quantum mechanics in the whole chapter. -- BenRG (talk) 16:33, 7 May 2008 (UTC)[reply]
BenRG: do you really think those scare quotes are necessary? I went back and saw your edit. I think "So called" is a little harsh as well. Granted, contemporary theory no longer describes the electron this way, but the paragraph qualifies the term as such. Scare quotes would imply that there is something intrinsically wrong with the phrase, which there is not. Qualifying the radius described as Classical gives it a timeframe during which the description was perceived as accurate theory. It would be like putting scare quotes around phlogiston or n-ray. Thoughts? --Shaggorama (talk) 19:57, 8 May 2008 (UTC)[reply]

Question regarding electricity

This isn't homework, it's part of my revision material and I need some help with understanding it. I'm not asking for an answer.

A filament bulb is connected to the 230V mains power supply and a meter indicates that 31 coulombs of charge flows in a period of 60 seconds. Calculate: a) the total electrical energy transferred to another form by the lamp b) the power dissipated by the lamp

Am I right in assuming that 31 coulombs of charge per 60 seconds is roughly 0.52 Amps since an amp is the flow of charge per second? My problem is i've forgotten whichever equation to use to calculate the electrical energy transferred.
As for part B, i'm right in assuming I need to use P = I^2 x V for this?

Regards, CycloneNimrodTalk? 10:47, 5 May 2008 (UTC)[reply]

Your 230V mains is AC Alternating current.71.236.23.111 (talk) 11:38, 5 May 2008 (UTC)[reply]
Your revision question is badly worded, but your calculation is right. You need Joule's Law to work out the energy and power. --Heron (talk) 12:22, 5 May 2008 (UTC)[reply]
You are right on the first statement but the second formula you give is wrong, it is . Ignore the statement above about alternating current, that has nothing to do with it. SpinningSpark 12:28, 5 May 2008 (UTC)[reply]
Thanks for your replies :) Regards, CycloneNimrodTalk? 17:18, 5 May 2008 (UTC)[reply]

A curiosity of the English usage difference between the UK and the U.S. is the British use of "revise" when we would say "review" or "study." On this side of the pond, to "revise" would only mean "to edit and make changes." Edison (talk) 18:44, 5 May 2008 (UTC)[reply]

You're editing and making changes to your knowledge of the subject, I suppose. --Tango (talk) 19:18, 5 May 2008 (UTC)[reply]
For once I'd agree that the U.S. grammar in this sense is correct, I think it makes more sense to 'review' than to 'revise', but alas, it's what i've been taught to say ;) Regards, CycloneNimrodTalk? 19:38, 5 May 2008 (UTC)[reply]
Actually the word is French in origin (ultimately Latin) and literally means to "to look at again" so it is entirely cognate with review. I suppose that the modern sense of "altering" arose because it is natural to find fault and change something after reviewing it. Anyway, when I was at school I felt it was not insignificant that the word was next to "revile" in the dictionary which is exactly how I felt about the process. SpinningSpark 20:06, 5 May 2008 (UTC)[reply]

GMT and UTC

How different are these two times? Is the difference always the same or does it vary?

Thank you. Wanderer57 (talk) 13:34, 5 May 2008 (UTC)[reply]

Try reading GMT and UTC, I expect you'll find your answer in them somewhere. --Tango (talk) 13:47, 5 May 2008 (UTC)[reply]
Here's a short answer. The term "UT" was basically invented as a replacement for "GMT" that, for political reasons, did not mention Greenwich. As time measurement improved, it was realized that GMT (UT) as it had been interpreted did not provide a suitable basis for timekeeping, and several kinds of UT were developed: UT0, UT1, UT2, and UTC. You can think of them as kinds of GMT if you like, or you can think of GMT as meaning a specific one of them. They all vary with respect to each other, but UT1 and UTC are never more than 9/10 of a second apart. Now see the articles for more detail. --Anonymous, 18:44 UTC, May 5, 2008.

White teeth

I've got a couple of questions I hope somebody can answer. Thank you.

  • if you drink black tea, and then brush your teeth immediately, how great is the risk that your teeth may become somewhat tainted anyway?
  • can your teeth become tainted to some degree even if you use a straw to drink black tea?
  • what kind of foods should one avoid in order to get whiter teeth? Mind you, I don't mean foods which damage teeth in general, just those which stain the teeth.

Thanks again. 83.37.4.200 (talk) 14:23, 5 May 2008 (UTC)[reply]

Look at Tannin to get a start. Eliminating all these foods from your diet doesn't seem a good idea. Antioxidants help your body stay healthy. Lisa4edit (talk) 14:54, 5 May 2008 (UTC)[reply]
No, it wouldn't be a good idea, it seems :) And as for my first question, has anyone got an answer please? 83.37.4.200 (talk) 14:58, 5 May 2008 (UTC)[reply]
I'm not sure avoiding foods will ever get you whiter teeth—what you mean is what kinds of foods can you avoid to avoid staining your teeth. That's not the same thing (your teeth will never "get whiter" on account of avoiding foods. You can, though, make them "get whiter" by various bleaches and etc., which are not at all expensive these days). Note that over-brushing is not a great idea in and of itself — you can damage the enamel, especially if you brush immediately after eating. Chewing sugarless gum after a meal is a better idea—the saliva cleans the teeth, restores the pH, and is non-abrasive. (I am not a dentist! But I read it in the New York Times not long back.) I have no clue though whether anything you do would have any effect on staining of that sort. --98.217.8.46 (talk) 15:09, 5 May 2008 (UTC)[reply]
Talking of chewing gum the Xylitol page might interest you. But that won't whiten your teeth either.Lisa4edit (talk) 15:58, 5 May 2008 (UTC)[reply]

Henry IV of France and gonorrhoea

I'm working on an account of Henry IV's love life. In his biography of Henry, David Buisseret writes: "Towards the end of the month he had another attack of gonorrhoea, which gave rise to a temporary but alarming heart condition". Could anyone tell me if temporary heart conditions can indeed be brought on by gonorrhoea? (Historians sometimes repeat this sort of thing without question, so I need to check this out.) The incident is important because Henry nearly died. Many thanks. qp10qp (talk) 15:06, 5 May 2008 (UTC)[reply]

If the bacteria spread through the body as "Disseminated Gonococcal Infection" endocarditis can occur. See http://www.ncbi.nlm.nih.gov/pubmed/1728091 William Avery (talk) 15:35, 5 May 2008 (UTC)[reply]
Many thanks. I will trust it. qp10qp (talk) 21:47, 6 May 2008 (UTC)[reply]

tuberculosis (moved from WP:HD)

what is the method of minimising resistance to treatment in tuberculosis?172.159.163.56 (talk) 16:27, 5 May 2008 (UTC)[reply]

Antibiotic resistance may be of use. Resistance to tuberculosis is pretty much the same as in other bactera. The best ways to reduce resistance are to be picky about who you give antibiotics to and to make sure that they finish their course of antibiotics. If bacteria aren't killed off completely, they'll likely come back with a resistance and hey presto, you have yourself a new strain of resistant bacteria. Of course it's not quite that simple, but it's along those lines ;) Regards, CycloneNimrodTalk? 17:16, 5 May 2008 (UTC)[reply]
The principles specific to antituberculosis treatment: [1] never use only one antibiotic; combination therapy is mandated. It is important to assess the resistance pattern of the specific strain of tuberculosis infecting the patient, as using an ineffective antitubercular may leave one effectively treating with only one (or no!) effective antibiotics. This is an exception to the usual admonition to avoid polypharmacy. [2] directly observed therapy: patient compliance with prescribed medication is monitored and enforced by public health officials. - Nunh-huh 22:54, 5 May 2008 (UTC)[reply]

Gondwanan distribution and Laurelia

There are two species of Laurelia, one native to the Andes - Laurelia sempervirens (Chilean Laurel, Peruvian Laurel, Peruvian Nutmeg) and the other found in New Zealand - Laurelia novae-zelandiae (Pukatea) .

When I saw one of these trees, growing at Enys in Cornwall, UK, yesterday, the Head Gardener said they were examples of "Gondwanan distribution". I have looked at the Gondwana article, but this is very brief on the topic, which is a REDIRECT to Godwana.

I would like to know more about the differences between the two species and when, approximately, they diverged, due to the separation of New Zealand and South America from the Gondwanan continent. Vernon White . . . Talk 19:24, 5 May 2008 (UTC)[reply]

I shall leave the differences to the botanists. As for the divergence, as I understand it, dating the Gondwanan distribution is not easy. For example, in a discussion on ferns, Te Ara Encyclopediaof NZ suggests that some distribution was windborne. this Te Ara page also discusses the time span, ocean distribution and later isolation. Googling Pukatea + Gondwanan yields 39 results which might bear investigation. Gwinva (talk) 05:17, 6 May 2008 (UTC)[reply]

engineering question

Which field fo engineering is the most multidiciplinary i.e. which concerns the widest range of topics from maths, physics, chemistry, geography, biology etc. Clover345 (talk) 20:53, 5 May 2008 (UTC)[reply]

Possibly constructions with biopolymers while exploiting advantages geological features. You know these disciplines are just groups into which we categorise knowledge, they interconnect depending on how do you define there disciplinesBastard Soap (talk) 21:21, 5 May 2008 (UTC)[reply]

Environmental engineering would be my suggestion. I'm not certain I'd recommend it as an area with stellar career opportunities, though. Lisa4edit (talk) 21:33, 5 May 2008 (UTC)[reply]
I would propose Electrical Engineering. After discussions with many of my colleagues, I have reached this conclusion. In 2008, "electrical engineering" may refer to the base science and the applied technologies in fields as diverse as:
I personally know "electrical engineers" who work in all of the above areas. The job market is huge - software companies, aerospace, communications, academia... Nimur (talk) 00:50, 6 May 2008 (UTC)[reply]
An interesting question. I would suggest that mechanical engineering would be as this covers a wide range of diciplines including electrical / electronics engineering, both on the micro scale, for example nanobots and macro scale, for example, lifts and ventilation in buildings. Civil engineering mainly covers all static objects while mechanical covers moving objects. Civil may also be multidiciplinary but usually only involve large scale building projects. Environmental engineering is a sub-dicipline of civil engineering. Tbo 157(talk) 18:04, 6 May 2008 (UTC)[reply]
Where they put Environmental Engineering depends on your U. MIT and Stanford put it with the Civs but lots of the smaller ones either have a separate department or put it somewhere else. In many other countries it is a separate discipline, too. 71.236.23.111 (talk) 06:56, 7 May 2008 (UTC)[reply]
Multidisciplinary with regard to engineering usually means it contains many aspects of engineering - e.g. electrical, civil, mechanical, electronic, aeronautical - rather than the multiple subjects you mention. However, from my experience, I would suggest that civil most closely fits your definition of multidisciplinary. There's the maths and physics, along with a lot of work on environmental/geographical elements, and fluid dynamics (wind flows). It lacks chemistry and biology though... LHMike (talk) 09:12, 8 May 2008 (UTC)[reply]

Atomic vibration

Can anyone please explain to me what makes the little buggers vibrate about?Bastard Soap (talk) 21:16, 5 May 2008 (UTC)[reply]

What kinds of atoms, in what situation? If you're talking about air at room temperature, then most of the molecules are not vibrating. They're moving and rotating, but not vibrating, because the first vibrational energy level above the non-vibrating ground state is too high compared to the average thermal energy. If you're talking about some solid material like a metal, then it is vibrating, because unlike vibrations of gas molecules, there is effectively no lower limit for the energy of a phonon.
So to sum up, the answer to your question is that when things are vibrating, they're usually vibrating because of heat. Heat is the microscopic motion of atoms. —Keenan Pepper 23:55, 5 May 2008 (UTC)[reply]
That seems a little circular to me. They're vibrating because of heat, which is vibration? --Tango (talk) 00:35, 6 May 2008 (UTC)[reply]
Heat is a form of energy. If you put energy into a system the atoms will move more, i.e. they will have more heat.--Shniken1 (talk) 00:44, 6 May 2008 (UTC)[reply]
In thermodynamics (as opposed to everyday usage) temperature and heat have very precise definitions and are not synonymous. Temperature is a measure of the thermal energy of an object, whereas heat is the energy exchanged between an object and its surroundings if they are at different temperatures. If an object is at a lower temperature than its surroundings then heat energy will flow into it, but this energy does not necessarily increase the object's temperature - the object might instead do work by expanding against an external pressure, or it might change state. Conversely, it is possible to increase an object's temperature above that of its surroundings by doing work on it - by compressing it, for example, or by boring a hole in it - or to reduce its temperature by making it do work - by allowing it to expand, for example. Gandalf61 (talk) 09:34, 6 May 2008 (UTC)[reply]
Heat is the macroscale result of atomic level vibration. They are the same. The faster the atomic vibration, the higher the bulk sample temperature. —Preceding unsigned comment added by 131.111.100.44 (talk) 16:30, 6 May 2008 (UTC)[reply]

I knew that heat makes them vibrate, my question is why is it so?Bastard Soap (talk) 18:32, 6 May 2008 (UTC)[reply]

Heat is energy. If you add heat to a molecule, either it becomes higher in kinetic and/or potential energy. Motion is kinetic energy... DMacks (talk) 21:48, 6 May 2008 (UTC)[reply]
Indeed. In thermodynamics, heat is just a type of energy flow. If you add energy to an object, some of it may go into doing work that we can measure at a macroscopic level - expansion, for example. Some of it may go into breaking inter-molecular bonds - a change of state. And some of it may go into increasing the vibrational and kinetic energy of each individual molecule. We cannot observe this molecular energy directly, but we can observe it indirectly, because the object then tends to transfer heat energy back to its surroundings in accordance with the second law of thermodynamics. What we call "temperature" is just a measure of the average molecular energy of an object. Gandalf61 (talk) 22:26, 6 May 2008 (UTC)[reply]

My question is still why and how do the molecules vibrate? I know that the energy of heat has to go somewhere, I just don't understand what system causes it to vibrate.Bastard Soap (talk) 17:26, 7 May 2008 (UTC)[reply]

This is one of those "just-because" answers. When a molecule increases in energy, that has to be reflected somehow. The somehow's include increases in translational, rotational, vibrational and excitational modes. Vibration is just one of the ways to store the extra energy. How? Energy is transferred to the molecule and taken up by the particular mode. Why? Because that is the most appropriate mode to accommodate the energy. There are no easy answers when it comes to really tiny things. Franamax (talk) 10:00, 8 May 2008 (UTC)[reply]
If you are asking about the mechanisms that transfer heat energy from the surface of an object or container into the molecules that may lie deep inside it, then the canonical answer involves the three mechanisms of conduction, convection and radiation. Radiation involves energy transfer via electromagnetic waves; conduction involves the transfer of energy via inter-molecular forces; convection involves the macroscopic movement and mixing of a fluid. There are other, less frequently quoted, mechanisms such as thermodynamic phonons in crystals. Gandalf61 (talk) 10:34, 8 May 2008 (UTC)[reply]

May 6

Dino-mite question

What are the 10 newest dinosaurs? --DinoDude08 (talk) 00:33, 6 May 2008 (UTC)[reply]

What do you mean by "newest"? Most recently discovered, or most recently lived? --Tango (talk) 00:36, 6 May 2008 (UTC)[reply]
General Motors, Ford, and Chrysler are starting to look like really good contenders for at least three of the ten positions.
Atlant (talk) 13:14, 6 May 2008 (UTC)[reply]
i think it's safe to say that Dinodude meant newest discovered.... but that can be a tricky question since paleontologists are reclassifying old bones all the time... added to that the debate that sometimes occurs when "new" dinosaurs emerge, which is actually a lot more often than you might think. you just want what the latest news in paleontology is, check out [www.palass.org this], or this. -ΖαππερΝαππερ BabelAlexandria 20:49, 6 May 2008 (UTC)[reply]
Ok. Dino Dude joking aside. Currently birds are believed to have evolved from dinosaurs. So, as our article says "they are the only living dinosaurs". Have a look at the Hoatzin which seems to have regained some dinosaur traits. (Claws on their wings.) Non-bird dinosaurs are usually thought to all have perished in the K-T extinction. So singling out 10 species would be rather arbitrary. There are occasionally bones found above that layer, but scientists think that's because they were moved later. As far as recent discoveries are concerned, it can take a very long time from the moment the bones are dug up, to the point when the bones have been prepared, to the identification of the species and finally the announcement. Look at Xenoposeidon for example. Since it is much more common that only one or two bones remain, rather than an entire specimen, scientists are very careful before they announce a new species. It is not uncommon, though, for bones to get relabeled to belong to a different species than the one originally identified. A quick search resulted in following other candidates for a "top ten" list, but please don't view this as correct and complete:
Paluxysaurus, Eocarcharia dinops, Kryptops palaios, Rugops primus, Velafrons, Nemicolopterus crypticus, Albertaceratops nesmoi, Gigantoraptor erlianensis, Mei (dinosaur), Onychonycteris was a bat and may not qualify
There was mention of a recent find in China, but I could not get at the information. Hope this helps. Lisa4edit71.236.23.111 (talk) 23:19, 6 May 2008 (UTC)[reply]

About NARCOTICS:

About NARCOTICS: I wanted to know can anyone help me find out all the slangs and everything there is to know about illegal narcotics from herb to cocaine, soft to hard, numbers, everything?? —Preceding unsigned comment added by KTTK2008 (talkcontribs) 00:39, 6 May 2008 (UTC)[reply]

What you want is www.erowid.org. Trust me, they have everything on every drug you can imagine, from caffeine to LSD. Chemistry, law, effects, even experience reports. Have (safe) fun. --Shaggorama (talk) 09:56, 8 May 2008 (UTC)[reply]

Genetic code

Do I understand correctly that if I have genes, which represent a high probability of my contracting a certain ailment that an insurance company can not use that information to deny insurance to me or as the basis for increasing my insurance premium? --Schaum 01:29, 6 May 2008 (UTC)

This is bordering on legal advice, which we are not really qualified to dispense. I suggest asking your/a lawyer. Wisdom89 (T / C) 01:30, 6 May 2008 (UTC)[reply]

Nonsense. --Schaum 10:52, 6 May 2008 (UTC)

In most places, the insurance company can use such information freely to jack up your rates. It's only in the very rare jurisdiction where a law has been passed to prevent this that they have to think twice about doing it. - Nunh-huh 01:41, 6 May 2008 (UTC)[reply]

If you're in the US, see Genetic Information Nondiscrimination Act. However, be aware that (1) that link is only to a Wikipedia article, which could be misleading, (2) the act has not yet been signed into law, and (3) we don't yet know how effective it will really be in meeting its intended purposes. --Anonymous, 04:44 UTC, May 6, 2008.

When I first learned about actuary tables I recall that insurance companies included anything they could correlate. In fact I learned that is what made insurance possible. Consequently I do not see them doing otherwise under any circumstance short of pulling their license to do business which would require a law that I am sure they will be sure is never passed. --Schaum 10:52, 6 May 2008 (UTC)
I agree. I also hope, though, that within the next few years we'll have (in the United States, anyway) turned health coverage into a public service (like police and fire coverage) and so these sorts of things will be moot concerns. I understand that those in a for-profit business need to make money, but I think it's stupid to have healthcare be a for-profit business in the first place. --98.217.8.46 (talk) 12:26, 6 May 2008 (UTC)[reply]
98.217.8.46 i'd read a bit more about public-provided health insurance schemes before deciding it is foolish to have for-profit provisions of healthcare. Most of the developed world have for-profit provisions of water, energy supplies and food-stuffs (all essential to supporting life) without major concerns. Drug research is also for-profit and is potentially the reason for such innovation (though patent/copyright law also potentially cause slow down of development). I recommend the chapter on healthcare from the book 'The Undercover Economist' - it mentions a very interesting health-system. I forget which country it is, but i think it is Sri Lanka or somewhere in the orient which had one that, for me, made the most sense (a somewhat hybrid of government, individual and insurance provision) —Preceding unsigned comment added by 194.221.133.226 (talk) 14:38, 6 May 2008 (UTC)[reply]
I've read into it—I think more people benefit from easy, affordable basic care than do, on the whole, from all of the supposed research and drug production that the money is paying for (and I think the costs of the privatized healthcare are far too high, both economically and from a human point of view). (And the economics of water, energy, and foodstuffs are highly regulated in most countries, though they have a superficial appearance of a free market on the client side of things.) Anyway, that's a different question, and a debate at that. We can stop this here. --98.217.8.46 (talk) 03:27, 7 May 2008 (UTC)[reply]

Washingtonion Palm trees

I have read the Wikipedia essays on Palm Trees but cannot find an answer to the following question: What is the life expectancy of Washingtonian Fillifera, Washingtonian Robusta and Medjuel date palms? Particulaly in the southern California area. Thanks, WSC —Preceding unsigned comment added by 75.85.203.191 (talk) 02:17, 6 May 2008 (UTC)[reply]

I'm not sure about date palms (Phoenix dactylifera), but Washingtonia filifera (California Fan Palms) and Washingtonia robusta (Mexican Fan Palms) can easily get to be 100 years old. There are numerous plantings of these palms in southern and central California that date from the early 20th century. I suspect that they can live much longer than that if in the right environment. W. filifera is, of course, native to the desert in southern California.--Eriastrum (talk) 15:37, 6 May 2008 (UTC)[reply]

Alaskan Mosquitos

How do Alaskan Mosquitos survive the winter? Do they fly south before winter? Do they hibenate? 203.94.145.126 (talk) 04:42, 6 May 2008 (UTC)[reply]

Mostly the mosquito eggs laid at the end of summer just wait to develop until it gets warm again. Dragons flight (talk) 04:57, 6 May 2008 (UTC)[reply]
They carry off bears to feed on during the winter.
Seriously, Dragons flight has it right. The eggs survive the winter and hatch again in spring. The adults simply die off. Which is a good thing. Those suckers are huge. There's a reason Alaskans call them the "state bird." -- Kesh (talk) 20:57, 6 May 2008 (UTC)[reply]

plastic containers

Before throwing a large number of various plastic food product containers in the trash where can I find information about personal methods of recycling such as using soda bottles as seedling starters, etc.? --Schaum 06:23, 6 May 2008 (UTC)

Okay then, where best to ask this green question, if not the Wikipedia reference desk? 71.100.14.205 (talk) 01:12, 7 May 2008 (UTC)[reply]
Most councils or government agencies will offer recycling tips. Check out these, for starters: [27], [28] [29], [30] and so on (try googling). http://www.recyclethis.co.uk/ looks good, at first glance. But remember that not all plastics are suitable for reuse. Gwinva (talk) 01:27, 7 May 2008 (UTC)[reply]

IMAX type 3D Camera doing time lapse photography!!??

OK, did I think of this first? Beautiful films of plants growing in time lapse AND 3D!!

Know what I'd really like to see? In Australia, after one of our bushfires, the whole scene looks like a blackened hell hole. Then in the space of several months, it regenerates. Oz bush is DESIGNED to burn and regrow. And no one has EVER filmed it in time lapse. It would look so coooool. National Geographic, where are you? And I would like a credit on your film. And some money. Myles325a (talk) 08:06, 6 May 2008 (UTC)[reply]

How do you know no one has filmed it with time lapse photography before? Also you don't a special camera special camera to do 3D photography just 2 identical cameras set at a fixed distance apart, see Stereoscopy Nil Einne (talk) 15:41, 6 May 2008 (UTC)[reply]

The size of the atom according to the ancient atomists

I've posed this question here as well, from WP:RD/H, in case anyone can give further scientific insight. PeterSymonds | talk 10:26, 6 May 2008 (UTC)[reply]

From this article, [31] I got this quote:

Some controversy surrounds the properties of the atoms. They vary in size: one report—which some scholars question—suggests that atoms could, in principle, be as large as a cosmos, although at least in this cosmos they all seem to be too small to perceive

Can anyone tell me more? Who was it who suggested atoms could be so huge? Were they suggesting another universe populated with a single gigantic atom?

Thanks Adambrowne666 (talk) 10:23, 6 May 2008 (UTC)[reply]

I can't find that quote in that article. Without context, it's difficult to work out what it means. It seems to be a philosophical article, which would suggest it doesn't really mean much at all. --Tango (talk) 13:35, 6 May 2008 (UTC)[reply]


Sorry, Tango, you're right; it was the wrong link - I've fixed it now.Adambrowne666 (talk) 21:18, 6 May 2008 (UTC)[reply]
Ok, even with context I have no idea what it's talking about. It does provide a reference, though - see if you can find a copy of "Diels, H. and W. Kranz, 1951, Die Fragmente der Vorsokratiker, 6th ed. (Berlin)." and find "68A47" in it (I'm not sure what that means, chapter and page, or something, maybe). Of course, it looks like it's in German, so if you don't speak German you will need to find a translation... --Tango (talk) 23:47, 6 May 2008 (UTC)[reply]
This sounds remarkably similar to a theory i've heard of which says the universe is contained within a quark? Regards, CycloneNimrodTalk? 17:10, 6 May 2008 (UTC)[reply]
Keep this in context. Thomas Kuhn showed that often the meaning of a scientific term changes after a paradigm shift in a way not perceived by the proponents of the new paradigm. Our notion of the atom is almost certainly incomensurable with what the ancient greeks meant; atomic theory as we know it today didn't really start to develop until the time of the alchemists, and what we believe now has been so drastically affected by quantum mechanics it would be incomprehensible that if you and democritus (or even if you and an alchemist) had a conversation about atoms that the two of you would be talking about the same thing. --Shaggorama (talk) 10:05, 8 May 2008 (UTC)[reply]

Why the contraction of pronator quadratus produces pronation, and not supination? Think of it, unlike pronator teres, it is not attached to a more proximal fixed point like the medial border of the humerus. This muscle runs absolutely perpendicular to the radius and the ulna.

I have thought about it, and feel that when the forearm is supine, there's no further margin for supination, so the muscle acts to bring the radius closer to the ulna, and not vice versa. And, when the forearm is pronated fully, the muscle must be getting so much relaxed (consequently, also shortened) that its contraction cannot further shorten the fibers, hence no movement occurs in this position.

Well, this is just a guess. It'd be nice if someone approves of it, or corrects my ideas. Regards.

PS: I have posted the same doubt on the talk page of the above article. Didn't get any response. Any way, it's not a "high profile" muscle ;). So, that's understandable.

Ketan Panchal, MBBS (talk) 17:48, 6 May 2008 (UTC)[reply]

Pronator quadratus runs from the lateral (outside) of the radius to the medial border of the ulna, and is found in the anterior compartment of the forearm. Thus when the arm is in a supine (palm up) position, contraction of pronator quadratus acts to roll the outside of the radius toward the ulna, which is pronation. It may help to visualize this by placing your right forearm palm up in front of you and then pinching your ulna and radius with the index finger (on the radius) and thumb (on the ulna) of your left hand. The pinching will simulate contraction of pronator quadratus, and will show the action of the muscle. -- Flyguy649 talk 06:01, 7 May 2008 (UTC)[reply]

The Law of Gravity

I was in science class yesterday doing a chapter on gravity. I asked the teacher whether or not gravity was a law and my teacher said that gravity was only a theory. This sparked a classwide debate, which resulted in my getting a detention for "disrupting class." My teacher will refuse to explain why gravity is a "theory." I think that gravity is a quite obviously a law. I need to know: am I correct? Could someone help me? Thanks. 31306D696E6E69636B6D (talk) 18:01, 6 May 2008 (UTC)[reply]

Newton's law of universal gravitation can be proved mathematically and is an accpeted physical law See responses below. However most of science is just accepted theory based on "what works". If you look through the history of science, theories and laws have constantly been debated and adapted using both mathematical and experimental evidence. Sorry about the detention. I really don't see how sparking a debate can be a valid reason for a detention unless it was during a test or something. Tbo 157(talk) 18:11, 6 May 2008 (UTC)[reply]
What do you mean when you say it "can be proved mathematically"? It's not even a correct model of reality—hence the formulation of general relativity. In response to the question, the distinction between a scientific law and a scientific theory is essentially meaningless, and often based on historical use. We talk about the "law of universal gravitation" and the "theory of relativity", even though the latter is certainly more correct than the former. If your teacher really said "only a theory", I would criticize that use—"theory" is quite a strong word in science, and is only used when the evidence is very strong. It doesn't mean "guess" or "hunch" like it might in everyday use. -- Coneslayer (talk) 18:22, 6 May 2008 (UTC)[reply]
  • (edit conflict) Indeed, the first response is wrong. Not only is Newton's law of universal gravitation not proven, it is, on the contrary, disproven and has been replaced by Einstein's General Theory of Relativity, which is a better description of reality, but also quite likely wrong. Strict proof is only possible in mathematics. In science, a theory is a self-consistent explanation of a set of observations that allows testable predictions. A "law" is a concise statement of (usually cental parts of) a theory that has been very successful in making correct predictions. Newton's theory, while certainly wrong, is a good enough approximation to be still very useful, so we still talk about the "Newton's law of gravity". In fact, a good theory is the best you can hope for in science. And, while I'm nitpicking anyways: Gravity is neither a law nor a theory, but rather a force ;-) --Stephan Schulz (talk) 18:31, 6 May 2008 (UTC)[reply]
Well, Newton did prove the law of gravitation mathematically - but only after taking Kepler's laws of cosmic motion and his own law's of motion as postulates. The great step forward Newton made was to show that both terrestial and celestial motion could be described by a common set of laws. All scientific theories have to start with something, ad what you can prove mathematically depends on what the "givens" are that you are assumming. We constantly modify the "givens" in the light of scientific experiments. We constantly try and reduce the number of givens we have to assume but we will never eliminate them entirely, there will always be at least one law that must be accepted without proof and from which everything else is derived. SpinningSpark 18:38, 6 May 2008 (UTC)[reply]
There's a common misunderstanding about what the word "theory" means. A theory is stronger than a law. A law is what you get when you do lots of experiments, plot them on a graph and draw a trendline - the formula for the trendline is the law. A theory is something which explains why that law holds, and allows you to predict more laws. "Only a theory" is nonsense, since a theory is the strongest thing you have in science. --Tango (talk) 18:29, 6 May 2008 (UTC)[reply]
You are drawing a narrow distinction between "law" and "theory" that is not well reflected in the way that the terms are used in practice. See physical law for discussion of characteristics often associated with "laws". Ultimately though it is a semantic distinction that is much more important to historians of science than it is actually to scientists. Dragons flight (talk) 18:37, 6 May 2008 (UTC)[reply]
To quote that very article: "Simply stated, while a law notes that something happens, a theory explains why and how something happens." I think that's pretty much what I said. --Tango (talk) 20:36, 6 May 2008 (UTC)[reply]
My concern, which may or may not have been what you intended, is that your original statement describes laws very emprically and suggests (at least in the way I read it) that laws describe relationships where the explanation isn't understood. In practice though, the expression of most laws isn't merely empirical and is motivated by some underlying theoretical understanding. Hence the line between laws and theory is inherent fuzzy. Maybe I reacted to your original statement differently than you intended. Dragons flight (talk) 16:26, 7 May 2008 (UTC)[reply]
Yes, I was a little unclear. Laws are strongly supported by empirical evidence. They are generally explained by theory, but don't need to me (I believe Kepler's Laws were purely empirical for a while before Newton showed they could be derived from his Universal Law of Gravity, for example). Theory, on the other hand, is mainly explanation and isn't particular supported by evidence directly - theories predict laws, which are then supported by evidence. --Tango (talk) 16:33, 7 May 2008 (UTC)[reply]
What if gravitomagnetism is proven right? What if antigravity turns out to be possible? What if antimatter turns out to be repelled instead of attracted by matter? Em3ryguy (talk) 20:16, 6 May 2008 (UTC)[reply]
Then we'll try and come up with a new theory that fits the new observations. That's how science works. --Tango (talk) 20:36, 6 May 2008 (UTC)[reply]
Wasnt that my point? its a theory. a law would be a rule that is not and cannot ever be broken. conservation laws, presumably, cannot be broken. Em3ryguy (talk) 20:58, 6 May 2008 (UTC)[reply]
Actually, conservation laws can be broken, albeit for an extremly short amount of time. I believe it's the law of conservation of matter. I don't know of an article about it, but on the science channel there was something that said particles can pop into existence but pop back out in such a short time that there is no effect. It had something to do with the Heisenburg Uncertainty principle allowing it.Zrs 12 (talk) 01:52, 7 May 2008 (UTC)[reply]
That's a fundamental misunderstanding of Scientific law and Scientific theory. Law is simply a mathematical proof, while a theory is a broad description of what happens.
That said, I'm confident gravitomagentism will never be proven. Gravity and electro-magnetism are polar opposites (pardon the pun). Magnetism is extremely powerful at close distances, but rapidly loses effectiveness once the distances increase. Gravity retains its strength over long distances, but is overpowered by magnetism on the small-scale. They work very differently. -- Kesh (talk) 21:04, 6 May 2008 (UTC)[reply]
You're confused in your example. Gravitomagnetism is a thus far impossible to measure property of general relativity that is unrelated to electromagnetism except that the equations have a similar structure. Dragons flight (talk) 21:09, 6 May 2008 (UTC)[reply]
Ahh, thank you for the explanation. The only times I've heard "gravitomagnetism" it was from self-styled Internet Einsteins who thought they had solved every problem in the universe by simply replacing gravity with electromagnetism. The math was horrid, and the logic was worse. -- Kesh (talk) 21:10, 7 May 2008 (UTC)[reply]
It is possible for laws to be broken. There's no way for us to know that V will always equal IR (Ohm's law) - it always has up until now, and everything suggests that it always will, but there is still a non-zero chance that it's just been a big coincidence. A law is *not* an absolute statement - that's a "theorem" and is a purely mathematical thing. You might have a theorem that says "given this theory, such and such will happen", for example, given Newton's Universal Law of Gravity, we can mathematically prove the shell theorem, but we still can't be sure that the result of that theorem will always hold, since we can't be sure the law will always hold. --Tango (talk) 12:39, 7 May 2008 (UTC)[reply]
Try this, which works in my world.
  • In the humanities etc, "theory" is synonymous with "hypothesis".
  • In the sciences, "theory" is synonymous with "model", and specifically, a model that explains all the existing observations.
Succinct. --Danh, 70.59.79.108 (talk) 23:02, 6 May 2008 (UTC)[reply]
Close enough. This is also why it's hard to explain to some people that just because a scientist calls something a theory doesn't meant that it's random conjecture. Proper scientists don't speak in absolute terms, but that doesn't mean they lake confidence, that just means they're real scientists. 206.126.163.20 (talk) 00:23, 7 May 2008 (UTC)[reply]
The way I would state it is, both laws and theories are build on axioms (things that must simply be assumed true, probably based on experience and observation). A law can be quite simple (energy is conserved) or it can be more complex (entropy always increasing in time, the definition of entropy being pretty non-trivial--at least quite a bit more than for energy). A theory takes a set of axioms and builds something somewhat complex on top of it; generally a large set of rules and conclusions. Both must be logically consistent in order to have a chance at being useful. We would generally call something a "law" or remove its status as mere "theory" once its axioms have been tested/experienced enough that we are pretty assured of their truth. If a law or theory is completely sound logically, testing the axioms may be done at least in part by testing the law/axiom's conclusions. That's just my take on it though. Think about it and see if it makes sense for you. --Prestidigitator (talk) 00:07, 7 May 2008 (UTC)[reply]
Consider this: Not only has Newton's law of gravity been proven to be but an approximation, but some physicists doubt that Einstein's theory of relativity is complete. Imagine Reason (talk) 20:04, 7 May 2008 (UTC)[reply]
People know Relativity isn't complete since experiments on very small scales deviate significantly from it. --Tango (talk) 22:21, 7 May 2008 (UTC)[reply]

Infinity

I'm not sure if this belongs here or the mathematics desk or possibly even the philosophy desk, but i'll ask it anyway. Is infinity accepted as being real or is it simply a state which we haven't reached? If that makes sense. I.E. If you divide 100 by 3 you get 33.3 with the .3 recurring. Does this go to infinity or is there a limit?

Regards, CycloneNimrodTalk? 19:01, 6 May 2008 (UTC)[reply]

Oh, I don't just mean it as a mathematical context either, anything where infinity is said to exist e.g. at the event horizon of a black hole? Regards, CycloneNimrodTalk? 19:03, 6 May 2008 (UTC)[reply]
A Repeating decimal is infinite, yes. Other than that, I'm not sure I understand the question. Friday (talk) 19:05, 6 May 2008 (UTC)[reply]
It's a mental and mathematical abstraction. Wisdom89 (T / C) 20:19, 6 May 2008 (UTC)[reply]
this is prolly better taken up with the philosophers, because i'm sure someone will come up with something extremely complicated and technical.... but i believe that science and math and people in general accept infinity as being real... it's the basis of our concepts like dimensional space and counting. -ΖαππερΝαππερ BabelAlexandria 21:04, 6 May 2008 (UTC)[reply]
I'll give an example. Time. I think most people agree that there must have been a start to time, or do we keep on getting infinitely closer to seeing the beginning? Regards, CycloneNimrodTalk? 21:14, 6 May 2008 (UTC)[reply]
Time is a convenient concept created by human beings in order to get up the next morning for work, or know not to go to work on Sunday. It has little to do with infinity. There are entire mathematical branches that deal with infinity starting with calculus ("approaching") and complex numbers (z), but that is scientific. It appears you first questioned maths, then ended off on a more philosophical note. Perhaps infinity was given to us by God, denoting that which cannot be obtained by mere mortals. I can go on. The bounds of one's imagination are infinite. Sandman30s (talk) 21:35, 6 May 2008 (UTC)[reply]
Infinity, or its many variants, is a mathematical construct. In the example of 33.333..., or more interestingly 0.999..., the "infinite number" of decimals places merely means that "for any decimal place you find, there is one after it". In the physical sciences, the appearance of infinity - or a singularity - as the solution to an equation generally translates as "things break down here". For example, the event horizon of a black hole is a "removable singularity", meaning that it's weird, but after a change of coordinates it disappears. The centre of a black hole, however, is an "essential singularity", which translates as "general relativity can't tell you what happens at the centre". Confusing Manifestation(Say hi!) 23:19, 6 May 2008 (UTC)[reply]
A fun thought experiment that relates to infinity and time (and space) is Xeno's_paradox#Achilles_and_the_tortoise. -- JSBillings 13:28, 7 May 2008 (UTC)[reply]

What does "qsp" in reference to gelatin or perhaps other polymers mean?

I found a reference of "qsp" to gelatin for which I couldn't find a definition on Google. Based on what I did see on Google, it seems to be a unit of measurement for gelatin and perhaps polymers. In the case I refer to, it said "gelatin qsp 100%" Can anyone help define/explain this further?

Thanks! —Preceding unsigned comment added by 65.209.203.254 (talk) 19:11, 6 May 2008 (UTC)[reply]

I web searched on "gelatin" together with "qsp" and eventually found some examples where qsp seemed to mean specific viscosity. However, on looking this up, I find that the symbol for it actually uses the Greek letter eta (looks like n with the bottom right corner extended), not the English letter q. In a Google search on both "specific viscosity" and "qsp", many of the results are papers that cannot be freely downloaded, but some hits -- for example this paper in PDF, the 10th hit when I did the search -- show qsp in Google's search synopsis while the actual document has an eta and sp.
So I'm guessing that qsp is being used to mean specific viscosity and the q is a mistake for eta, perhaps originating due to a character-set problem, or else an alternate symbol used if Greek letters are not available.
Either that or I'm completely wrong.
I see that Wikipedia once had a WikiProject Polymers that intended to produce an article on specific viscosity, but the project appears to be moribund. --Anonymous, 20:03 UTC, May 6, 2008.
I think that this probably refers to the reaction quotient for solubility, Qsp. See also Solubility equilibrium. Rmhermen (talk) 16:03, 7 May 2008 (UTC)[reply]

If you saw this on a food, drug or cosmetic, it might have actually been USP for United States Pharmacopeia which would indicate the gelatin met standards set by USP. ike9898 (talk) 21:48, 7 May 2008 (UTC)[reply]

Swinging

Why does swinging one's legs when on a swing help? —Preceding unsigned comment added by 76.69.240.138 (talk) 22:05, 6 May 2008 (UTC)[reply]

Well, I expect it's about moving your centre of mass, which imparts energy to the swinging. Your weight can be modelled as a point force acting through your centre of mass, and the swing itself as an inextensible string. At rest, the swing hangs straight down, through your centre of mass, so the forces balance out. If you move your centre of mass, you generate a turning moment, which moves the swing as it tries to find equilibrium again. -mattbuck (Talk) 23:53, 6 May 2008 (UTC)[reply]
Scientific American had an article in their Amateur Scientist section in March 1989 (unfortunately they don't have that accessible online) The interesting thing was the discussions that article started. Our swing article has a pdf link at the bottom. This site has video that might help answer your question. [32] Lisa4edit (talk) 03:11, 7 May 2008 (UTC)[reply]

Bicycles

Why is it easier to balance a bicycle when it is moving quickly. I asked someone this, and he said that it was because gravity was causing the bike to go in centripetal motion, but I don't think that makes much sense because it should only account for the bike turning. I've tried looking for the answer online, but I haven't been able to find consistent answers. Thanks. —Preceding unsigned comment added by 76.69.240.138 (talk) 23:09, 6 May 2008 (UTC)[reply]

Don't nail me to this, but I think it has to do with the force vectors. While you are stationary any forces to the left or right will take full effect. While you are moving forward the main momentum/force is in that direction and the left and right tilts will have less effect. 71.236.23.111 (talk) 23:34, 6 May 2008 (UTC)[reply]
Wikipedia has an article on everything....Bicycle and motorcycle dynamics--Shniken1 (talk) 23:43, 6 May 2008 (UTC)[reply]
Conservation of angular momentum. It's the spinning mass of the tires that help keep you upright. If you add weight to the tires and go at the same speed, it will be even harder to tip over. Here's a video.--Duk 03:30, 7 May 2008 (UTC)[reply]
Took the liberty of fixing your broken link. —Keenan Pepper 05:24, 7 May 2008 (UTC)[reply]
Actually, the effect of angular momentum is very minor. It has the role of a steering force, but is not a significant righting force. Our article, mentioned above, discusses all the effects in much detail. --Stephan Schulz (talk) 05:35, 7 May 2008 (UTC)[reply]
The effect of angular momentum depends on speed (squared). At slow speeds it doesn't do much, at fast speeds and with heavy tires it does a lot. Riding a large motorcycle is a good example, you can practically hang off to one side and the bike will not tip. --Duk 14:53, 8 May 2008 (UTC)[reply]
When you are moving forward on the bicycle, and begin to tilt to one side you use the handlebars to stop the fall by turning the bicycle that way. When you are stationary it does not help! Graeme Bartlett (talk) 22:04, 7 May 2008 (UTC)[reply]
I think that's mostly true for low speeds, but once you get going fast enough you don't even need to touch the handlebars. --Duk 03:01, 8 May 2008 (UTC)[reply]
But that's still not due to the gyroscope effect. You may be right for motorcycles -- their wheels and tires may have enough moment of inertia, and be rotating fast enough, to gyro-stabilize the bike. But for bicycles it just doesn't contribute much. The tires are intentionally light (because you have to spin them up with your own muscles -- you wouldn't want to do that to a motorcycle tire). Rather, the self-righting effect comes from the geometry of the bike, the way the force from the road tends to act on your front wheel to move it back to the center. --Trovatore (talk) 19:50, 8 May 2008 (UTC)[reply]

Anatomy and physiology

Describe the role of the thymus gland and the Gut-associated tissue (GALT) in the establishment of active immunity —Preceding unsigned comment added by ELVIOTA (talkcontribs) 23:43, 6 May 2008 (UTC)[reply]

This looks like a homework question, as as it says at the top of the page, we will not do your homework for you. If you have a specific section you're having trouble with, and you have made an effort to do it youself, THEN we will help. -mattbuck (Talk) 23:50, 6 May 2008 (UTC)[reply]
Have you read our article on Thymus - that would certainly be the place to start. --Tango (talk) 23:50, 6 May 2008 (UTC)[reply]
For the other part look at Gastrointestinal tract and follow the links there.71.236.23.111 (talk) 01:58, 7 May 2008 (UTC)[reply]


May 7

degree of benefit

Where might I obtain the data to determine if the installation of Internet capable water meters might reduce fuel consumption by various utility suppliers? 71.100.14.205 (talk) 01:08, 7 May 2008 (UTC)[reply]

You could try asking the suppliers. Otherwise, try googling for some study into them. --Tango (talk) 12:41, 7 May 2008 (UTC)[reply]

Infinity of space

Today my Physics teacher was discussing how the universe can go on forever or it can be like a snowglobe, and no one can know what's outside the globe. I'm extremely intrigued by the unknown factors in this and was wondering if someone could point me in the right direction to go read more into this (typing in "past what we think is the universe" didn't get me anywhere haha). Thank you! Evaunit♥666♥ 02:09, 7 May 2008 (UTC)[reply]

We've got an article on Shape of the universe, but it gets a bit technical. There's also the issue of the observable universe versus the "whole" universe. The problem about talking about things "outside the universe" or "before the beginning of the universe" is that all of our physics is based on observations inside the universe, so it's hard to say that they apply to things outside that. Confusing Manifestation(Say hi!) 03:22, 7 May 2008 (UTC)[reply]
I think your teacher is talking about the Ultimate fate of the universe and whether the universe will continue to expand such that the energy density approaches zero (infinite universe i.e. Heat death of the universe), or will it contract such that the energy density approaches infinity (reaches a maximum "snow globe" i.e. Big Crunch). Information cannot travel faster than light so the snowglobe is the size of light traveling since the big bang and there should always be a boundary, even in the collapsing model. --DHeyward (talk) 06:04, 7 May 2008 (UTC)[reply]
Data in the past decade seem to be pointing to a hyperbolic, infinite universe that does not contain enough mass to stop expansion. Imagine Reason (talk) 20:00, 7 May 2008 (UTC)[reply]
One weird idea that alot of people seem to have trouble with is that although the universe may have a contained shape, that doesn't mean that anything lies outside the shape. When physicicsts talk about "the universe," they are attempting to describe everything that exists. If the place where everything exists is in some dimension spherical (which it's probably not, by the way), then nothing lies outside of that sphere because the sphere just is everything that exists. Contemporary scientific theory has raised the possibilty of multiple universes, so maybe this should be qualified as "everything that exists for us" or as Confusing Manifestation pointed out "the observable universe." --Shaggorama (talk) 20:09, 8 May 2008 (UTC)[reply]

chemical, mineral, vitamin make-up of menstrual blood?

I have read that menstrual blood contains all the contents of regular blood, such as plasma, but doesn't it also contain vitamins and minerals and growth hormones? Also I just read of an English firm that is recruiting women to submit their menstrual blood because it contains stem cells. A landscaper friend of mine regularly uses birth control pills to make all his bedding plants look spiffy, which implies that mammalian hormones stimulate plants- and perhaps plant hormones stimulate people?- (Spring fever and all that) Really, I am just looking for a layman's list of the contents of menstrual blood.People777 (talk) 05:34, 7 May 2008 (UTC)[reply]

Menstrual "blood" is a misnomer: what is shed during menstruation is the lining of the endometrium. "Menstrual fluid" is closer to correct. There's blood mixed in there, but it's not the main constituent. I don't know where you'd find an analysis of it, but I'm sure it contains as many vitamins or minerals as you'd find in any body tissue. It certainly contains endometrial cells, cervical mucus and vaginal secretions. As to the birth control pills: plant hormones and human ones are similar; plant sterols and estrogen are synthesized by similar pathways. - Nunh-huh 07:14, 7 May 2008 (UTC) (actually, there are papers, but I don't know that there's online access: e.g. Am J Obstet Gynecol. 1965 May 15;92:183-8 14281826 (P,S,E,B) THE CONTENTS OF MENSTRUAL FLUID: AN ANALYSIS OF 260 SAMPLES FROM HUMAN FEMALES. M S BURNHILL, C H BIRNBERG[reply]
Have you looked at our Menstruation article? Blood as the body's transport medium contains many substances including vitamins. Phytoestrogens are plant substances that mimic estrogen. 71.236.23.111 (talk) 07:37, 7 May 2008 (UTC)[reply]

A new Vector Energy Interpretation of Quantum Mechanics - what to do?

This derives from a very thorough paper (external) that examines relativity if time were treated as a vector. To the best of my ability, the derivations look valid. Combined with the transactional interpretation one can derive an "offshoot" interpretation that treats a negative energy vector as a photon traveling backwards in time. The interpretation's main premise is that these photons are absorbed by empty space and since energy is added, must be conserved as in pair production (other things conserved too); or the tunnel effect being a particle in an energy well converting all mass/energy into a negative vector energy photon emission and disappears. Outside the energy barrier empty space absorbs a negative vector photon and the particle appears there.


The problem is that this can be used in several ways, all IMHO have merit.

One of these appears to be absurd at first; it is much deeper, though not many I feel will see the deeper significance. So I don't know what to do?

any help appreciated

reference user:fx303 page for the 'absurd' view. —Preceding unsigned comment added by 67.174.224.78 (talk) 06:07, 7 May 2008 (UTC)[reply]

Hello. Can you supply us with anything at all to read on the subject, or at least point to where/under what name this paper has been publicised? Scaller (talk) 14:42, 7 May 2008 (UTC)[reply]
I think the OP was referring to User:Fx303#Vector_Energy_Interpretation_of_Quantum_Mechanics. -- JSBillings 01:01, 8 May 2008 (UTC)[reply]

Amount of zinc gluconate in Zicam

I understand the formula is propietary, and I have been looking for the patent, but I was wondering how much zinc is actually in it in mg. This is due to problems of zinc destroying nasal tissue, although I'm not sure of the mechanism.

Thanks, JD139.225.81.128 (talk) 14:57, 7 May 2008 (UTC)[reply]

Could time be expanding?

Hi. Have physicists considered that time could be expanding like space is? Or does the standing definition of 'time' and the physics around it somehow make this impossible? What if time were expanding (somehow), but we just haven't noticed it (either in real life or theoretically)? I wish I could give more 'punch' to this possibility, but I'm no physics expert... Thanks in advance, Kreachure (talk) 16:44, 7 May 2008 (UTC)[reply]

I'm not an expert either but i'm pretty sure time is part of the whole space idea. If space expands, time should expand proportionally and it'd make no difference? Probably wrong, but hey, worth a shot! Regards, CycloneNimrodTalk? 16:59, 7 May 2008 (UTC)[reply]
We mostly define distance in terms of the time it takes light to travel over that distance. I suspect you could describe the expansion of the universe as mathematically equivalent to a static universe with a changing time dimension. As we live in the universe, if the two descriptions are equivalent (as I suspect) then it is probably easier for us mortals to assume the nature of time is not changing. Dragons flight (talk) 17:09, 7 May 2008 (UTC)[reply]
The big bang cosmological model doesn't contain anything that I'd describe as "expanding time". The concept doesn't make much sense because expansion is something that happens over time. When you say that something is expanding you mean it's smaller at an earlier time and larger at a later time. You could probably come up with some sense in which "time is expanding"—for example, the time you've been alive is "expanding" at a rate of one second per second—but probably nothing very interesting or meaningful. -- BenRG (talk) 17:28, 7 May 2008 (UTC)[reply]
Well, let's pull an Einstein and think about what time is: a measurement of how long it takes for some regular interval to happen. Let's say we had a light clock with a 1 second interval—that is, a photon bouncing between two mirrors and it takes it one second to get from one mirror to the other. Now let's say the space between the mirrors expands with space so it takes longer than 1 second for the photon to go over the space: do we redefine the second, or do we say that that particular clock no longer measures what it used to measure? I would think the latter. But I think the rub is that if all space was expanding evenly, you'd have no way of knowing, though: how would you know your clock no longer worked the same way it used to, without another clock to compare it to? If the same expansion of space affected that clock too, in exactly the same way, then in effect there'd be no way to determine the difference. If, however, the expansion was detectable in one way or another (which our current expansion is, because it affects different scales more than others), then it would be easy to say that the clock had just gotten inaccurate. Someone correct me if I'm reasoning totally wrong here. --98.217.8.46 (talk) 18:23, 7 May 2008 (UTC)[reply]
First you have to define time. In an attempt to be overly general (so don't respond with quotes out of your physics book to clarify the specifics), there are two views on time. One view is that time is a dimension (like length, width, and height) that contains events. Objects travel through the time dimension and measure it by the time distance between events. Another view is that time is not a dimension, nothing travels through it, and it does not contain events. It is merely a value placed by humans to make formulas work properly. If you use the first definition, it is possible to claim that the time dimension is expanding since it has the same properties as the other three dimensions. If you use the second definition, time cannot expand because it doesn't exist. -- kainaw 18:45, 7 May 2008 (UTC)[reply]
The definition of the term "expanding" breaks the symmetry between time and space. Even if time can somehow expand, it can't expand in the same way that space does. It's like asking whether space expands with distance. There's a weird circularity in the idea. You can think of time as an implicit dimension that parametrizes change, or you can think of it as an explicit dimension like space, but you can't have both at once. In the spacetime picture there's no implicit change. Objects don't move along their worldlines, they simply are their worldlines. Changing the past isn't a physical impossibility, it's a logical contradiction. (Actually that's true of either notion of time.) If the original question was whether the time dimension can expand over the course of some other (implicit) time interval, then the answer is no, that doesn't make sense. If the question was whether it can expand over the course of the same time that it itself measures, then I think the answer is still that it doesn't make sense. -- BenRG (talk) 20:12, 7 May 2008 (UTC)[reply]
To expand on my comment above. You generally describe the structure of the universe as a whole through the FLRW metric of spacetime:
Where the scale factor a(t) describe the expansion of space as a function of time. However, I am fairly confident that one could construct an equivalent metric:
where b(t) is a new scale factor, and yet I suspect that if you apply the Einstein equations and follow the construction through it could be designed to give rise to the same dynamical structure. The second metric would be described as having static spatial dimensions and a dynamic time dimension, but from the point of view of any quantities observable by people living in the universe they would give exactly the same behavior. Dragons flight (talk) 20:32, 7 May 2008 (UTC)[reply]
P.S. I also think it is possible to construct a system with both dynamic time and dynamic space, but I think there no physically useful reason for doing so as one can always condense all of the variation into one or the other. Dragons flight (talk) 20:39, 7 May 2008 (UTC)[reply]
More importantly, if time was expanding in this way (which I think it probably is), do you think we would be capable of noticing it? in relative terms, the amount of time we experience probably would stay the same, for the same reason we don't notice out bodies expanding as the universe does. relativity is a pretty elegant theory. --Shaggorama (talk) 10:11, 8 May 2008 (UTC)[reply]
Our bodies don't expand as the universe does. The expansion is defined in terms of real physical distances that we can measure; our measuring devices tell us that the universe is expanding at large scales but human bodies aren't. The trouble with the b(t) metric above is that it doesn't describe a measurable expansion the way the FLRW metric does. -- BenRG (talk) 18:05, 8 May 2008 (UTC)[reply]
No, this metric won't work. Make the substitution and it becomes
,
which is the FLRW metric with a(t ') = 1. The observed geometry of the universe requires a non-constant a. Also, b isn't actually a parameter, since you get the same geometry regardless of b. You could try making b a function of r instead of t. I think this would get you a family of geometries distinct from the FLRW geometries (in which case they still don't describe the real world), but I'm not entirely sure. -- BenRG (talk) 18:05, 8 May 2008 (UTC)[reply]

The Doolittle Raid

Why wouldn't the U.S. navy install booster rockets to the B-25s of the Doolittle Raid? Wasn't it cheap and proven technology? -- Toytoy (talk) 17:29, 7 May 2008 (UTC)[reply]

Perhaps a better question is "why would they?" Based on our JATO article, it doesn't appear the US had them at this point, and I doubt that they'd be classified as "cheap and proven" at that stage even if they did. Training for the raid had already established that B-25s should be launchable from a carrier, so why would the Navy and Army add extra unnecessary complexity? — Lomn 19:01, 7 May 2008 (UTC)[reply]
The rocket boosters, had they been a proven and available technology, might have allowed greater takeoff eweight, thereby allowing more fuel load (possibly via aux tanks) with greater range and thus the possibility to launch farther from Japan with greater safety for the carrier and less chance of premature detection, or with a greater bombload, or with a greater chance of making it on to a safe landing in China. Edison (talk) 19:43, 7 May 2008 (UTC)[reply]
As I recall (from the book version of Thirty Seconds Over Tokyo), the B-25s were already doing stuff like carrying jerry cans of fuel internally. While I concur that in principle with the higher weight -> more range -> safer mission principle, I don't know that any practical payoff was likely (bomb load vs range is a different matter, I guess, but meh -- it was a propaganda raid. 5% more bombs doesn't affect that). Regardless, a large part of the range problem was the premature launch of the mission due to possible detection, though my personal opinion from reading about this as well as the Flying Tigers is that the US was overly optimistic to expect these aircraft to successfully rebase out of China. — Lomn 19:59, 7 May 2008 (UTC)[reply]
Retrospective analysis can always find superior solutions. The Doolittle raid was cobbled together from available pieces in a very short time. With the benefit of hindsight, we can find all sorts of clever improvements. In your scenario, JATO units would have increased the bomb payloads, resulting in much more damage to Tokyo. but in an equally likely scenario, one or more JATO units would have failed catastrophically, causing distruction of a B-25 and renderingthe carrier incapable of launching other B-25s. -Arch dude (talk) 02:33, 8 May 2008 (UTC)[reply]

I can hardly think of a reason why they could not modify a military rocket for a mission like this. The army has been using ground-based rockets for many years. This proven technology shall be adapted for launching bombers on an aircraft carrier's deck. It can decrease the risks.

Installing jerrycans on a bomber seems to be ... well ... stupid. These cans are heavy and not very efficient. Why couldn't they make a large aluminium oil tank?

I find the jerrycan article funny. Allied scientists had to reverse engineer steel oil cans? Was it rocket science to make cheap and durable oil cans? Hitler ordered jerrycans secretly? Was it really such a big deal to make some oil cans for people? Couldn't the Germans make up a good excuse for making oil/water cans? e.g., "These cans are for delivery of cooking oil to the hospitals." -- Toytoy (talk) 12:44, 8 May 2008 (UTC)[reply]

Methinks you overestimate the ease of something like this and the likelihood of success. Getting rockets to work reliably was not an easy task; to do it with manned flight and without causing danger to the launching platform was simply not a risk worth taking in 1942. In any case, thinking backwards like this is a poor historical methodology—all you learn about is yourself, not the past.
As for the jerrycan, I think you also underestimate the materials science and good engineering that goes into that particular steel oil can. Think about it this way: if it was easy to do, they wouldn't have needed to reverse engineer it. They weren't any stupider than people today are. --98.217.8.46 (talk) 13:33, 8 May 2008 (UTC)[reply]
What ground-based rockets did the U.S. have in 1942? The German and the Russians deployed things like the Katyusha but these probably didn't have the power to accelerate a bomber significantly. The development of large rockets by the Germans during the war was a major advance. (And so we acquired their technology and personnel after the war.) See Rocket artillery#World War II, V2 rocket. Rmhermen (talk) 14:36, 8 May 2008 (UTC)[reply]
Digging deeply through our poorly organized material on Wikipedia, Aerojet tested a U.S. design for a JATO rocket in August 1941 but it didn't go into production until 1943. Elsewhere I found, that in the 1941 test, it took 12 rockets to launch a 1,260 lb max weight plane. A loaded B-25 weighed 33,510 lb so we only need to attach about 320 rockets to each bomber! In April of 1942 (the same month as Doolittle), the U.S. began testing a liquid-fueled JATO on a bomber smaller than the B-25; however, development was abandoned in 1944. Rmhermen (talk) 14:54, 8 May 2008 (UTC)[reply]

Proving Technology for Aerospace Applications

Hi all, does anyone know what sort of tests are required to prove a new technology or material for use on aeroplanes or spacecraft? And what issues need to be considered? So far I've considered:

  • An ability to be implemented consistently, reliably
  • Withstanding repeated exposure to the pressure and temperature changes likely to be experienced
  • A knowledge of how to detect flaws and failings in the material or technology.

I have also found, British Aerospace EAP an example of testing aerospace technology.

This is part of revision for an exam coming up. If anyone has any thoughts, knowledge or ideas I'd be very grateful. Many thanks, LHMike (talk) 18:43, 7 May 2008 (UTC)[reply]

Technology Readiness Level has some information on American standards Mad031683 (talk) 19:49, 7 May 2008 (UTC)[reply]
Thanks, that's really helpful. Also found Mature technology linked. LHMike (talk) 00:26, 8 May 2008 (UTC)[reply]

biology

Can human polyspermy end as a viable embryo (e.g. by mixoploidy etc.)? Eliko (talk) 19:52, 7 May 2008 (UTC)[reply]

Swings and insurance liability

I've seen in other countries swings on which children stand up. I see only swing seats here in America. Does this have to do with the rate of injuries? I find the seats rather boring. Imagine Reason (talk) 20:08, 7 May 2008 (UTC)[reply]

That doesn't sound like a science question to me... just google it and see if you can find some studies. I doubt anyone here just happens to be an expert on playground accident statistics (although anything is possible...). --Tango (talk) 22:16, 7 May 2008 (UTC)[reply]
You can stand up on any swing with a hard seat. You've never seen a swing with a wooden plank for a seat? --Shaggorama (talk) 10:12, 8 May 2008 (UTC)[reply]
Not in public in America you won't (see any swings with hard seats). The legal threat posed by such a deadly item is just too much! So very uncomfortable, not very swingable "sling seats" are all you'll see.
Atlant (talk) 13:14, 8 May 2008 (UTC)[reply]

Are old-growth forests being cleared faster than they regenerate?

My question is specifically directed towards the forests in British Columbia, where environmentalists (namely Greenpeace) have targeted most. I know that the UN's Forest and Agriculture Organization (FAO) reports that Canada has maintained over 400,000 hectares of forest cover, but how much of that is old-growth? Does that amount of cover provide old-growth forests to regenerate by the time they are logged again? —Akrabbimtalk 22:06, 7 May 2008 (UTC)[reply]

Although the term Old growth forest is somewhat disputed, from what I know it is generally agreed that once it has been cut down it's gone. What you replant is a managed tree lot usually called Secondary forest. Certain features of old growth forest like natural selection of species growth pattern, influence of dead trees and fallen old logs, established drainage patterns and natural territory boundaries are not re-created once the old growth has been logged. UW researchers studied Mount St. Helens after the blast [33] what is not mentioned in this article is, the study plot that was cleared of fallen and dead logs and replanted actually recovered slower than the plots left to their own devices. Some forestry managers now use limited controlled fires to control underbrush, because they found that dousing all fires just made the next fire bigger and more dangerous. Sustainable forestry is no substitute for old growth because it just means that loggers don't take out more trees than they grow. It's like taking out a creek and putting a concrete lined canal there instead. The water still flows but many other factors can not be re-created. On a positive note, some areas of South American rain forest used to be farmed by natives in ancient times and are now almost indistinguishable from undisturbed growth. We just won't leave a forest alone for a couple of centuries for "old growth" to develop again. Hope this will help with answering your question. --71.236.23.111 (talk) 23:52, 7 May 2008 (UTC)[reply]

Question: What is the compound/name of the chelate of calcium produced by plants?

As chlorophyl is a chelate of magnesium produced by plants, I wish to know any chelate of calcium that is produced during the symbiotic relationship of plant roots and soil microbes. Best answered by a SOIL BIOLOGIST.

I have asked several chemists this question and they always say, "EDTA." We are talking about organic chemistry here. Engintinc (talk) 22:50, 7 May 2008 (UTC)[reply]

Well, EDTA certainly will chelate calcium ions, but it's a synthetic molecule, and not really something made by plants. (And if you're interested in calcium specifically, EGTA may be a better bet for a synthetic chelator.) For natural compounds, you're probably looking for a calcium equivalent to siderophore. Unfortunately, I am unaware of any such molecules, although they certainly may exist. -- 128.104.112.85 (talk) 14:37, 8 May 2008 (UTC)[reply]

Conservation of angular momentum

During a physics lab on the conservation of angular momentum, we were asked to sit on a chair could rotate effectively frictionlessly. After firmly placing our feet on the ground, someone spun the wheel, and then we were asked to flip the wheel. Now, as per the conservation of angular momentum, it is expected that we would spin in the opposite direction, and that is what we observed. However, what I don`t understand is what causes the person in the chair to spin. To illustrate my point, let us consider linear momentum. Linear momentum, like angular momentum, is conserved if the sum of the external forces on the system is zero. Now, force is by definition what causes a change in momentum, so the fact that momentum is conserved if there are no external forces is fairly obvious. But, when a stationary object is hit by a moving object, the fact that the stationary object moves is because not because momentum is conserved, but because there is an internal force acting on the object. So, my question is this: when considering angular momentum, what causes the person in the chair to spin? It must be a torque (the rotational equivalent of force). But what is the source of the torque? Also, a perhaps related question, why is it hard to flip a spinning wheel? Can how hard it is to spin be calculated?


Could you please re-explain the scenario in your lab? I'm having trouble following it. Oh, and please remember to sign your posts using four tildes (~~~~). -mattbuck (Talk) 23:22, 7 May 2008 (UTC)[reply]
You're sitting in a chair that can spin, and you're holding a wheel (the wheel can spin; it has handles of sorts going through the center of the wheel). Someone spins the wheel (it is parallel to the floor), and the you flip it 180 degrees so that it's spinning in the opposite direction. Afterwards, you begin to spin. —Preceding unsigned comment added by 76.69.240.138 (talk) 23:30, 7 May 2008 (UTC)[reply]
I'm pretty sure it's Newton's 3rd law - equal and opposite forces. The spinning wheel has angular momentum, and when you flip it, the torque you provide changes the direction of the angular motion 180 degrees, so you receive an equal force in the opposite direction, causing you to spin, conserving (angular) momentum. —Akrabbimtalk 23:38, 7 May 2008 (UTC)[reply]
Yeah, the force making you spin is the reactive force to you moving the wheel. It's the same basic principle as rocket propulsion - sit on the same chair with a heavy weight and throw it away from you and the chair will move backwards. This is just the rotational equivalent. --Tango (talk) 00:35, 8 May 2008 (UTC)[reply]
Okay, I think I get it. But why is it hard to flip the wheel, and how can this be calculated? —Preceding unsigned comment added by 76.69.240.138 (talk) 00:53, 8 May 2008 (UTC)[reply]
You can think of it as the rotational analogue of Newton's 3rd law - essentially, when you try to flip the wheel, you are applying a torque to the wheel (torque is a change in momentum over time, just like force is a change in momentum over time). The wheel applies an equal and opposite torque to you! You may need a pencil and paper to see this properly - the angular momentum vector of the wheel is initially vertical. As you flip the wheel, you're changing the angular momentum vector, and the torque is in the direction of that change; as you flip the wheel, there is a torque with some component in the vertical direction, and the equal and opposite torque on you and your chair produces your angular acceleration. It is "hard" to flip the wheel for the same reason it's "hard" to throw the wheel across the room to give yourself linear momentum - both you and the wheel have rotational inertia! Using the equation in our article on precession (look in the torque-induced precession section), the net torque causing you to spin will be where is the angular speed of your flipping motion, is the spin angular speed of the wheel, is the angular moment of the wheel, and is the angle of the axis of the wheel to the vertical. It is possible to think of the forces "microscopically", i.e. in terms of the forces on each part of the wheel but it is pretty tedious in my opinion. If you want to see that treatement, there's a decent one in the aforementioned article on precession. --Bmk (talk) 01:30, 8 May 2008 (UTC)[reply]
Got it! Thanks a lot. —Preceding unsigned comment added by 76.69.240.138 (talk) 02:49, 8 May 2008 (UTC)[reply]

Momentum of a photon

How can a photon, which is massless, have momentum? —Preceding unsigned comment added by 76.69.240.138 (talk) 23:14, 7 May 2008 (UTC)[reply]

A photon has zero rest mass. However, under special relativity, accelerating particles gain mass. If you were to accelerate a massive particle to the speed of light, its mass would become infinite. Hence it is necessary for a photon to be "massless" for it to be able to travel at the speed of light. Confusing Manifestation(Say hi!) 23:26, 7 May 2008 (UTC)[reply]
You've probably heard of the equation "E=mc2". It describes an equivalence between mass and energy - that equivalence goes a further than that simple formula (which is the special case for objects at rest - a photon isn't at rest, since it always moves at the speed of light). While a photon has zero rest mass, it has a positive energy, and that gives it a momentum. That momentum (in a vacuum, at least) is where h is the Plank constant and lambda is the wavelength. --Tango (talk) 23:32, 7 May 2008 (UTC)[reply]
I never thought about that. that's messed up. --Shaggorama (talk) 10:14, 8 May 2008 (UTC)[reply]
Relativity can be rather counter intuitive... For example, even though photons are massless, they are still affected by gravity in the same way as anything else (a beam of light on Earth will fall at 9.81m/s/s - it's hard to tell, since it's out of sight before it's fallen a noticeable amount, but it does fall). --Tango (talk) 13:28, 8 May 2008 (UTC)[reply]
[repost from old question] Be careful with that equivalence! As described at general relativity, light suffers twice the deflection that one would naïvely expect from analogy with a fast rock. --Tardis (talk) 15:30, 8 May 2008 (UTC)[reply]
Interesting... what causes the difference between a very light particle travelling at 0.999999c and a photon? --Tango (talk) 20:18, 8 May 2008 (UTC)[reply]
I'm confused. I was told once that it was one half the expected value. Also you might find this interesting-http://www.experiencefestival.com/a/Newtons_aether_model_-_Modernised_Newtonian_theory/id/5333093Em3ryguy (talk) 20:41, 8 May 2008 (UTC)[reply]

centripetal motion of car

I was given a question where a car of known mass was turning on a circular road of known radius and coefficient of static friction. The question then asks what's the maximum velocity of the car. That wasn't to difficult, but I have a theoretical question. In this question, I treated the centripetal force as the static friction force, but for this to work the static friction force should be pointing to the center of the road. But it's pointing along the path of the car. Then, how can static friction cause centripetal motion. Wouldn't we need the angle of the tires to treat friction vectorally? —Preceding unsigned comment added by 76.69.240.138 (talk) 23:25, 7 May 2008 (UTC)[reply]

The scope of the question seems to be assuming that the tires are rolling ideally, that is to say no slipping. The torque the axle provides causes the tire to push backwards on the road. Since the wheel is not slipping, that force is solely static friction, as the rubber is not necessarily moving in relation to the road, as each infinitesimal point of the tire touches down on the road in an instant, pushes back, and then is removed from the road, as the tire rolls. —Akrabbimtalk 23:44, 7 May 2008 (UTC)[reply]
I don't think you've understood my question; perhaps I was unclear. I know why the car moves. It is a simple application of Newton's third law. My question is why we treat the centripetal force as equal to μ*m*g. After all, isn't the friction force not directed at the center of the circle of motion. —Preceding unsigned comment added by 76.69.240.138 (talk) 00:18, 8 May 2008 (UTC)[reply]
I think it has to do with the fact that the tires aren't pointing in the direction of travel. I'm not sure about the details, though... My attempt to work it out ended up with the car going in the wrong direction, so I'm clearly missing something... --Tango (talk) 00:31, 8 May 2008 (UTC)[reply]
The car is observed to be moving in a circle at constant speed. Therefore, the car is constantly accelerating toward the center of the circle. Therefore, there a is force acting on the car and the force is acting in the diretion of the center of the circle. The only force acting on the car (in a pure gedankenexperiment) is the friction between the roadway and the tires. Therefore, the direction of this force must be toward the center of the circle. Conclusion: the vector of the force is in this case perpendicular to the direction of motion. -Arch dude (talk) 02:03, 8 May 2008 (UTC)[reply]
From this, we can see that your "obvious" assertion (that the friction force is in the direction of motion), is incorrect. -Arch dude (talk) 02:12, 8 May 2008 (UTC)[reply]

May 8

Taste vs. Smell

I read our articles on "the 5 senses."

When i get sick, I can't really taste anything. Does this mean that taste is a sub-set of the sense of smell? Are the two linked in some way? If I smell something foul, am I also tasting particles of it?

Can those without a sense of smell still taste, what about the inverse?

Thanks-- —Preceding unsigned comment added by 24.218.24.148 (talk) 00:22, 8 May 2008 (UTC)[reply]

Apparently if you eat an orange with a banana under your nose, the orange tastes like a banana. Don't know where I read that, sorry. LHMike (talk) 00:28, 8 May 2008 (UTC)[reply]
There is definitely a strong relationship between taste and smell. Just try eating something while holding your nose. I don't know the details, though, really... They are the same basic thing, though - both involve detecting certain molecules, either in the air or in things we eat and drink. --Tango (talk) 00:30, 8 May 2008 (UTC)[reply]
The taste+smell thing is called flavor. See the article for loads of info. — Kieff | Talk 01:43, 8 May 2008 (UTC)[reply]
You may be interested in a recent profile of noted chef Grant Achatz, who has partially lost his sense of taste due to cancer. The article discusses the relationship of scent and taste in some detail. -- Coneslayer (talk) 17:56, 8 May 2008 (UTC)[reply]

Decibels

I read about a set of billboards that show the noise level in decibels of the surrounding area. From an acoustics class I took, I remembered there were a few ways to measure sound levels in decibels. After reviewing the relevant articles in Wikipedia, I deduced the decibel numbers on the billboards are likely sound pressure levels, maybe unweighted or using A-weighting.

When measuring street noise, how large of a difference would there be between the different weightings?

Also in terms of street noise, how much does the location of the sensor affect the decibel reading?

--Bavi H (talk) 00:45, 8 May 2008 (UTC)[reply]

dB (decibel) is a ratio, not an absolute measure of sound. Thus, it is valid to say that "ambient sound increased by 20dB" (i.e, there is 100 times more acoustic power) without specifying any particular absolute level. To specify an abolute level, you must relate the level to a particular absolute standard: 0dB(SPL) is the smallest (absolute) acoustical energy perceptible to a (statistically average) human, and 20dB(SPL) is 100 times as much acoustical energy. The various alternative measures differ in the ways they assign weights to the frequencies in the acoustic spectrum. Each such weighting is a convention that derives from some perspective, not a mathematical or physical fact. The only objective measure of sound at a particular location would be a spectral graph, and even that is only valid for a particular integration period.-Arch dude (talk) 01:35, 8 May 2008 (UTC)[reply]

Expanding Universe

Thanks everyone for the help so far! But there's still something I'm missing: if the universe is expanding, what is in the space before the universe expands into it? Is it impossible to know how far that area on non-universe goes or if there's something on the other side? Evaunit♥666♥ 00:52, 8 May 2008 (UTC)[reply]

Generally, the Metric expansion of space is defined by the objects in the universe, and the distance between objects. Said simply, the expanding universe isn't expanding into anything, it is simply increasing the distance between objects. -- JSBillings 00:58, 8 May 2008 (UTC)[reply]
Take a non-inflated balloon. Draw some dots on it. Now blow it up and watch the dots - they move apart as the balloon expands. There isn't an edge of the balloon (considered as just a 2D surface) that's expanding into anything, the expansion is caused by the balloon itself stretching at every point. The universe is the same, just with one more dimension. The only problem with the analogy is that the balloon is a 2D surface existing inside our 3D world - the universe doesn't exist within anything, it simply is, so you need to ignore all of the world except the balloon. Consider what an ant living on the balloon would see. --Tango (talk) 02:04, 8 May 2008 (UTC)[reply]
Here's a very concrete thought experiment you can use to understand this concept: Send an astronaut somewhere into space with a stopwatch and a flashlight, and put a mirror very far away from him/her, such that the astronaut and the mirror are stationary with respect to each other. Have the astronaut shine the flashlight at the mirror, and time how long it takes for the light to make the round trip. When cosmologists say "space is expanding", it means that if the astronaut repeats the experiment over time, the astronaut will measure that the round trip time is increasing!! There are a few tricky issues about making the mirror and astronaut "stationary", but they aren't too difficult to figure out. That's basically all we know about this concept of "expanding space". It's anyone's guess what it really means - physicists have come up with an intuitive description of the concept, and a comprehensive mathematical formulism (see general relativity) that seems to describe it. -Bmk (talk) 02:23, 8 May 2008 (UTC)[reply]
PS: They have to be very far apart wait a very long time in between trials for a measurable effect (i.e. not on humanly accessible distances or time scales). --Bmk (talk) 04:48, 8 May 2008 (UTC)[reply]
Mu (negative) --Shaggorama (talk) 10:17, 8 May 2008 (UTC)[reply]

Centrifuge

Seeing as we are now in the era of unmanned space travel for a while, why would it not be a good idea to build a centrifuge in orbit around the earth and use it to catapult unmanned space ships at extreme velocity, thus shortening the time for travel to other planets? Also, as we will be re-entering the 'manned space race to unearthly bodies' in the near future, what effect would this system have on astronauts, considering they are already in practically zero gravity? Would the 9Gs or more affect them? My question is, in a zero gravity environment, how would this work, if at all?--ChokinBako (talk) 03:30, 8 May 2008 (UTC)[reply]

A centrifuge, flinging a ship with that much velocity, would itself be flung back towards to Earth with tremendous speed, would it not? I can't envision how this will work... 206.126.163.20 (talk) 03:37, 8 May 2008 (UTC)[reply]
Well, you could fling two similar ships in opposite directions without altering the centrifuge station's trajectory much, but I don't really see how this system could become more feasible than rockets. Perhaps it could be valuable for unmanned missions, but according to our article on g-force, NASA recommends limiting human acceleration to around 10 Gs for a few minutes of exposure. A centrifuge that is - let's be very generous - 500 m in radius (that's a kilometer in diameter!) with a centripetal acceleration at the edge of 10 Gs would only send a spaceship flying off at 220 m/s (that's 500 mph), which is not terribly impressive considering unmanned spacecraft going to mars average speeds in the 30,000 m/s (75,000 mph) range. Unfortunately, the speed goes up with the square root of radius and square root of acceleration, so if you double the permissible acceleration and double the radius (which seems improbable to me), you only get 1000 mph exit velocity. I think moon-based linear accelerators (using a rail gun or something) for unmanned craft is more feasible. --Bmk (talk) 04:38, 8 May 2008 (UTC)[reply]
Additionally, how are you putting the spacecraft on the centrifuge to begin with? It seems that your options are to either stop and restart the centrifuge or accelerate the craft up to centrifuge speed, neither of which suggests an advantage over just launching the craft normally. — Lomn 12:15, 8 May 2008 (UTC)[reply]
You could load them in at the center, then feed them down the length of the arm from there. Also the 2 arms don't have to be the same length, one side could be shorter but more massive, but you still wouldn't even be able to get velocities out of it that would even be close to the orbital velocity of the centrifuge, and if Bmk's correct it would still be going less than escape velocity. Mad031683 (talk) 15:14, 8 May 2008 (UTC)[reply]
It might be easier to use a Gravitational slingshot, though I couldn't say for certain as I don't have much expertise in this area. AlmostReadytoFly (talk) 13:18, 8 May 2008 (UTC)[reply]

Pls, helping to answer this question !?

What is the difference between macronutrients and micronutrients? Will the consequences of depriving a plant of a micronutrient be less severe than those of depriving the plant of a macronutrient? —Preceding unsigned comment added by 121.1.58.38 (talk) 03:51, 8 May 2008 (UTC)[reply]

I would recommend you read wikipedia's article on micronutrients. A macronutrient is simply a nutrient which is required in large quantities to sustain life, like water, carbon dioxide, and nitrates for plants. It's hard to answer your second question precisely - can you be more specific? I would suspect removing water or air supply will kill a plant much more quickly than removal of the supply of most micronutrients. --Bmk (talk) 04:22, 8 May 2008 (UTC)[reply]

First of all I would like to differ and negate the answer given above. It was wrongly dealt with. However, to answer your question, both micro and macro nutrients can be defined as essetial elements required for the plant growth and defficiency of which leads to certain metabolic disorders having definite symptoms. Only difference is that micronutrients are necessary in micro quantity while macro nutrients are necessary in large quantity for the plants growth. Carbon, Oxygen, Nitrogen, Hydrogen, Calcium, Magnaseum, Phosphorus, Potassium and Sulfur are macro nutrients, while Iron, Chlorine or Chloride, Zinc, Molybdenum, Manganese, Copper, Boron are major micronutrients. Sometimes Iron is also kept in the list of macro nutrients. So, difficiency of any of any of the nutrients will be similarly sever, and that will have some definite symptoms. —Preceding unsigned comment added by Dr.Rajarshi (talkcontribs) 05:14, 8 May 2008 (UTC)[reply]

Organic Matter

Organic matter is not advisable to be added to paddy soils. Yes or No? Why? —Preceding unsigned comment added by 121.1.58.38 (talk) 03:59, 8 May 2008 (UTC)[reply]

Decomposition#Plant decomposition, Eutrophication and Hypoxia (environmental) might help. --71.236.23.111 (talk) 06:17, 8 May 2008 (UTC)[reply]

Pic

At 2:05 in [34], there's a picture of a cat with crazy blue eyes. Is that really possible? Black Carrot (talk) 05:23, 8 May 2008 (UTC)[reply]

No, it is photoshopped, to make the kitten look like someone who takes spice. --98.217.8.46 (talk) 20:22, 8 May 2008 (UTC)[reply]

Gingiva Recession

My gums are receding. My dentist says it's probably genetic. But I did not tell my dentist that my sister and mom have recession (see below).

My sister has recession, and I believe she has been flossing once and brushing twice a day. I never had recession, and my dental hygenine is the same. I think I am actually noticing recession. Is that possible? Because I haven't changed how I brush and floss to months ago, but it seems like my gums are receding faster. Also, my mom has recession, but she doesn't floss every day, but I think she at least brushes once a day.

Why can't recession be reversed, as in, why can't I build up bone growth again? How come I lose bone growth and can't gain it back? What are the causes of genetic gum recession? And have there been research to determine if bone can be rebuilt (in terms of gum recssion)? If not, why can't bone be rebuilt (in terms of gum recssion)? And what new research is being done to see if bone can be rebuilt? What are the new techniques? Thanks so much for all your help.68.148.164.166 (talk) 02:45, 7 May 2008 (UTC)68.148.164.166 (talk) 02:48, 7 May 2008 (UTC)68.148.164.166 (talk) 02:48, 7 May 2008 (UTC)[reply]

This is not medical advice and I'm no expert, but you might find more information on the following pages Receding gums, Periodontitis (look at the mention of risk factors there), Osteoimmunology As to your question why it can't be reversed (yet?) may be that despite surprising results (see Refdesk, "Humpty" May 2 ) our understanding of how cells work and what it takes to start and stop them from growing is still poorly understood. Particularly in dentistry, bone grafting and autologous (own) Stem cell treatments are areas with lots of studies on the way. Whether the results will be generally applicable, feasible and when they will reach common practice is another question. Lisa4edit (talk) 04:05, 7 May 2008 (UTC)[reply]
(see Refdesk, "Humpty" May 2 )???68.148.164.166 (talk) 09:40, 8 May 2008 (UTC)[reply]
Look particularly at the Scientific American article linked [35] I hope this works. If not click "Archive" above and look at May 2 in the Science desk archive. 71.236.23.111 (talk) 19:51, 8 May 2008 (UTC)[reply]

subliminal masseging

how to make subliminal messaging to subconcious mind —Preceding unsigned comment added by 117.197.48.133 (talk) 17:09, 8 May 2008 (UTC)[reply]

Subliminal message? Not sure what you're asking. --Ouzo (talk) 17:45, 8 May 2008 (UTC)[reply]
As a note, subliminal massaging would be, well, not that much fun, I imagine. Or would it? --98.217.8.46 (talk) 18:03, 8 May 2008 (UTC)[reply]
Really? Then why do you feel so good today? Nil Einne (talk) 19:34, 8 May 2008 (UTC)[reply]
Let's just get that nice spot right between your shoulder blades. Isn't that nice? Just relax, and let my hands take care of the work.
Huh? I don't know what you are talking about. There's no such thing. Obvious nonsense, that's it. --98.217.8.46 (talk) 20:24, 8 May 2008 (UTC)[reply]
It was once 'obvious nonsense' to propose the Earth was round. Don't dismiss things, unless you have proof to the contrary. Regards, CycloneNimrodTalk? 20:29, 8 May 2008 (UTC)[reply]
Please then, share your subliminal massaging technique. 24.76.169.85 (talk) 20:52, 8 May 2008 (UTC)[reply]

Validity of Sigmund Freud's work

From a scientific standpoint, how accurate or valid are Freud's theories? In particular, he introduced the world to the concepts of id, ego and super-ego, the unconscious, and others. Are these measurable? If so, how? If, not, why do they still hold such prominence in western society? —Preceding unsigned comment added by 72.35.21.126 (talk) 20:42, 8 May 2008 (UTC)[reply]